You are on page 1of 279

DEPARTMENT OF MATHEMATICS

UNIVERSITY OF GHANA, LEGON

MATH 122: CALCULUS I

Lecture Notes and Exercises

by

Cartious Enyonyoge Kojo Aziedu


(CEKA)

February 13, 2016


Contents

1 Preliminaries: Review of Some Relevant Concepts on Functions. 8


1.1 Review of Functions . . . . . . . . . . . . . . . . . . . . . . . . . . . . . . . . . . . . . 8
1.1.1 Intervals on the Real Line . . . . . . . . . . . . . . . . . . . . . . . . . . . . . . 8
1.2 Some Standard Functions we Need in this Course . . . . . . . . . . . . . . . . . . . . . 9
1.2.1 Polynomial Functions . . . . . . . . . . . . . . . . . . . . . . . . . . . . . . . . . 9
1.2.2 Rational Functions . . . . . . . . . . . . . . . . . . . . . . . . . . . . . . . . . . 10
1.2.3 Power Functions . . . . . . . . . . . . . . . . . . . . . . . . . . . . . . . . . . . 10
1.2.4 Trigonometric Functions . . . . . . . . . . . . . . . . . . . . . . . . . . . . . . . 11
1.2.5 Exponential and Logarithmic Functions . . . . . . . . . . . . . . . . . . . . . . . 11
1.2.6 Piecewise Functions . . . . . . . . . . . . . . . . . . . . . . . . . . . . . . . . . . 12
1.2.7 Absolute-value Functions. . . . . . . . . . . . . . . . . . . . . . . . . . . . . . . 13
1.3 Composition of Functions . . . . . . . . . . . . . . . . . . . . . . . . . . . . . . . . . . 13
1.4 Odd and Even Functions . . . . . . . . . . . . . . . . . . . . . . . . . . . . . . . . . . . 13
1.4.1 Basic properties of Odd and Odd Functions . . . . . . . . . . . . . . . . . . . . 14
1.5 The Factor Theorem for Polynomial Functions . . . . . . . . . . . . . . . . . . . . . . . 14
1.6 Rationalisation of Surds . . . . . . . . . . . . . . . . . . . . . . . . . . . . . . . . . . . 15
1.7 Functional Equations. . . . . . . . . . . . . . . . . . . . . . . . . . . . . . . . . . . . . 15
1.8 Exercise. . . . . . . . . . . . . . . . . . . . . . . . . . . . . . . . . . . . . . . . . . . . . 16

2 Inverse Functions. 17
2.1 General Inverse Functions. . . . . . . . . . . . . . . . . . . . . . . . . . . . . . . . . . . 17
2.1.1 Existence of an Inverse of a Function. . . . . . . . . . . . . . . . . . . . . . . . . 17
2.1.2 Properties of Inverse Functions. . . . . . . . . . . . . . . . . . . . . . . . . . . . 18
2.1.3 Finding the Inverse of a Function. . . . . . . . . . . . . . . . . . . . . . . . . . . 18
2.2 Logarithmic and Exponential Functions. . . . . . . . . . . . . . . . . . . . . . . . . . . 20
2.3 Inverse Trigonometric Functions. . . . . . . . . . . . . . . . . . . . . . . . . . . . . . . 20
2.3.1 The Inverse Sine Function. . . . . . . . . . . . . . . . . . . . . . . . . . . . . . . 20
2.3.2 The Cosine Function. . . . . . . . . . . . . . . . . . . . . . . . . . . . . . . . . . 21
2.3.3 The Tangent Function and its Inverse. . . . . . . . . . . . . . . . . . . . . . . . 21
2.3.4 The Secant Function and its Inverse. . . . . . . . . . . . . . . . . . . . . . . . . 22
2.3.5 The Cosecant Function. . . . . . . . . . . . . . . . . . . . . . . . . . . . . . . . 23
2.3.6 The Cotangent Function. . . . . . . . . . . . . . . . . . . . . . . . . . . . . . . . 23

1
2.3.7 Some Inverse Trigonometric Identities. . . . . . . . . . . . . . . . . . . . . . . . 23
2.3.8 Finding Exact Values of Composite Functions with Inverse Trigonometric Functions. 24
2.3.9 Equations Involving Inverse Trignonometric Functions. . . . . . . . . . . . . . . 26
2.4 Summary on Inverse Functions. . . . . . . . . . . . . . . . . . . . . . . . . . . . . . . . 27
2.5 Exercises. . . . . . . . . . . . . . . . . . . . . . . . . . . . . . . . . . . . . . . . . . . . 28

3 Limits and Continuity of Functions. 29


3.1 Limit of a Function. . . . . . . . . . . . . . . . . . . . . . . . . . . . . . . . . . . . . . 29
3.1.1 The Intuitive Notion of a Limit . . . . . . . . . . . . . . . . . . . . . . . . . . . 29
3.1.2 One-sided Limits . . . . . . . . . . . . . . . . . . . . . . . . . . . . . . . . . . . 29
3.1.3 Evaluating Limits. . . . . . . . . . . . . . . . . . . . . . . . . . . . . . . . . . . 30
3.1.4 Analytical Evaluation of Limits: Laws of Limits. . . . . . . . . . . . . . . . . . . 36
3.1.5 Some Useful Limits Involving Trigonometric Functions . . . . . . . . . . . . . . 46
3.1.6 Limits Involving Inverse Trigonometric Functions. . . . . . . . . . . . . . . . . . 52
3.1.7 Infinite Limits: Vertical Asymptotes . . . . . . . . . . . . . . . . . . . . . . . . 55
f (x)
3.1.8 Limits at Infinity: General Rule for Evaluating lim . . . . . . . . . . . . . 59
x→∞ g (x)

3.2 Exercise on Limits. . . . . . . . . . . . . . . . . . . . . . . . . . . . . . . . . . . . . . . 62


3.3 Continuity of a Function . . . . . . . . . . . . . . . . . . . . . . . . . . . . . . . . . . . 66
3.3.1 The Intuitive Notion of the Continuity of a Function . . . . . . . . . . . . . . . 66
3.3.2 Types of Discontinuity. . . . . . . . . . . . . . . . . . . . . . . . . . . . . . . . . 73
3.3.3 Continuity on an Interval . . . . . . . . . . . . . . . . . . . . . . . . . . . . . . 78
3.3.4 Intervals of Continuity. . . . . . . . . . . . . . . . . . . . . . . . . . . . . . . . . 81
3.3.5 Intermediate Value Theorem (IVT) . . . . . . . . . . . . . . . . . . . . . . . . . 84
3.3.6 Application of the IVT: Locating Roots of Equations . . . . . . . . . . . . . . . 85
3.4 Exercises on Continuity. . . . . . . . . . . . . . . . . . . . . . . . . . . . . . . . . . . . 89

4 Differentiation. 92
4.1 The Concept of the Derivative of a Function . . . . . . . . . . . . . . . . . . . . . . . . 92
4.1.1 The Gradient of a Straight Line . . . . . . . . . . . . . . . . . . . . . . . . . . . 92
4.1.2 The Gradient to a Curve at a Point and the Derivative . . . . . . . . . . . . . . 92
4.1.3 Differentiation from First Principles. . . . . . . . . . . . . . . . . . . . . . . . . 93
4.1.4 Differentiability of a Function at a Point or on an Interval. . . . . . . . . . . . . 96
4.2 Algebraic Rules of Differentiation. . . . . . . . . . . . . . . . . . . . . . . . . . . . . . . 105
4.3 Differentiation of Composite Functions: The Chain Rule. . . . . . . . . . . . . . . . . . 110
4.4 Derivative of Some Standard Functions. . . . . . . . . . . . . . . . . . . . . . . . . . . . 112
4.4.1 Derivative Exponential Functions. . . . . . . . . . . . . . . . . . . . . . . . . . . 112
4.4.2 Derivatives of Trigonometric Functions. . . . . . . . . . . . . . . . . . . . . . . . 115
4.5 Derivatives of Inverse Functions. . . . . . . . . . . . . . . . . . . . . . . . . . . . . . . . 119
4.5.1 Differentiating General Inverse Functions. . . . . . . . . . . . . . . . . . . . . . 119

2 MATH 122: Calculus I - Kojoga Aziedu


4.5.2 Derivatives of Inverse Trigonometric Fuctions. . . . . . . . . . . . . . . . . . . . 122
4.5.3 Derivative of the Logarithmic Function. . . . . . . . . . . . . . . . . . . . . . . . 126
4.6 Deivatives of Implicit Functions. . . . . . . . . . . . . . . . . . . . . . . . . . . . . . . . 130
4.6.1 Definition and Examples. . . . . . . . . . . . . . . . . . . . . . . . . . . . . . . . 130
4.6.2 Procedure for Differentiating Implicit Functions. . . . . . . . . . . . . . . . . . . 130
4.7 Logarithmic Differentiation. . . . . . . . . . . . . . . . . . . . . . . . . . . . . . . . . . 132
4.7.1 Logarithmic Differentiation of y = (f (x))n and y = (f (x))g(x) . . . . . . . . . . . 132
f (x) g (x)
4.7.2 Logarithmic Differentiation of y = . . . . . . . . . . . . . . . . . . . . 133
h (x) k (x)
4.7.3 Derivatives of Functions in Parametric Forms. . . . . . . . . . . . . . . . . . . . 135
4.8 Higher-order Derivatives and the Formation of Differential Equations. . . . . . . . . . . 138
4.8.1 Higher Derivatives for Some Standard Functions. . . . . . . . . . . . . . . . . . 141
4.8.2 Forming Differential Equations. . . . . . . . . . . . . . . . . . . . . . . . . . . . 154
4.8.3 Differential Equations Involving nth Derivatives. . . . . . . . . . . . . . . . . . . 156
4.9 Exercises. . . . . . . . . . . . . . . . . . . . . . . . . . . . . . . . . . . . . . . . . . . . 157

5 Applications of Differentiation. 161


5.1 Critical (Stationary) Points (Numbers). . . . . . . . . . . . . . . . . . . . . . . . . . . . 161
5.2 Increasing and Decreasing (Monotone) Functions. . . . . . . . . . . . . . . . . . . . . . 162
5.2.1 Proof of Monotonicity of Functions. . . . . . . . . . . . . . . . . . . . . . . . . . 165
5.2.2 Determining the Intervals of Increase and Decrease of a Function. . . . . . . . . 166
5.2.3 Determining Positivity and Negativity of Functions. . . . . . . . . . . . . . . . . 168
5.2.4 Proof of Inequalities. . . . . . . . . . . . . . . . . . . . . . . . . . . . . . . . . . 170
5.2.5 Real World Applications of Monotonicity . . . . . . . . . . . . . . . . . . . . . . 171
5.2.6 Extreme Values (The Extrema). . . . . . . . . . . . . . . . . . . . . . . . . . . . 172
5.2.7 Classification of Critical Points. . . . . . . . . . . . . . . . . . . . . . . . . . . . 178
5.2.8 Optimisation Problems. . . . . . . . . . . . . . . . . . . . . . . . . . . . . . . . 182
5.3 Rolle’s Theorem. . . . . . . . . . . . . . . . . . . . . . . . . . . . . . . . . . . . . . . . 194
5.3.1 Applications of the Rolle’s Theorem. . . . . . . . . . . . . . . . . . . . . . . . . 196
5.4 Taylor Polynomials. . . . . . . . . . . . . . . . . . . . . . . . . . . . . . . . . . . . . . . 197
5.4.1 Maclaurin’s Expansions for Some Standard Functions. . . . . . . . . . . . . . . . 204
5.5 Linear Approximations and Differentials. . . . . . . . . . . . . . . . . . . . . . . . . . . 204
5.5.1 Linear Approximations. . . . . . . . . . . . . . . . . . . . . . . . . . . . . . . . 204
5.5.2 Differentials. . . . . . . . . . . . . . . . . . . . . . . . . . . . . . . . . . . . . . . 206
5.6 Rates of Change. . . . . . . . . . . . . . . . . . . . . . . . . . . . . . . . . . . . . . . . 209
5.7 Related Rates of Change: Use of the Chain Rule. . . . . . . . . . . . . . . . . . . . . . 213
5.8 Equations of the Tangent and Normal to a Curve. . . . . . . . . . . . . . . . . . . . . . 215
5.9 Curves Sketching . . . . . . . . . . . . . . . . . . . . . . . . . . . . . . . . . . . . . . . 223
5.9.1 Curve Sketching of Rational Functions. . . . . . . . . . . . . . . . . . . . . . . . 223
5.9.2 Sketching Polar Curves. . . . . . . . . . . . . . . . . . . . . . . . . . . . . . . . 229

3 MATH 122: Calculus I - Kojoga Aziedu


5.9.3 Families of Polar Curves. . . . . . . . . . . . . . . . . . . . . . . . . . . . . . . . 230
5.9.4 m . . . . . . . . . . . . . . . . . . . . . . . . . . . . . . . . . . . . . . . . . . . . 230
5.9.5 Finding Velocity of a Particle in Motion . . . . . . . . . . . . . . . . . . . . . . 230
5.9.6 Finding the Marginal Cost of a Commodity . . . . . . . . . . . . . . . . . . . . 230
5.10 Exercises. . . . . . . . . . . . . . . . . . . . . . . . . . . . . . . . . . . . . . . . . . . . 231

6 Antidifferentiation and Integration. 233


6.1 Antiderivatives. . . . . . . . . . . . . . . . . . . . . . . . . . . . . . . . . . . . . . . . . 233
6.2 The Indefinite Integral . . . . . . . . . . . . . . . . . . . . . . . . . . . . . . . . . . . . 234
6.2.1 Some Rules of Indefinite Integration. . . . . . . . . . . . . . . . . . . . . . . . . 234
6.2.2 Some Standard Antiderivatives. . . . . . . . . . . . . . . . . . . . . . . . . . . . 234
6.3 The Definite Integral. . . . . . . . . . . . . . . . . . . . . . . . . . . . . . . . . . . . . . 238
6.3.1 Some Rules of the Definite Integral. . . . . . . . . . . . . . . . . . . . . . . . . . 238
6.4 Methods of Integration. . . . . . . . . . . . . . . . . . . . . . . . . . . . . . . . . . . . . 243
6.4.1 Integration by Substitution. . . . . . . . . . . . . . . . . . . . . . . . . . . . . . 243
6.4.2 Trigonometric Substitutions. . . . . . . . . . . . . . . . . . . . . . . . . . . . . . 246
6.4.3 Integration by Parts. . . . . . . . . . . . . . . . . . . . . . . . . . . . . . . . . . 249
6.4.4 Integration of Rational Functions: The Method of Partial Fractions. . . . . . . . 249
6.4.5 Integration of Trigonometric Functions. . . . . . . . . . . . . . . . . . . . . . . . 252
6.5 Exercise. . . . . . . . . . . . . . . . . . . . . . . . . . . . . . . . . . . . . . . . . . . . . 259

7 Applications of Integrations. 261


7.1 Area Under a Curve. . . . . . . . . . . . . . . . . . . . . . . . . . . . . . . . . . . . . . 261
7.2 Volume of Solid (Surface) of Revolution. . . . . . . . . . . . . . . . . . . . . . . . . . . 265
7.3 Exercise. . . . . . . . . . . . . . . . . . . . . . . . . . . . . . . . . . . . . . . . . . . . . 267

8 First Order Ordinary Differential Equations. 268


8.1 Definitions and Terminologies. . . . . . . . . . . . . . . . . . . . . . . . . . . . . . . . . 268
8.2 Classifications of Differential Equations. . . . . . . . . . . . . . . . . . . . . . . . 269
8.2.1 Classification by Type. . . . . . . . . . . . . . . . . . . . . . . . . . . . . . . . . 269
8.2.2 Classification by Order. . . . . . . . . . . . . . . . . . . . . . . . . . . . . . . . . 270
8.2.3 Classification by Linearity. . . . . . . . . . . . . . . . . . . . . . . . . . . . . . . 270
8.2.4 Formation of differential equations . . . . . . . . . . . . . . . . . . . . . . . . . 270
8.3 First-order Differential Equations. . . . . . . . . . . . . . . . . . . . . . . . . . . . . . . 270
8.3.1 Methods of Solving First Order Differential Equations. . . . . . . . . . . . . . . 271
8.4 Applications of First Oder Differential Equations. . . . . . . . . . . . . . . . . . . . . . 274
8.4.1 Growth and Decay. . . . . . . . . . . . . . . . . . . . . . . . . . . . . . . . . . . 274
8.4.2 Inductance-Resistance Electrical Circuits . . . . . . . . . . . . . . . . . . . . . . 275
8.4.3 Motion of a Particle Under Gravity With Air Resistance . . . . . . . . . . . . . 276
8.4.4 Predator-prey Systems. . . . . . . . . . . . . . . . . . . . . . . . . . . . . . . . . 277
8.4.5 m . . . . . . . . . . . . . . . . . . . . . . . . . . . . . . . . . . . . . . . . . . . . 277

4 MATH 122: Calculus I - Kojoga Aziedu


Course Information and Outline (2015/16)

Course Information.

Course Description.

Course Objectives.

Learning Outcomes and Assessments.

Other Informations.

Course Outline.
1. Limits and Continuity of a Real-valued Function of a Single Real Variable.

• Limits of a real-valued function.


 Intuitive idea of the limit of a function.
 One-sided limits
 Rules for evaluting limits.
 Algebraic techniques for evalusting indeterminate limits.
 Limits of trigonometric and inverse trigonometric functions.
 Limits and infinity and their meaning; asymptotic behaviour.
• Continuity of a function.
 Intuitive idea of the continuity at a point.
 Continuity on an interval.
 Discontinuity at at point.
 Removable discontinuity.
 Intermediate-value theorem and its applications.

2. Differentiation.

• The Derivative
 The derivative as a limit - first principles.
 Algebraic rules of differentiation.
 The Chain Rule
 Power rule

• Derivatives of specific functions.


 Trigonometric functions.
 Logarirhmic and exponential functions.

5
• Implicit differentiation.
• Derivative of a function in parametric form.
• Increasing and decreasing functions.
• Critical points.
• Points of inflexion and concavity.
• Rolle’s Theorem.
• Applications to Taylor polynomials.

3. Applications of Differentiation.

• Use of differentials in approximation.


• Equations of tangents and normals to curves.
• Rates of change.
• Curve sketching of rational functions.
• Sketching polar curves.

4. Derivative of an Inverse Function.

• The trigonometric functions and inverses.


• The exponential function and its inverse.
• The functions ax and loga x, their derivatives and logarithmic differentiation.

5. Antidifferentiation and Integration.

• Antiderivatives.
 The indefinite integral.
 Some rules of indefinite integration.
 Some standard antiderivatives.
 The definite integral.
 Some rules of definite of the integral.
 The definite integral as a function - the Fundamental Theorem of Calculus I (FTC I).
 Evaluating definite integrals - the Fundamental Theorem of Calculus II (FTC II).
• Methods of integration.
 Integration by method of substitution.
 Integration of rational functions - use of partial fractions.

• Integration of trigonometric functions.


 Integration of products of trigonometric functions.
 Integration of powers of trigonometric functions.

• Trigonometric substitutions.
• Integration by parts.

6. Applications of Integration.

• Area under a curve.


• Volume of solid of revolution.

7. First Order Differential Equations.

6 MATH 122: Calculus I - Kojoga Aziedu


• Separable forms.
 Applications to:
• Laws of growth and decay.
• Newton’s law of cooling.
• Homogeneous form and integrating factor method.
 Applications to:
• Terminal velocity.

Recommended Textbooks

7 MATH 122: Calculus I - Kojoga Aziedu


Chapter 1

Preliminaries: Review of Some Relevant


Concepts on Functions.

1.1 Review of Functions


.
.
.

Look at Chapters P4, P5 of Adams for more


on Functions
.
.
.
Let X and Y be non-empty sets, and f : X → Y a function. Then

• y = f (x) or f : x → y for x ∈ X and y ∈ Y .

• domain of f , Df , is all x ∈ X for which the function is defined.

• range of f , Rf = {y ∈ Y : y = f (x) , x ∈ X} .

• f is said to exist or to be defined at a point a if a ∈ Df .

• f is injective (one-to-one) if ∀a, b ∈ X with a = b ⇒ f (a) = f (b) or f (a) 6= f (b) ⇒ a 6= b.

• f is surjective (onto) if ∀b ∈ Y , ∃a ∈ X such that b = f (a).

1.1.1 Intervals on the Real Line


Definition 1.1. Intervals on the Real Line.
An interval is a subset of the real line that contains at least two numbers and all real numbers therein.

In this course, most of the domains we shall be considering are intervals on the real line. There are four
(4) of such types. Suppose a, b ∈ R with a < b. Then the four intervals are:

8
• Open interval from a to b; a and b excluded: Consists of all x ∈ R such that a < x < b. It
is denoted (a, b).

Figure 1.1: Open interval (a, b) ≡ a < x < b

• Closed interval from a to b; a and b included: Consists of all x ∈ R such that a ≤ x ≤ b. It


is denoted [a, b].

Figure 1.2: Closed interval [a, b] ≡ a ≤ x ≤ b

• Open interval from a to b, a included but b excluded: Consists of all x ∈ R such that
a ≤ x < b. It is denoted [a, b).

Figure 1.3: Half-open (or half-closed)interval [a, b) ≡ a ≤ x < b

• Open interval from a to b; a excluded but b included: Consists of all x ∈ R such that
a < x ≤ b. It is denoted (a, b].

Figure 1.4: Half-open (or half-closed) interval (a, b] ≡ a < x ≤ b

1.2 Some Standard Functions we Need in this Course

1.2.1 Polynomial Functions


• These are functions of the form
0
X
f (x) = ar x r
r=n
≡ an xn + an−1 xn−1 + an−2 xn−2 + · · · + a1 x + a0
where r, n ∈ {0} ∪ N and ai are real constants.
• The integer n, which is the highest power of f , is called the degree of the polynomial; written
deg (f (x)) = n.
• Examples of polynomials are

 f (x) = a; its degree is 0, and is called a constant polynomial.


 f (x) = ax + b; its degree is 1, and is called a linear polynomial.
 f (x) = ax2 + bx + c; its degree is 2, and is called a quadratic polynomial.
 f (x) = ax3 + bx2 + cx + d; its degree is 3, and is called a polynomial polynomial, etc.

• Note that the sum and product of two or more polynomials is still a polynomial.

9 MATH 122: Calculus I - Kojoga Aziedu


1.2.2 Rational Functions
• These are functions of the form
N (x)
f (x) =
D (x)
where N (x) and D (x) are polynomials in x.

• f (x) is said to be proper if deg (f (x)) < deg (g (x)), otherwise, it is improper.

• If f (x) is improper, then we can express it as

R (x)
f (x) = Q (x) + ,
D (x)

where Q (x), called the quotient, is a polynomial, and R (x) called the remainder. In this case,
R (x)
is a proper rational function.
D (x)
• Examples are

x+1
 f (x) = ,
x−1
1
 f (x) = 2 ,
x −1
x2 − 1
 f (x) = x − 1 = ,
x+1
etc.

• Note that every polynomial is a rational function.

1.2.3 Power Functions


• In simple terms, power functions are functions of the form

f (x) = xn ,

where n ∈ R.

• If n ∈ N, then the power function becomes a polynomial.

• Examples include

 f (x) = x1/2 = x,
x
 f (x) √ ,
x2 + 1
etc.

10 MATH 122: Calculus I - Kojoga Aziedu


1.2.4 Trigonometric Functions
• These are functions involving the sine and the cosine functions, as well as their algebraic combin-
ations.

• Examples are

 f (x) = cos x,
 f (x) = sin x,
 f (x) = tan x,
 f (x) = cos (csc x),
 f (x) = cot (sec x),
 
1
 f (x) = tan , x 6= 0,
x
 f (x) = sin 2x − 1,
 f (x) = cos2 x + sin x − 2x,

etc.

1.2.5 Exponential and Logarithmic Functions


• has domain R

• is positive

• is concave up

• passes through the point (0, 1)

• is either increasing for a > 1 or decreasing for 0 < a < 1

• m

Go back to photocopied note


m
m
m

• Let a > 0 and x ∈ R. Then the function

f (x) = ax

is called the (general) exponential function.

• Then range of the exponential function is R+ .

• The graph of the the exponential function is

Figure 1.5: Graph of the exponential and logarithmic functions.

• Suppose x > 0. Then the function


f (x) = loga x
is called the (general) logarithmic function.

11 MATH 122: Calculus I - Kojoga Aziedu


• If a = e, then the function
f (x) = ex > 0
is called the (natural) exponential function, and
• the function
f (x) = loge x := ln x
is called the (natural) logarithmic function.

1.2.6 Piecewise Functions


• Suppose a = x0 , x1 , · · · , xn = b is a finite sequence of ordered points in R. A function f defined
on [a, b] except possibly at some points xi (i = 0, 1, · · · , n), is called a piecewise function (also
called a case-defined function) on [a, b] if ∀i ∈ (0, 1, · · · , n), there exists a function fi on the
closed interval [xi−1 , xi ] such that f (x) = fi (x) on the open interval (xi−1 , xi ).
• Essentially, the graph of a piecewise function consists of 2 or more pieces defined by different
formulae, or by a single rule whose implementation changes with some subsets of the domain of
the function.
• That is, for example 

 f1 (x) if x ≤ x0 = a

f2 (x) if x0 < x < x1

f (x) = .

 ..

f (x) if x

n n−1 < x ≤ xn = b

is a splined function.
• The graph of a splined function is shown below:

Figure 1.6: Graph of a peicewise (splined) function.

12 MATH 122: Calculus I - Kojoga Aziedu


1.2.7 Absolute-value Functions.
• The function
f (x) = |x| ,
defined by (
x if x > 0
|x| =
−x if x < 0
is called the absolute-value function.

• Thus (
f (x) if f (x) > 0
|f (x)| =
−f (x) if f (x) < 0;
and so (
2
x + x − 1 = x2 + x − 1 if x2 + x − 1 > 0 ⇒ x2 + x > 1
− (x2 + x − 1) if x2 + x − 1 < 0 ⇒ x2 + x < 1.

1.3 Composition of Functions


• Let f and g be two functions defined by y = f (u) and u = g (x). Then function f (g (x)) or
(f ◦ g) (x) is called a composite function, or “function of a function.”

• m

• m

• m

1.4 Odd and Even Functions


Definition 1.2. Even and Odd Functions.
A function f is said to be even if f (−x) = f (x) and odd if f (−x) = −f (x) for all x in its domain.

Some Examples of Even and Odd Functions

• Even functions: f (x) = x2k ,k ∈ Z, cos x, etc.

• Odd functions: f (x) = x2k+1 , k ∈ Z, sin x, etc.

Theorem 1.3. Decomposition of Functions into Odd and Even Parts.


Let f (x) be any given function. Then the functions 12 (f (x) + f (−x)) and 12 (f (x) − f (−x)) are even
and odd functions respectively. That is, every function can be written as the sum of an even and an
odd function.

Proof. Let fe (x) = 21 (f (x) + f (−x)) and fo (x) = 12 (f (x) − f (−x)). Then
1
fe (−x) = 2
(f (−x) + f (x))
1
= 2
(f (x) + f (−x)) = fe (x) ,

13 MATH 122: Calculus I - Kojoga Aziedu


and

fo (−x) = 21 (f (−x) − f (x))


= − 21 (f (x) − f (−x)) = −fo (x) .

Now,

fe (x) + fo (x) = 21 (f (x) + f (−x)) + 12 (f (x) − f (−x))


= 21 (f (x) + f (−x) + f (x) − f (−x))
= f (x) ,

completing the proof.

1.4.1 Basic properties of Odd and Odd Functions


• The graphs of both even and odd functions exhibit symmetries;

 even functions are symmetric about the coordinate axes


 odd functions are symmetric about the origin.

• The sum (or difference) of two even functions is even, and any constant multiple of an even
function is even.
• The sum (or difference) of two odd functions is odd, and any constant multiple of an odd function
is odd.
• The product of two even functions is an even function but the product of two odd functions is an
even function. However, the product of an even function and an odd function is an odd function
• The quotient of two even or two odd functions is an even function, but the quotient of an even
function and an odd function is an odd function.
• The derivative of an even function is odd and the derivative of an odd function is even.

1.5 The Factor Theorem for Polynomial Functions


• This states that if f (x) is a polynomial in x and f (x) = 0, then (x − a) is a factor of f (x).
• Note that every polynomial with real constants can be factorised into linear and quadratic factors
• More importantly, for all n ∈ N

an − bn = (a − b) an−1 + an−2 b + an−3 b2 + · · · + abn−2 + bn−1




and
an + bn = (a + b) an−1 − an−2 b + an−3 b2 − · · · − abn−2 + bn−1 , when n is odd;


Specifically,
( n  n
if n is even

x2 − 1 x2 + 1
xn − 1 =
(x − 1) (x n−1
+x n−2
+x n−3
+ · · · + 1) if n is odd;

( n 2 n n √ n  n √ n
x 2 + 1 − 2x 2 = x 2 + 2x 4 + 1 x 2 − 2x 4 + 1 if n is even

n
x +1 =
(x + 1) (xn−1 − xn−2 + xn−3 − xn−4 + · · · − 1) if n is odd.

14 MATH 122: Calculus I - Kojoga Aziedu


• For example

x3 − 1 = (x − 1) x2 + x + 1


x3 + 1 = (x + 1) x2 − x + 1


x4 − 1 = x2 − 1 x2 + 1
 

= (x − 1) (x + 1) x2 + 1

2
x4 + 1 = x2 + 1 − x2
 √  √ 
= x2 − 2x + 1 x2 + 2x + 1 .

1.6 Rationalisation of Surds



• Surds are irrational numbers of the form a, where a > 0 is not a perfect square.

• Multiples of surds, and their sums with themselves as well as their sums, products and quotients
with other rational numbers are also surds.

• Where a surd which is a quotient contains a radical in the denominator, rationalisation seeks to
remove that radical from the denominator. That is
a √
 √ can be rationalise by multiplying both the numerator and denominator my b, i.e.
b
√ √
a a b a b
√ =√ ×√ = .
b b b b
√ √ √
 If the denominator is a binomial such as a + b or a + b, we can rationalise quotient
surds by multiplying√both the√numerator and the
√ denominator
√ √ with√ their conjugates, i.e.
the conjugate of a + b is a − b whiles that of a + b is a − b. That is, for example,

√ √
√ √ 
a a a a− b a a− b
√ √ =√ √ =√ √ ×√ √ = .
a+ b a+ b a+ b a− b a2 − b 2

• As we shall see later on, we can also rationalise the numerator of surds.

1.7 Functional Equations.


See hmtl file in Calculus I Folder

15 MATH 122: Calculus I - Kojoga Aziedu


1.8 Exercise.
1. m

2. m

3. Given that f (x) = x2 − 2x + 4, find f (−1), f (0), f (2), f (a) and f (a + 3).

4. Given that f (x) = x2 + 2, show that

f (a) f (b) − f (ab)


= 2.
f (a) + f (b)
 
1
5. Let f (x) = sin .
x

(a) Show that, for any integer n, " #


1
f =1
2n + 12 π


and " #
1
f = −1.
2n + 23 π


(b) Plot the points on the graph of f with x = ± π2 , ± 5π


2 2
, ± 9π .

6. m

16 MATH 122: Calculus I - Kojoga Aziedu


Chapter 2

Inverse Functions.

2.1 General Inverse Functions.


Definition 2.1. Inverse of a Function.
A function g : Dg → Rg is said to be the inverse of the function f : Df → Rf if and only if

Dg = Rf and Df = Rg ,

and
f (g (x)) = x ∀x ∈ Dg and g (f (x)) = x ∀x ∈ Df ,
where Df is the domain of f and Rf is the range of f , etc. We write g = f −1 and f = g −1 . Thus, the
definition of inverse says that,
f −1 (x) = y ⇔ f (y) = x.
Thus

f −1 (f (x)) = f −1 ◦ f (x) = x ∀x ∈ Df


and
f f −1 (x) = f ◦ f −1 (x) = x ∀x ∈ Df −1 .
 

2.1.1 Existence of an Inverse of a Function.


• If f is a bijective function on an interval [a, b]. Then f −1 exists and is bijective on [a, b].

• And every strictly monotone function is bijective.

• Hence, every monotone function has an inverse.

• m

• m

• m

17
2.1.2 Properties of Inverse Functions.

2.1.3 Finding the Inverse of a Function.


.
.

Modify
.
.
Given the function we want to find the inverse function, .
First, replace with y. This is done to make the rest of the process easier. Replace every x with a y
and replace every y with an x. Solve the equation from Step 2 for y. This is the step where mistakes
are most often made so be careful with this step. Replace y with . In other words, we’ve managed to
find the inverse at this point! Verify your work by checking that and are both true. This work can
sometimes be messy making it easy to make mistakes so again be careful.

Example 2.2. m
.
.
Dawkins, net
.
.

Solution.

Example 2.3. m

Solution.

Example 2.4. m

Solution.

Example 2.5. m

Solution.

Problem 2.6. Show that the function f (x) = 2x3 + 3x2 − 36x has no inverse on (−∞, ∞).

18 MATH 122: Calculus I - Kojoga Aziedu


Proof.

To prove that f (x) = 2x3 + 3x2 − 36x is non-invertible on (−∞, ∞), we either prove that f is not
one-to-one or f does not strictly increase nor decrease on (−∞, ∞). Now, suppose f is one-to-one,
then for all a, b ∈ (−∞, ∞) we have f (a) = f (b), i.e.
2a3 + 3a2 − 36a = 2b3 + 3b2 − 36b
⇒ a2 = b2
⇒a = ±b
i.e. a = b or a = −b.
In other words, f (a) = f (b) ; a = b and hence, f is not 1 − 1 on (−∞, ∞) and so non-invertible.

Problem 2.7. Show that if f (x) = xn , where n is odd, then f −1 exists.

Proof.
f (x) = xn ⇒ f 0 (x) = nxn−1 .
Suppose n is odd, then n = 2k + 1, k ∈ N.
∴ f 0 (x) = (2k + 1) x2k
2
= (2k + 1) xk
> 0 ∀x ∈ R.
Thus, f increases for all real numbers and so is one-to-one. Hence f −1 exists.
Problem 2.8. If f and g have respective inverses f −1 and g −1 , show that the composite function f ◦ g
has inverse (f ◦ g)−1 = g −1 ◦ f −1 .

Proof. .
Problem 2.9. For what values of the constants a, b, and c is the function
x−a
p (x) =
bx − c
is self-inverse.

Proof. First, the function p (x) will be self-inverse if p−1 (x) = p (x). Notice that
cx − a
p−1 (x) =
bx − 1
since p (p−1 (x)) = x. Now, if p (x) is self-inverse, then
p−1 (x) = p (x)
cx − a x−a
⇔ =
bx − 1 bx − c
⇔ (bx − 1) (x − a) = (cx − a) (bx − c)
⇔ b (1 − c) + c2 − 1 + a (1 − c)

= 0.

19 MATH 122: Calculus I - Kojoga Aziedu


⇔ b (1 − c) = 0, c2 − 1 = 0 and a (1 − c)
⇔ a = 0, b = 0 and c = 1.

That is, the function f (x) = −x is its own inverse (see graph).

2.2 Logarithmic and Exponential Functions.


Copy and paste from .........

2.3 Inverse Trigonometric Functions.

2.3.1 The Inverse Sine Function.


2.3.1.1 The Sine Function.

• The sine the function


sin : R → [−1, 1] .

• It is not one-to-one and so has no inverse.

Figure 2.1: The general and restricted sine functions.

20 MATH 122: Calculus I - Kojoga Aziedu


2.3.1.2 The Inverse Sine Function.
h π πi
For f (x) = sin x to to have an inverse, we restrict the domain to − , . Thus,
2 2
h π πi
sin : − , → [−1, 1]
2 2
has an inverse denoted by sin−1 or arcsin. Therefore,
h π πi
−1
sin : [−1, 1] → − , ⇒ y = sin−1 x ⇔ sin y = x,
2 2
π π
− ≤ x ≤ , −1 ≤ y ≤ 1.
2 2

2.3.2 The Cosine Function.


The sine function
cos : R → [−1, 1]
is also not one-to-one and so has no inverse.

Figure 2.2: The general and restricted cosine functions.

2.3.2.1 The Inverse Cosine Function.

For f (x) = cos x to to have an inverse, we restrict the domain to [0, π]. Thus,

cos : [0, π] → [−1, 1]

has an inverse denoted by cos−1 or arccos. Hence,

cos−1 : [−1, 1] → [0, π] ⇒ y = cos−1 x ⇔ cos y = x,


π π
− ≤ x ≤ , −1 ≤ y ≤ 1.
2 2

2.3.3 The Tangent Function and its Inverse.


The tangent function n π o
tan : R\ (2n + 1) , n ∈ Z → R
2

21 MATH 122: Calculus I - Kojoga Aziedu


Figure 2.3: The tangent function.
 π π
Restricting the domain to − , , the restricted function
2 2
 π π
tan : − , →R
2 2
has an inverse denoted by tan−1 or arcsin and defined as
 π π
−1
tan : R → − , ⇒ y = tan−1 x ⇔ tan y = x,
2 2
y ∈ R, − π2 < x < π2 .

2.3.4 The Secant Function and its Inverse.


2.3.4.1 The Secant Function.

.
.
.
.
.

2.3.4.2 The Inverse Secant Function.

Let |x| ≥ 1. Then


y = sec−1 x ⇔ sec y = x and y ∈ 0, π2 ∪ π2 , π .
  

22 MATH 122: Calculus I - Kojoga Aziedu


Figure 2.4: Graphs of y = sin x and sin−1 x for − π2 ≤ x ≤ π
2

2.3.5 The Cosecant Function.

Figure 2.5: Graph of and y = csc x for y ∈ 0, π2 ∪ π2 , π .


  

Let |x| > 1. Then


y = csc−1 x ⇔ csc y = x and y ∈ 0, π2 ∪ π2 , π .
  

2.3.5.1 The Inverse Cosecant Function.

Figure 2.6: Graph of and y = csc−1 x for y ∈ 0, π2 ∪ π2 , π .


  

2.3.6 The Cotangent Function.

Figure 2.7: Graph of y = cot x for x ∈ R.

Let x ∈ R. Then
y = cot−1 x ⇔ cot y = x and y ∈ (0, π).

2.3.6.1 The Inverse Cotangent Function.

Figure 2.8: Graph of y = cot−1 x for x ∈ R.

2.3.7 Some Inverse Trigonometric Identities.

(a) sec−1 x = cos−1 1


(d) cos−1 x + cos−1 (−x) = π

Theorem 2.10. x
, |x| ≥
1.
−1 −1 1
 (e) cos−1 x + sin−1 x = π2 , |x| ≤ 1
(b) csc x = sin x
, |x| ≥ 1.
π
(c) cot−1 x = tan−1 1 (f ) cot−1 x = − tan−1 x, x ∈ R.

, x 6= 0.
x 2

Proof. (a)

23 MATH 122: Calculus I - Kojoga Aziedu


(b)

(c)

(d)

(e)

(f)

2.3.8 Finding Exact Values of Composite Functions with Inverse Trigono-


metric Functions.
Suppose we want to find exact values for the expressions of a trigonometric function composed with an
inverse of another trigonometric function without resorting to a calculator. Thus suppose we are given
an expression of the forms
f −1 (f (θ)) ; f f −1 (θ) ,


where f ∈ {sin, cos, tan, sec, cot, csc}. To evaluate these,

• if θ is in the restricted domain of f , then


f −1 (f (θ)) = f f −1 (θ) = θ.


• otherwise find an angle φ within the restricted domain of f such that


f (φ) = f (θ) .
Then
f −1 (f (θ)) = f f −1 (θ) = φ.


Note the following: The composition of a trigonometric function and its inverse.

• sin sin−1 (x) = x only for −1 ≤ x ≤ 1.




• cos (cos−1 (x)) = x only for −1 ≤ x ≤ 1.


• tan (tan−1 (x)) = x only for −∞ ≤ x ≤ ∞.
• sin−1 (sin (x)) = x only for − π2 ≤ x ≤ π2 .
• cos−1 (cos (x)) = x only for − π2 ≤ x ≤ π2 .
• tan−1 (tan (x)) = x only for − π2 < x < π2 .
Example 2.11. Find the exact value of the following.

(a) sin−1 sin π3 (b) sin−1 sin 2π (c) cos−1 cos 2π


(d) cos−1 cos − 2π
   
3 3 3

Solution.

Example 2.12. Prove the following identities.


Find the truth set of the following equations.

24 MATH 122: Calculus I - Kojoga Aziedu


 √
(a) cos sin−1 x = 1 − x2 , |x| ≤ 1 (c) sin (cos−1 x) =

(b) cos (tan−1 x) = (d) tan (sec−1 x) =

Solution.

(a)

(b)

(c)

(d)

Example 2.13. Show that .............


Prove the following identities.
  ( √
x tan−1 x2 − 1 if x ≥ 1
(a) sin x = tan
−1 −1 √ if |x| < 1 (b) sec x =
−1 √
1 − x2 π − tan−1
x2 − 1 if x ≤ −1

Solution.

(a)

(b)

 √ 2 
x −1
sin −1
if x ≥ 1


x√
Problem 2.14. Prove that sec x =
−1  
x2 − 1
π − sin−1 if x ≤ −1.


x
Solution.
 
x
Example 2.15. Show that tan −1 −1
x = sin √ for all x.
1 + x2
Solution.
Example 2.16. Evaluate the following.

25 MATH 122: Calculus I - Kojoga Aziedu


√    
(a) csc−1 5 + cot−1 3 (d) cot sec−1 − √23 + csc−1 (−2)
  
(b) sin tan−1 √ x
x2 +1 (e) cot−1 cot − π4



(c) sin tan−1 x2 − 2x , x ≥ 2 (f)


Solution.

(a)

(b)

(c)

(d)

(e)

(f)

2.3.9 Equations Involving Inverse Trignonometric Functions.


Example 2.17. Find the truth set of the following equations.
(a) sin−1 x = cos−1 x (c) tan (cot−1 x) = 2 sin (cos−1 x)

(b) tan−1 (x + 1) + tan−1 (x − 1) = π


4
(d) cot−1 2 = cot−1 x + cot−1 7.

Solution.

(a)

(b)

(c)

(d)

Example 2.18. Find the truth set of the following equations.

26 MATH 122: Calculus I - Kojoga Aziedu


2x (c) x2 sec−1 (1 + x2 )
(a) sin−1
1 + x2
sin−1 x
(b) (1 + x ) tan
2 −1
x (d)
sin−1 2x

Solution.

(a)

(b)

(c)

(d)

Problem 2.19. Let t be the number such that −1 ≤ t ≤ 1. Show that


n π o n πo
(x, y) : sin−1 t ≤ x ≤ , t ≤ y ≤ sin x = (x, y) : t ≤ y ≤ 1, sin−1 y ≤ x ≤ .
2 2

Solution.

2.4 Summary on Inverse Functions.

27 MATH 122: Calculus I - Kojoga Aziedu


2.5 Exercises.
1. m

2. m

3. m

4. m

5. m

6. Let
ax + b
f (x) = .
cx + d
Show that f is invertive if and only if bc − ad 6= 0. Hence, given that bc − ad 6= 0, find f −1 and
determine the values of a, b, c, and d such that f = f −1 .

28 MATH 122: Calculus I - Kojoga Aziedu


Chapter 3

Limits and Continuity of Functions.

3.1 Limit of a Function.

3.1.1 The Intuitive Notion of a Limit


Definition 3.1. Limit of a Function
The number L is said to be limit of the function f (x) as x approaches a number a, and written as

lim f (x) = L,
x→a

if the value of f (x) can be made arbitrarily close to L by choosing x sufficiently close to, but not equal to,
a.

Alternatively, we can write lim f (x) = L equivalently as “f (x) → L” as x → a or “as x → a, f (x) → L.”
x→a
If the limit of the f (x) as x → a is L, then f (x) is said to converge to L as x → a.

Go on line and pix and redraw

Figure 3.1:

3.1.2 One-sided Limits


The limits of piecewise functions f (x) are evaluated by considering how x approaches approaches the
point of separation a. If we consider x to approach a from the left (i.e. we consider x < a), then we
write lim− f (x) = L− . Similarly, lim+ f (x) = L+ means that we approach a from the right, and so
x→a x→a
we considered x > a.
Consider (
f1 (x) if x < a
f (x) =
f2 (x) if x ≥ a
as shown in the diagram below.

29
Figure 3.2: One-sided limits

Then the following limits can be read from the graph of f as follows:

lim f (x) = M lim f (x) = N lim f (x) = L lim f (x) = L lim f (x) = L
x→a− x→a+ x→b− x→b+ x→b

Definition 3.2. Existence and Non-existence of Limits.

• A limit is said to exist if and only if both one-sided limits exist and are equal.

 That is
lim f (x) = L ⇔ lim− f (x) = L and lim+ f (x) = L. (3.1)
x→a x→a x→a

• The limit lim f (x) does not exist under any of the following conditions:
x→a

 f (x) approaches a different number from the right side of a than it approaches from the left
side of a; i.e.
lim− f (x) 6= lim+ f (x) .
x→a x→a

 f (x) increases or decreases without bound approaches a; i.e.

lim f (x) = ±∞.


x→a

 oscillates between two fixed values as approaches a.

3.1.3 Evaluating Limits.


In this course, we shall evaluate limits by three approaches, graphically, numerically and analytically.

3.1.3.1 Graphically Evaluation of Limits.

Here, all we do is to read the limit at a point from the graph of the function as the point is approached
from both left and right.

Example 3.3. Use the graphh of f (x) below to find following:

30 MATH 122: Calculus I - Kojoga Aziedu


f (0) f (4) lim f (x) lim f (x) lim f (x)
x→2− x→0+ x→0
f (2)
f (−2) lim f (x) lim f (x) lim f (x)
x→2+ x→2 x→0−

Solution. Notice that closed circle denotes where the graph is defined; hollow ones shows that portion
of the function is not defined there.

Value/limit of function Remark


f (0) = 4 There is a closed circle at (0, 4).
f (2) = undefined There is no closed circle there.
f (−2) = undefined There is no closed circle here either; vertical asymptote.
f (4) = 2 There is a closed circle at (4, 2).
1
lim+ f (x) = The y-value is approaching 1
2
as x approaches 2 from the right.
x→2 2
1
lim− f (x) = The y-value is approaching as x approaches 2 from the left.
1
2
x→2 2
1 1
lim f (x) = Since lim+ f (x) = lim− f (x) = .
x→2 2 x→2 x→2 2
lim+ f (x) = 4 The y-value is approaching 4 as x approaches 0 from the right.
x→0
1
lim− f (x) = The y-value is approaching 21 as x approaches 0 from the left.
x→0 2
1
lim f (x) does not exist Since lim+ f (x) = 4 6= = lim− f (x).
x→0 x→0 2 x→0
Example 3.4. Consider the function

a + bx
 if x > 2
f (x) = 3 if x = 2
if x < 2.

b − ax2

Determine the values of constants a and b so that lim f (x) exists.


x→2

Solution. Begin by computing one-sided limits at x = 2 and setting each equal to 3. Thus,

lim f (x) = lim+ (a + bx) = a + 2b = 3


x→2+ x→2

31 MATH 122: Calculus I - Kojoga Aziedu


and
lim− f (x) = lim− b − ax2 = b − 4a = 3.

x→2 x→2

We solve the system of equations for a and b. Thus,


5
a = 3 − 2b ⇒ b − 4 (b − 4a) = 3 ⇒ b =
3
and so  
5 1
a=3−2 =− .
3 3
Example 3.5. Let f be a function defined as

−8
 if x ≤ −6

3x + 10 if − 6 < x < −2



f (x) = −5 if x = −2
if

x2 −2<x≤3




if

−2x + 9 x > 3.
(a) Sketch the graph of f . (c) Show that lim f (x) 6= f (−2).
x→−2

(b) Show that lim f (x) = f (−6). (d) Show that lim f (x) does not exist.
x→−6 x→3

Solution.

(a) The sketch of the graph of f is shown in the diagram below.


[For now, refer to your graph sketch techniques in your MATH 121: Algebra and Trigono-
metry lecture.]

Figure 3.3:

From the graph,

32 MATH 122: Calculus I - Kojoga Aziedu


(b) lim f (x) = lim − (−8) = −8 (c) lim f (x) = lim − (3x + 10) = 4
x→−6− x→−6 x→−2− x→−6

lim f (x) = lim − (3x + 10) = −8. lim + f (x) = lim + x2 = 4.


x→−6+ x→−6 x→−2 x→−2

Therefore, lim f (x) = −8 = f (−6). Thus lim f (x) = 4 6= −5 = f (−2).


x→−6 x→−2

(d) lim f (x) = lim− x2 = 9


x→3− x→3
lim f (x) = lim+ (−2x + 9) = 3.
x→3+ x→3
Thus, lim− f (x) = 9 6= 3 = lim+ f (x) and so the limit does not exist.
x→3 x→3

Example 3.6. Let 



 7 if x < −5

−2x − 11 if − 5 < x ≤ −3



f (x) = 4 − x2 if −3<x<1
if

−2

 x=1

if 1 < x < 4 or x > 4

x2 − 4x + 5

be a function defined on (−∞, ∞).

(a) Sketch the graph of f . (d) Show that lim f (x) = f (−1).
x→−1

(b) What is f (−5)? Show that lim f (x) does (e) What is f (1)? Show that lim f (x) does not
x→−5 x→1
not exist. exist.

(c) Show that lim f (x) = f (−3). (f) What is f (4)? Does lim f (x) exist?
x→−3 x→4

Solution.

(a) The diagram below shows the graph of f . Note that the point x = 4 is not in the domain of f .

Figure 3.4:

(b) f (−5) = 7.
Now, lim − f (x) = lim − (7) = 7 and lim + f (x) = lim+ (−2x − 11) = −1.
x→−5 x→−5 x→−5 x→3

Since lim − f (x) = 7 6= −1 = lim + f (x), the limit does not exist.
x→−5 x→−5

33 MATH 122: Calculus I - Kojoga Aziedu


(c) Notice that −3 ∈ (−5, −3] and so f (−3) = −2 (−3) − 11 = −5. Now, from the graph,
lim − f (x) = lim − (−2x − 11) = −5 and lim + f (x) = lim − 4 − x2 = −5 ⇒ lim f (x) = −5.
x→−3 x→−3 x→−3 x→−3 x→−3
Hence, lim f (x) = −5 = f (−3).
x→−3

(d) Again, −1 ∈ (−3, 1), so f (−1) = 4−(−1)2 = 3. From the graph, lim − f (x) = lim − 4 − x2 = 5

x→−1 x→−1
and lim + f (x) = lim − 4 − x2 = 5. Thus lim f (x) = 5 = f (−1).

x→−1 x→−1 x→−1

(e) From the graph (even by definition of the function), f (1) = −2. Now lim− f (x) = lim− 4 − x2 = 3

x→1 x→1
and lim+ f (x) = lim+ x2 − 4x + 5 = 2 are not equal, lim f (x) does not exist.

x→1 x→1 x→1

(f) f (4) does not exist. However, f has the same definition both to the left and the right of x = 4,
i.e. f (x) = x2 − 4x + 5 around x = 4. Therefore,
lim f (x) = lim x2 − 4x + 5 = 5.

x→4 x→4

Example 3.7. Sketch the graph of f (x) and prove that lim f (x) does not exist, where
x→2
(
3 − 2x if x ≤ 2
f (x) =
x3 − 5 if x > 2

Solution.

Figure 3.5: Graph of f (x)

lim f (x) = lim (3 − 2x) = 3 − 2 (2) = −1


x→2− x→2
and
lim+ f (x) = lim x3 − 5 = (2)3 − 5 = 3.

x→2 x→2

Since lim− f (x) = −1 6= 3 = lim+ f (x), the limit does not exist.
x→2 x→2

3.1.3.2 Numerical Evaluation of Limits.

Numerical approaches to limits where the function is undefined are only capable of estimating heurist-
ically the correct value that a function approaches at the specified argument.
Example 3.8. Evaluate the following limits numerically.

34 MATH 122: Calculus I - Kojoga Aziedu


 
(i) lim x2 + 3 1

x→3 (v) lim sin
x→0 x
2x − 1
(ii) lim 3
x→0 x (vi) lim −
x→−2 x+2
ex − 1
(iii) lim (vii) lim tan x
x→0 x x→ π2

|x − 1| 2x + 1
(iv) lim (viii) lim
x→1 x − 1 x→−∞ 3x − 4

Solution. We shall compute these limits using tables.

x → 3− 3+ ← x
(i) x −3.1 −3.01 −3.001 −3.0001 3 3.0001 3.001 3.01 3.1
2
x +3 12.7 12.06 12.006 12.0006 12 12.0006 12.006 12.06 12.7
From the table, as x approaches 3 both from the left and the right, f approaches 12.
∴ lim x2 + 3 = 12.

x→3

x → 0− 0+ ← x
(ii) x −0.1 −0.01 −0.001 −0.0001 0 0.0001 0.001 0.01 0.1
2x − 1
0.6697 0.6908 0.6929 0.6931 Undefined 0.6931 0.6934 0.6956 0.7177
x
From the table, as x → 0± , f → 0.6931.
2x − 1
∴ lim = 0.6931 (= ln 2) .
x→0 x

x → 0− 0+ ← x
(iii) x −0.1 −0.01 −0.001 −0.0001 0 0.0001 0.001 0.01 0.1
ex − 1
0.9516 0.9950 0.0.9995 0.99995 Undefined 1.00005 1.0005 1.0050 1.0517
x
Here too, f → 1 as x → 0± . Hence
ex − 1
lim = 0.6931 (= ln 2) .
x→0 x

x → 0− 0+ ← x
x −1.1 −1.01 −1.001 −1.0001 1 1.0001 1.001 1.01 1.1
(iv)
|x − 1|
−1 −1 −1 −1 Undefined 1 1 1 1
x−1
From the table,
|x − 1| |x − 1|
lim− = −1; lim+ = 1.
x→1 x−1 x→1 x−1
|x − 1| |x − 1| |x − 1|
Thus lim does not exist since lim− = −1 6= 1 = lim+ .
x→1 x − 1 x→1 x−1 x→1 x−1
Notice that we have used the function
x − 1

|x − 1|  x − 1 = 1 if x − 1 ≥ 0 ⇒ x ≥ 1
= 1−x
x−1 
 = −1 if x − 1 < 0 ⇒ x < 1
x−1
to find the left and right hand limits.

35 MATH 122: Calculus I - Kojoga Aziedu


x → 0− 0+ ← x
(v) x  − π2 2
− 3π 2
− 5π 2
− 7π 0 2

2

2

2
π
sin x1 −1 1 −1 1 Undefined −1 1 −1 1
This limit does not exist since it oscillates between two numbers x = −1 and x = 1.
x → −2−
(vi) x −2.1 −2.01 −2.001 −2.0001 −2.00001 −2.000001 −2.0000001
3
−30 −300 −3000 −30000 −300000 −3000000 −30000000
x+2
From the diagram, as x gets closer and closer to −2 from the left, f gets negatively large without
bound. Thus
3
lim − = −∞
x→−2 x + 2

and so does not exist.


x → π2 − ≈ 1.5708−
(vii) x 1.5 1.57 1.5707 1.57079 1.570796
tan x 14.1 1255.8 10381.3 158057.9 3060023.3
With a similar reason as in (vi) above, we have that

lim tan = ∞.
x→ π2 −

−∞ ← x
(viii) x −1000000 −100000 −10000 −1000 −100 −10
2x + 1
0.66666 0.66665 0.6665 0.6654 0.6546 0.5588
3x − 4
Here, as x → −∞, f → 0.66666. Therefore,
2x + 1 2
lim = 0.66666 = .
x→−∞ 3x − 4 3

3.1.4 Analytical Evaluation of Limits: Laws of Limits.


Let k be a constant and suppose lim f (x) and lim g (x) exist. Then
x→a x→a

(a) lim k = k
x→a

(b) lim x = a
x→a

(c) lim [kf (x)] = klim f (x)


x→a x→a

(d) Sum rule: The limit of a sum of functions is the sum of the limits.

lim [f (x) ± g (x)] = lim f (x) ± lim g (x). (3.2)


x→a x→a x→a

Deductions from the sum rule:


 For any additional function h (x),

lim [f (x) + g (x) + h (x)] = lim f (x) + lim g (x) + lim g (x).
x→a x→a x→a x→a

Thus in general, for any k functions fi (x),


" k # k
X X
lim fi (x) = lim fi (x). (3.3)
x→a x→a
i=1 i=1

36 MATH 122: Calculus I - Kojoga Aziedu


 If fi (x) = f (x) ∀i = 1, · · · , k, then
" k # " k #
X X
lim fi (x) = lim f (x) = klim f (x) (3.4)
x→a x→a x→a
i=1 i=1

as in (c) above.

(e) Product rule: The limit of a product is equal to the product of the limits.

lim [f (x) · g (x)] = lim f (x) · lim g (x). (3.5)


x→a x→a x→a

Deductions from the Product rule:

 Suppose f (x) = g (x), then

lim [f (x) · f (x)] = lim f (x) · lim f (x)


x→a x→a x→a

or h i2
lim [f (x)]2 = lim f (x) . (3.6)
x→a x→a

Thus the limit of the square of a function is the square of the limit.
 By repeatedly applying the product rule, one can easily obtain that
h in
lim [f (x)]n = lim f (x) , n ∈ N. (3.7)
x→a x→a

 Thus if f (x) = x, then


lim xn = an . (3.8)
x→a

1
Also, by writing f (x) = [f (x)]2 2 and applying the above result, we have



1
1 1 2
  
1 2
×2 ×
p
lim f (x) = lim [f (x)] 2 = lim [f (x)] 2 2
x→a x→a x→a
  2    1
1 1 1 2
= lim [f (x)] 2 × 2 = lim [f (x)] 2 ×2
x→a x→a
"  #1  1
1 2 2

1 2
= lim [f (x)] 2 = lim [f (x)]2× 2
x→a x→a

 1 " #1
h i2  21 2 h i2× 1 2
=  lim f (x)  = lim f (x) 2
x→a x→a

h i 1 q
2
= lim f (x) = lim f (x). (3.9)
x→a x→a

Hence, ∀n ∈ N, hp i q
n
lim f (x) = n lim f (x).
x→a x→a

 Therefore, in general, h in
lim [f (x)]n = lim f (x) , n ∈ Q. (3.10)
x→a x→a

This is known as the power rule.

37 MATH 122: Calculus I - Kojoga Aziedu


 Let n = −1 in (3.10). Then
1 h i−1
lim [f (x)]−1 = = lim f (x)
x→a lim [f (x)] x→a
x→a
or
1 1
= .
lim [f (x)] lim f (x)
x→a x→a
This can equivalently be obtained from the quotient rule below.
(f) Quotient Rule: Provided lim g (x) 6= 0 the limit of a quotient is equal to the quotient of the
x→a
limits, i.e.
f (x) lim f (x)
lim = x→a .
x→a g (x) lim g (x)
x→a

(g) If f (x) is a polynomial then


lim f (x) = f (a) .
x→a

(h) If f (x) is a rational function then, for all a in the domain of f ,


lim f (x) = f (a) .
x→a

(i) lim |f (x)| = lim f (x) .

x→a x→a

(j) If f (x) ≥ 0 for all x in the domain of f , then lim f (x) ≥ 0.


x→a

Example 3.9. Evaluate the following limits using the limit laws:

(a) lim x2 + 3x − 2 x2 + 4x − 3

x→−3 (c) lim
x→−2 5x3 − 9
 
(d) 3 2 2

x−2 lim lim 4x + 6x h + 4xh
(b) lim 2 x→
1 h→−1
x→−1 x + 4x − 3 2

Solution.

(a)
lim x2 + 3x − 2 = lim x2 + lim (3x) − lim (2)
 
x→−3 x→−3 x→−3 x→−3
2

= lim x + 3 lim (x) − lim (2)
x→−3 x→−3 x→−3
2
= (−3) + 3 (−3) − 2 = 9 − 9 − 2 = −2.

(b)

x−2 lim (x − 2)
x→−1
lim 2
=
x→−1 x + 4x − 3 lim (x2 + 4x − 3)
x→−1
lim x − lim (2)
x→−1 x→−1
=
lim (x2 ) + lim (4x) − lim (3)
x→−1 x→−1 x→−1
lim x − lim (2) (−1) − 2
x→−1 x→−1
= = 2
lim (x2 ) + 4 lim x − lim (3) (−1) + 4 (−1) − 3
x→−1 x→−1 x→−1
−1 − 2 −3 1
= = = .
1−4−3 −6 2

38 MATH 122: Calculus I - Kojoga Aziedu


(c)

x2 + 4x − 3 lim x2 + 4 lim x − lim 3


x→−2 x→−2 x→−2
lim = lim
x→−2 5x3 − 9 x→−2 3
5 lim x − lim 9
x→−2 x→−2
4−8−3 −7 1
= = = .
5 (−8) − 9 −49 7

(d)
   
3 2 2 3
+ lim 6x h + lim 4xh2
2
   
lim lim 4x + 6x h + 4xh = lim lim 4x
1 h→−1 1 h→−1 h→−1 h→−1
x→ x→
2 2

= lim 4x3 + 6x2 (−1) + 4x (−1)2 = lim 4x3 − 6x2 + 4x


   
1 1
x→ x→
2 2
 3  2  
3
 2
 1 1 1
= lim 4x − lim 6x + lim (4x) =4 −6 +4
x→
1
x→
1
x→
1 2 2 2
2 2 2
4 6 4
= − + = 1.
8 4 2

Example 3.10. If lim f (x) = −4 and lim g (x) = 3, evaluate


x→3 x→3
2 3
(b)
  p
f (x) + x lim x2 − 4f (x) + 11
(a) lim x→3
x→3 3g (x) + 1

Solution.

(a)
!3

f (x) + x2
3 
f (x) + x2
3 lim (f (x) + x2 )
x→3
lim = lim =
x→3 3g (x) + 1 x→3 3g (x) + 1 lim (3g (x) + 1)
x→3
2 !3
2 3
lim f (x) + lim (x )
!
x→3 x→3 −4 + (3)
= =
3lim g (x) + lim (1) 3 (3) + 1
x→3 x→3
 3  3  3
−4 + 9 5 1 1
= = = = .
9+1 10 2 8

(b)
p q
lim x2 − 4f (x) + 11 = lim (x2 − 4f (x) + 11)
x→3 x→3
q
= lim (x2 ) − 4 lim f (x) + lim (11)
x→3 x→3 x→3
q √
= (3)2 − 4 (−4) + 11 = 9 + 16 + 11

= 36 = 6.

Example 3.11. Suppose the limits of the functions f (x) and g (x) exist, and that lim [f (x) + g (x)] = 2
  x→a
f (x)
and lim [f (x) − g (x)] = 1. Find lim [f (x) · g (x)] and lim .
x→a x→a x→a g (x)

39 MATH 122: Calculus I - Kojoga Aziedu


Solution. Given that

lim [f (x) + g (x)] = 2 ⇒ lim f (x) + lim g (x) = 2 (3.11)


x→a x→a x→a

and
lim [f (x) − g (x)] = 1 ⇒ lim f (x) − lim g (x) = 1. (3.12)
x→a x→a x→a

We solve these two equations simultaneously to find the values of lim f (x) and lim g (x). i.e.
x→a x→a
3
(3.11) + (3.12): 2 lim f (x) = 3 ⇒ lim f (x) = .
x→a x→a 2
1
(3.11) − (3.12): 2 lim g (x) = 1 ⇒ lim g (x) = .
x→a x→a 2
Therefore
3 1 3
lim [f (x) · g (x)] = lim f (x) · lim g (x) = × =
x→a x→a x→a 2 2 4
and
  lim f (x)
f (x) 3 1
lim = x→a = ÷ = 3.
x→a g (x) lim g (x) 2 2
x→a

3.1.4.1 Theorems of Limits.

We shall state the following important theorems on limits without proof.

Theorem 3.12. Order Preservation Theorem


Suppose f (x) ≤ g (x) for all x in the domain of f except possibly at a. Then lim f (x) ≤ lim g (x).
x→a x→a

Theorem 3.13. Squeeze (Sandwich/Pinching) Theorem.


Suppose f (x) ≤ g (x) ≤ h (x) for all x near a. If lim f (x) = lim h (x) = L, then lim g (x) = L.
x→a x→a x→a
 
1
Example 3.14. Show that lim x sin
2
= 0.
x→0 x

Solution. From the properties of the function,


 
1
−1 ≤ sin ≤ 1.
x

Thus  
2 21
−x ≤ x sin ≤ x2 .
x
But lim −x2 = lim x2 = 0. Hence, from the Sandwich Theorem, we have that

x→0 x→0
 
2 1
lim x sin = 0.
x→0 x
    
1 1
2
Note: lim x sin 2
6= lim x · lim sin .
x→0 x x→0 x→0 x

Example 3.15. Suppose 2x − 1 ≤ f (x) ≤ x2 for 0 < x < 3. Find lim f (x).
x→1

40 MATH 122: Calculus I - Kojoga Aziedu


Solution. Since ∀x ∈ Df,g with f (x) ≤ g (x) ⇒ lim f (x) ≤ lim g (x) and 1 ∈ (0, 3), we have
x→a x→a

lim (2x − 1) ≤ lim f (x) ≤ lim x2



x→1 x→1 x→1
⇒ 1 ≤ lim f (x) ≤ lim 1.
x→1 x→1

Hence, by the Squeeze Theorem, we have that lim f (x) = 1.


x→1

Example 3.16. Suppose that |f (x)| ≤ g (x) for all x. Find lim f (x) if lim g (x) = 0.
x→a x→a

Solution. Suppose |f (x)| ≤ g (x) for all x. Then 0 ≤ f (x) ≤ g (x) since by definition,
(
f (x) , f (x) ≥ 0
|f (x)| =
−f (x) , f (x) < 0.
Therefore, since limit preserves orders, we have
lim (0) ≤ lim f (x) ≤ lim g (x) ⇒ 0 ≤ lim f (x) ≤ 0.
x→a x→a x→a x→a

Hence
lim f (x) = 0
x→a
by the squeeze theorem.

Example 3.17. Show that if lim |f (x)| = 0 , then lim f (x) = 0.


x→a x→a

Solution. Since
− |f (x)| < f (x) < |f (x)| .
Thus
lim (− |f (x)|) < lim f (x) < lim |f (x)|
x→a x→a x→a
⇒ 0 < lim f (x) < 0.
x→a

Hence, lim f (x) = 0, by the Squeeze Theorem, as required.


x→a

Example 3.18. Suppose lim − f (x) exists and that


x→−1

x2 + x − 2 f (x) x2 + 2x − 1
≤ 2 ≤ .
x+3 x x+3
Find lim − f (x).
x→−1

Solution. Notice that


x2 + x − 2 (−1)2 − 1 − 2
lim − = = −1
x→−1 x+3 −1 + 3
and
x2 + 2x − 1 (−1)2 + 2 (−1) − 1
lim − = = −1.
x→−1 x+3 −1 + 3
It thus follows from the Squeeze Principle that
f (x)
lim − = −1,
x→−1 x2
that is
lim f (x) lim f (x)
x→−1− x→−1−
= = −1 ⇒ lim − f (x) = −1.
lim − x2 (−1)2 x→−1
x→−1

41 MATH 122: Calculus I - Kojoga Aziedu


3.1.4.2 Indeterminate Limits.

The limit of a function is said to be indeterminate when its evaluation results in one of the following:
0 ∞
, 0.∞, , ∞ − ∞, 00 , ∞0 , 1∞ .
0 ∞
0
At this stage, we shall consider only the case of , for which we shall adopt two main techniques for
0
evaluating these kinds of limits: use of the factor theorem and rationalisation.
f (x)
Theorem 3.19. Suppose lim g (x) = 0. If lim f (x) 6= 0, then lim does not exist.
x→a x→a x→a g (x)

f (x)
Equivalently, if lim exists, and if lim g (x) = 0, then lim f (x) = 0.
x→a g (x) x→a x→a

Proof. We prove this by contradiction.


f (x)
Let H (x) = and suppose that lim H (x) = L and that lim g (x) = 0 but lim f (x) 6= 0. Then
g (x) x→a x→a x→a

f (x)
H (x) = ⇒ f (x) = H (x) · g (x) .
g (x)

Thus
lim f (x) = lim [H (x) · g (x)] lim H (x) · lim g (x) = L · 0 = 0,
x→a x→a x→a x→a

contradicting the hypothesis that lim f (x) 6= 0.


x→a

We shall evaluate indeterminate limits using the following methods.


Caution: Before proceeding to use these methods to evaluate limits, always verify that the limit
obtained by directly substituting the point into the function yields an indeterminate result.

3.1.4.3 Using the Factor Theorem.

Suppose
f (x) 0
lim =
x→a g (x) 0
and that f (a) = g (a) = 0. Then by the factor theorem, there exist some functions F (x) and G (x)

f (x) = (x − a) F (x)

and
g (x) = (x − a) G (x)
where deg (F (x)) = deg (f (x)) − 1 and deg (G (x)) = deg (g (x)) − 1. Therefore,

f (x) (x − a) F (x)
lim = lim
x→a g (x) x→a (x − a) G (x)

F (x)
= lim .
x→a G (x)

Example 3.20. Evaluate the following limits:

x2 − 3x + 2 (x − 1) (x2 + 2x + 4)
(a) lim (b) lim
x→1 1−x x→1 x2 + 5x − 6

42 MATH 122: Calculus I - Kojoga Aziedu


1
− a1 xn − 1
(c) lim x (d) lim , n odd positive integer.
x→a x − a x→1 x − 1

Solution.

(a) Write
x2 − 3x + 2 x2 − 3x + 2 (x − 1) (x − 2)
=− =− .
1−x x−1 x−1
Hence
x2 − 3x + 2 (x − 1) (x − 2)
lim = −lim
x→1 1−x x→1 x−1
= −lim (x − 2) = − (1 − 2)
x→1
= 1.

(b)

(x − 1) (x2 + 2x + 4) (x − 1) (x2 + 2x + 4)
lim = lim
x→1 x2 + 5x − 6 x→1 (x − 1) (x + 6)
2
x + 2x + 4 1+2+6 9
= lim = = .
x→1 x+6 1+6 7

(c)

a−x − (x − a)
1
− a1
lim x
= lim ax = lim ax
x→a x − a x→a x − a x→a x−a
−1 1
= lim = − 2.
x→a ax a

(d) Since n is an odd positive integer, we have

xn − 1 (x − 1) (xn−1 + xn−2 + · · · + x + 1)
lim = lim
x→1 x − 1 x→1 x−1
n−1 n−2

= lim x +x + ··· + x + 1
x→1
= 1| + 1 + 1 +{z· · · + 1 + 1} +1
n−1 times
= (n − 1) (1) + 1 = n.

3.1.4.4 Using the Technique of Rationalisation.

Suppose the √
function f (x) is a binomial and characteristically contains a square root term, for example,
x − ax
f (x) = , a, x > 0. Then
x−a
 √ 
x − ax 0
lim f (x) = lim = ,
x→a x→a x−a 0

which is indeterminate. To evaluate this we can adopt the method of “rationalising” the function by
multiplying it by “1” in the form of the the quotient of “conjugate” of the binomial term containing the
radical to itself.

43 MATH 122: Calculus I - Kojoga Aziedu



For example in the above example, the binomial term containing
√ the radical is x− ax, whose conjugate
√ x + ax
is x + ax). So, we multiply f (x) by “1” as 1 = √ to obtain
x + ax
√  √ √ 
x2 − ax
 
x − ax x − ax x + ax
lim = lim × √ = lim √
x→a x−a x→a x−a x + ax x→a (x − a) (x + ax)
a (x − a) a
= lim √ = lim √
x→a (x − a) (x + ax) x→a x + ax
a a 1
= √ = = .
a+ a 2 2a 2

This technique is usually employed when the function whose limit at a point is to be evaluated is a

binomial with at least one term containing a radical, .

Example 3.21. Evaluate the following limits:


√ √
x−3 x2 + 9 − 3
(a) lim (c) lim
x→9 x − 9 x→0 x2
√ √ √
x−1 x + 2 − 2x
(b) lim (d) lim
x→1 x − 1 x→2 x2 − 2x

Solution.

x−3 3−3 0
(a) First of all, lim = = which is indeterminate. Rationalising the radical, we have
x→9 x−9 9−9 0
√ √ √ 
x−3 x−3 x+3
lim = lim ×√
x→9 x − 9 x→9 x−9 x+3
 
x−9 1
= lim √ = lim √
x→9 (x − 9) ( x + 3) x→9 x+3
1 1 1
= √ = = .
9+3 3+3 6

x−1 0
(b) Clearly, lim = .
x→1 x − 1 0
√ √ √ 
x−1 x−1 x+1 1
∴ lim = lim ×√ = lim √
x→1 x − 1 x→1 x−1 x+1 x→1 x+1
1 1
= = .
1+1 2

x2 + 9 − 3 0
(c) lim 2
= . Therefore,
x→0 x 0
√ √ √ !
x2 + 9 − 3 x2
+9−3 2
x +9+3
lim = lim ×√
x→0 x2 x→0 x 2
x2 + 9 + 3
!
x2 + 9 − 9 1
= lim √  = lim √
x→0 x2 x2 + 9 + 3 x→0 x2 + 9 + 3
1 1 1
= √ = = .
0+9+3 3+3 6

44 MATH 122: Calculus I - Kojoga Aziedu


√ √
x + 2 − 2x 4−4 0
(d) lim 2
= = .
x→2 x − 2x 4−4 0
√ √ √ √ √ √ !
x + 2 − 2x x + 2 − 2x x + 2 + 2x
lim = lim ·√ √
x→2 x2 − 2x x→2 x2 − 2x x + 2 + 2x
! !
x + 2 − 2x 2−x
= lim √ √  = lim √ √ 
x→2 (x2 − 2x) x + 2 + 2x x→2 x (x − 2) x + 2 + 2x
!
x−2 1
= − lim √ √  = − lim √ √ 
x→2 x (x − 2) x + 2 + 2x x→2 x x + 2 + 2x
1 1 1
= − √  =− =− .
2 (2 + 2) 8
p
2 2 + 2 + 2 (2)

Example 3.22. Find the value of a so that

3x2 + ax + a + 3
lim
x→−2 x2 + x − 2
exists. Hence evaluate the limit.

Solution. Let f (x) = 3x2 + ax + a + 3 and g (x) = x2 + x − 2. Then since g (−2) = (−2)2 − 2 − 2 = 0,
f (x)
(x + 2) is a factor of g (x). Also, lim exists means f (−2) = 0 as well; i.e.
x→−2 g (x)

f (−2) = 3 (−2)2 − 2a + a + 3 = 0 ⇒ a = 15.

Hence,

3x2 + 15x + 18 3 (x + 2) (x + 3)
lim = lim
x→−2 x2 + x − 2 x→−2 (x + 2) (x − 1)
 
x+3 1
= 3 lim =3 = −1.
x→−2 x − 1 −3

√ √
ax + b − 2
Example 3.23. If lim = 1, find the values of a and b.
x→2 x−2
√ √
Solution. Let f (x) = ax + b − 2 and g (x) = x − 2. Then since g (2) = 2 − 2 = 0, (x − 2) is a
factor of g (x). But for the limit to exist, f (2) = 0. That is
√ √
f (2) = 2a + b − 2 = 0 ⇒ 2a + b = 2

or
b = 2 − 2a. (3.13)
Also, since
√ √ √ √ √ √
ax + b − 2 ax + b − 2 ax + b + 2 ax + b − 2
= ×√ √ = √ √ ,
x−2 x−2 ax + b + 2 (x − 2) ax + b + 2

we have that √ √
ax + b − 2 ax + b − 2
lim = lim √ √  = 1. (3.14)
x→2 x−2 x→2 (x − 2) ax + b + 2

45 MATH 122: Calculus I - Kojoga Aziedu


Substituting (??) into (3.14), we have
ax + 2 − 2a − 2 a (x − 2)
lim √ √  = lim p √ 
x→2 (x − 2) ax + 2 − 2a + 2 x→2
(x − 2) a (x − 2) + 2 + 2
a a
= lim p √ = √ √  = 1.
x→2
a (x − 2) + 2 + 2 2+ 2

Therefore,
√  √  √
a = 2 2; b = 2 − 2 2 2 = 2 − 4 2.
√ √ 
Example 3.24. Calculate lim x2 + x − x2 − x .
x→∞

Solution. First of all, note that


h√ √ i q q √
lim x + x − x − x = (∞) + ∞ − (∞)2 − ∞ = ∞ − ∞ − ∞.
2 2 2
x→∞

However,
" # " s #
h√ √ i r
x2 − x x (x − 1)
lim x2 + x − x2 − x = lim 1 − = lim 1 −
x→∞ x→∞ x2 + x x→∞ x (x + 1)
r r
∞−1 ∞
= 1− =−
∞+1 ∞
which is indeterminate. Hence, we apply rationalisation technique to evaluate the limit. Thus
 √x2 + x + √x2 − x
" #
h√ √ i √ √
lim x2 + x − x2 − x = lim x2 + x − x2 − x × √ √
x→∞ x→∞ x2 + x + x2 − x
x2 + x − (x2 − x) 2x
= lim √ √ = lim √ √
x→∞ x2 + x + x2 − x x→∞ x2 + x + x2 − x
2x 2
= lim q q = lim q q
x→∞ x→∞
x 1 + x1 + 1 − x1 1 + x1 + 1 − x1
2 2
= √ √ = = 1.
1+0+ 1−0 1+1

3.1.5 Some Useful Limits Involving Trigonometric Functions


Consider a unit circle centred at the point O such that A and B are points on the circle AC perpendicular
to OB, as shown in the diagram below.

π
Let θ rad, 0 < θ < , be the angle between OA and OB. Let AD be a tangent to the circle at the
2
−→

point A. Then, since OA = 1, we have the following:

46 MATH 122: Calculus I - Kojoga Aziedu


−→
(a) Arc length AB = rθ = θ ∵= 1 AC −→

(c) sin θ = −→ = AC
OA
−→ −→ −−→
OC −→ AC AD −−→

(b) cos θ = −→ = OC (d) tan θ = −→ = −→ = AD .
OA OC OA

Now, as θ → 0, A → B, C → B and D → B. Therefore, as θ → 0, cos θ → 1 and sin θ → 0; and

lim cos θ = 1 (3.15)


θ→0

lim sin θ = 0 (3.16)


θ→0

and
lim tan θ = 0 (3.17)
θ→0

since
sin θ lim sin θ 0
lim tan θ = lim = θ→0 = = 0.
θ→0 θ→0 cos θ lim cos θ 1
θ→0

• Consequently, we also have the following:


1 1
lim sec θ = = = 1. (3.18)
θ→0 lim cos θ 1
θ→0
1 1
lim csc θ = = = ∞. (3.19)
θ→0 lim sin θ 0
θ→0
1 1
lim cot θ = = = ∞. (3.20)
θ→0 lim tan θ 1
θ→0

3.1.5.1 The Fundamental Trigonometric Limit.

Notice from the diagram that

Area of 4AOB < Area of sector AOB < Area of 4AOD,

where
−−→ −→
• Area of 4AOB = 21 OB OA = 21 sin θ;

−→ 2
• Area of sector AOB = OA θ = 21 θ; and
1

2

1 −→ −
−→
• Area of 4AOD = 2 OA AD = 21 tan θ.

Hence,
sin θ
sin θ < θ < tan θ =
cos θ
π
for 0 < θ < . That is,
2
θ 1
1< <
sin θ cos θ
or
sin θ
cos θ < < 1.
θ
47 MATH 122: Calculus I - Kojoga Aziedu
Therefore,

sin θ
lim cos θ < lim < lim 1
θ+ →0 θ+ →0 θ θ+ →0
 
sin θ
⇒ 1 < lim < 1, ∵ lim cos θ = 1.
θ+ →0 θ θ+ →0

Hence, from the Sandwich Theorem, we have that


 
sin θ
lim = 1.
θ+ →0 θ
sin θ sin (−θ) sin θ
Note that is an even function. That is = . Therefore,
θ −θ θ
   
sin θ sin θ
lim = lim = 1.
θ− →0 θ θ+ →0 θ
Hence,  
sin θ
lim = 1. (3.21)
θ→0 θ
θ 1
Now, since = sin θ , we have that
sin θ θ
  !
θ 1 1 1
lim = lim sin θ = sin θ
 = = 1.
θ→0 sin θ θ→0
θ
lim θ 1
θ→0
   
sin θ θ
lim = lim = 1. (3.22)
θ→0 θ θ→0 sin θ
This limit is sometimes referred to as the fundamental trigonometric limit.

Corollary 3.25. Let lim f (x) = 0. Then


x→a

sin f (x)
lim = 1. (3.23)
x→a f (x)

Proof. Let z = f (x). Then as x → a, z = f (x) → 0.


sin f (x) sin z
∴ lim = lim = 1.
x→a f (x) x→0 z

sin 4x
Example 3.26. Find lim .
sin 3x
x→0

4 sin 4x
sin 4x
Solution. Write = 4x . Therefore,
sin 3x 3 sin 3x
3x
4 sin 4x sin 4x
 
sin 4x 4lim
 4x  x→0 4x
lim = lim  = .
x→0 sin 3x x→0 3 sin 3x  sin 3x
3lim
3x x→0 3x
Let y = 4x and z = 3x. Then as x → 0, y → 0 and z → 0. Hence,
sin y
lim  
sin 4x 4 y→0 y 4 1
lim = =
x→0 sin 3x 3 sin z 3 1
lim
z→0 z
4
= .
3

48 MATH 122: Calculus I - Kojoga Aziedu


1 − cos x
3.1.5.2 The Limit lim .
x→0 x

From the trigonometric identity sin2 x = 21 (1 − cos 2x), we write

2 x 2 x
    
1 − cos x = 1 − 1 − 2 sin = 2 sin ,
2 2
so that x x
1 − cos x 2 sin2 sin2
= 2 = 2 .
x
x x 2
Hence,
x
1 − cos x sin2
lim = lim x
2
x→0 x x→0
2
x
x sin
= lim sin · lim x
2
x→0 2 x→0
2
= 0·1=0

since x
2
→ 0 as x → 0. Therefore,
1 − cos x
lim = 0. (3.24)
x→0 x

Alternative Approach.
1 − cos x 0
Since lim = is indeterminate, we have, from
x→0 x 0

1 − cos x 1 − cos x 1 + cos x 1 − cos2 x


= · =
x x 1 + cos x x (1 + cos x)
2
sin x sin x sin x
= = · ,
x (1 + cos x) x 1 + cos x

that
1 − cos x sin x sin x
lim = lim · lim
x→0 x x→0 x x→0 1 + cos x
lim sin x 0
x→0
= 1· = = 0.
lim (1 + cos x) 1+1
x→0

Example 3.27. Evaluate the following limits.

sin2 3x sin (x − 1)
(i) lim (v) lim
x→0 x2 x→1 x2 + x − 2

sin x sin x − x
(ii) lim (vi) lim
x→0 x + tan x x→0 tan x
sin x2 3x2 + x
(iii) lim (vii) lim
x→0 x x→0 sin 2x

1 − tan x sin (x + 3)
(iv) limπ (viii) lim
x→ 4 sin x − cos x x→−3 x3 + 3x2

49 MATH 122: Calculus I - Kojoga Aziedu


Solution.

(i)

sin2 3x
 
3 sin 3x 3 sin 3x 3 sin 3x 3 sin 3x
lim 2
= lim · = lim · lim
x→0 x x→0 3x 3x x→0 3x x→0 3x
   
sin 3x sin 3x
= 3 lim · 3 lim = 3 · 1 · 3 · 1 = 9.
x→0 3x x→0 3x

Alternatively, write
2 2
sin2 3x
 
3 sin 3x 3 sin 3x
lim = lim = lim
x→0 x2 x→0 3x x→0 3x
= (3)2 = 9.

(ii) Write
   
sin x 1 1
lim = lim   = lim  x
   
x→0 x + tan x x→0 x + tan x x→0 tan x 
+
sin x sin x sin x
1 1
= =
x 1

x sin x 1
lim + · lim + lim
x→0 sin x cos x sin x x→0 sin x x→0 cos x

1 1 1
= = = .
1 1+1 2
1+
lim cos x
x→0

(iii) Write

sin x2 sin x2 x x sin x2


 
lim = lim · = lim
x→0 x x→0 x x x→0 x2
sin x2
= lim x · lim =0·1=0
x→0 x→0 x2

since as x2 → 0 as x → 0.


Alternatively, let u = x2 ⇒ u = x. Then as x → 0, u → 0. Thus

sin x2 sin u u sin u √ sin u
lim = lim √ = lim = lim u · lim = 0 · 1 = 0.
x→0 x u→0 u u→0 u u→0 u→0 u

1 − tan x 1−1 0
(iv) Since limπ = 1 = , we rationalise. Thus
x→ 4 sin x − cos x √1− 2
√ 0
2

sin x sin x
1 − tan x 1 − cos x
1 − cos x cos x
limπ = limπ = limπ ·
x→ 4 sin x − cos x x→ 4 sin x − cos x x→ 4 sin x − cos x cos x

cos x − sin x sin x − cos x


= limπ = − limπ
x→ 4 (sin x − cos x) cos x x→ 4 (sin x − cos x) cos x

1 1 √
= − limπ = − 1 = − 2.
x→ 4 cos x √
2

50 MATH 122: Calculus I - Kojoga Aziedu


(v)
sin (x − 1) sin (x − 1) sin (x − 1) 1
lim = lim = lim · lim
x→1 x2 + x − 2 x→1 (x − 1) (x + 2) x→1 (x − 1) x→1 x + 2
1 1
= = .
1+1 2

(vi)
sin x − x sin x − x cos x (sin x − x)
lim = lim sin x = lim
x→0 tan x x→0
cos x
x→0 sin x
 x   x 
= lim cos x 1 − = lim cos x · 1 − lim
x→0 sin x x→0 x→0 sin x
= 1 · (1 − 1) = 0.

(vii)
3x2 + x 3x2 3x2
 
x x
lim = lim + = lim + lim
x→0 sin 2x x→0 sin 2x sin 2x x→0 sin 2x x→0 sin 2x
2x 3 1 2x 1 1
= lim · lim x + lim =1·0+ ·1= .
x→0 sin 2x 2 x→0 2 x→0 sin 2x 2 2
Alternatively,
3x2 + x  x  1 2x
lim = lim (3x + 1) = lim · lim (3x + 1)
x→0 sin 2x x→0 sin 2x 2 x→0 sin 2x x→0
1 1
= · 1 (3 · 0 + 1) = .
2 2
(viii)
sin (x + 3) sin (x + 3) 1 sin (x + 3)
lim 3 2
= lim 2 = lim 2 · lim .
x→−3 x + 3x x→−3 x (x + 3) x→−3 x x→−3 x+3
sin (x + 3)
For lim , let z = x + 3. Then as x → −3, z → 0.
x→−3 x+3
sin (x + 3) 1 sin z 1 1
∴ lim 3 2
= lim 2 · lim = 2 ·1= .
x→−3 x + 3x x→−3 x z→0 z (−3) 9

Example 3.28. Evaluate the following limits.

tan 2x
(a) limπ
z→ 2 z − π2
(b) lim
z→

(c)
(d)

Solution.

(a) Approach const html


(b)
(c)
(d)

51 MATH 122: Calculus I - Kojoga Aziedu


3.1.6 Limits Involving Inverse Trigonometric Functions.
Here, we shall heavily use the fact that inverse trigonometric functions are continuous (see sub-
subsection 3.3.1.2) on their respective domains. The following theorem tells it all.

Theorem 3.29. Suppose the function f is continuous at x = b and that lim g (x) = b. Then
x→a

lim f (g (x)) = f (b) .


x→a

That is,  
lim f (g (x)) = f lim g (x) .
x→a x→a

Example 3.30. Evaluate the following limits.


  √  √
1− x

x2 − 1
(a) lim sin−1
(d) lim sin−1
x→1 1−x x→∞ x
 
x
2
(e) √ −1
  
x −4 lim tan
(b) lim tan−1 x→∞ x2 + 1
x→2 3x2 − 6x
" √ #
x 3
(c) lim tan−1 ex (f) lim cos−1 2 +
x→±∞
x→−∞ x + 10 2

Solution. Since the inverse trigonometric functions are continuous, we can interchange the functions
and limits.

(a)
  √    √   
−1 1− x −1 1− x −1 0
lim sin = sin lim = sin
x→1 1−x x→1 1−x 0

which is indeterminate. So we rationalise to get


  √    √ √    
−1 1− x −1 1− x 1+ x −1 1−x
lim sin = sin lim × √ = sin lim √
x→1 1−x x→1 1−x 1+ x x→1 (1 − x) (1 + x)
      
−1 1 −1 1 −1 1 π
= sin lim √ = sin = sin = .
x→1 1+ x 1+1 2 6

(b)

x2 − 4 x2 − 4 22 − 4
         
−1 −1 −1 −1 0
lim tan = tan lim = tan = tan ,
x→2 3x2 − 6x x→2 3x2 − 6x 3 (22 ) − 6 (2) 0

a indeterminate ratio. But


  2       
−1 x −4 −1 (x + 2) (x − 2) −1 x+2
lim tan = tan lim = tan lim
x→2 3x2 − 6x x→2 3x (x − 2) x→2 3x
   
2+2 2
= tan−1 = tan−1 .
3 (2) 3

52 MATH 122: Calculus I - Kojoga Aziedu


(c) Since tan−1 is continuous on (−∞, ∞), we have
 
−1 x −1 x
lim tan e = tan lim e .
x→±∞ x→±∞

Let z = ex . Then as x → ∞, z → ∞ and as x → −∞, z → 0, i.e.


1 1
lim ex = ∞; lim ex = ∞
= = 0.
x→∞ x→−∞ e ∞
Therefore,   π
lim tan−1 ex = tan−1
lim ex = tan−1 (∞) =
x→∞ x→∞ 2
and  
−1 x −1
lim tan e = tan lim e = tan−1 (0) = 0.
x
x→−∞ x→−∞

(d)
√ 2   √ 2 
−1 x −1 −1 x −1 h∞i
lim sin = sin lim = sin−1 ,
x→∞ x x→∞ x ∞
an indeterminacy. However, factorising x2 out in the numerator of the argument of the limit, we
have
√ 2 √ 2
  q 
1
x −1
 
x −1
 x 1 − x2
lim sin−1 = sin−1 lim = sin−1  lim  
x→∞ x x→∞ x x→∞ x
" r !# "s  #
1 1
= sin−1 lim 1− 2 = sin−1 1 − lim
x→∞ x x→∞ x2
√ π
= sin−1 1 − 0 = sin−1 (1) = .

2

(e)
     h∞i
−1 x −1 x
lim tan √ = tan lim √ = tan−1 .
x→∞ x2 + 1 x→∞ x2 + 1 ∞

Therefore,
  
    
x x x
lim tan−1 √ = tan−1 lim √ = tan−1  lim  q 
x→∞ 2
x +1 x→∞ x2+1 x→∞ 1
x 1 + x2
    
1 1
= tan−1  lim  q  = tan−1  q 
x→∞ 1
1
1 + x2 1 + lim x2
x→∞
 
1 π
= tan−1 √ = tan−1 (1) = .
1+0 4

(f) Notice that


" √ # " √ !# " √ #
x 3 x 3 ∞ 3
lim cos−1 2
+ = cos−1 lim 2
+ = cos−1 2 +
x→−∞ x + 10 2 x→−∞ x + 10 2 (∞) + 10 2
" √ #
−1 ∞ 3 ∞
= cos + =
∞ + 10 2 ∞

53 MATH 122: Calculus I - Kojoga Aziedu


which is indeterminate too. Hence,
" √ # " √ !#
x 3 x 3
lim cos−1 2 + = cos−1 lim 2
+
x→−∞ x + 10 2 x→−∞ x + 10 2
" √ #
x 3
= cos−1 lim 2 + lim
x→−∞ x + 10 x→−∞ 2
"
1
√ # " √ #
3 0 3
= cos−1 lim x
10 + = cos−1 +
x→−∞ 1 + 2
x
2 1+0 2
"√ #
3 π
= cos−1 = .
2 6

Note the following carefully about the combination of the ∞s and 0s with finite constants and their
indeterminacy or otherwise. Let a be a finite constant. Then

No. Combination Remark


a
1. =0 Finite
±∞
±∞
2. = ±∞ Infinite but not indeterminate
a ∞
3. Indeterminate

4. ∞±a=∞ Infinite but not indeterminate
5. a ± ∞ = ±∞ Infinite but not indeterminate
6. ∞−∞ Indeterminate
7. ∞+∞ Infinite but not indeterminate
8. ∞·∞=∞ Infinite but not indeterminate
±a
7. = ±∞ Infinite but not indeterminate
0
0
8. Indeterminate
0

3.1.6.1 Some Special Limits.

The following limits are of interest to us in this course.


sin x◦ π
lim+ = (3.25)

x →0 x◦ 180◦
x
e −1
lim = 1 (3.26)
x→0 x
ln (1 + x)
lim = 1 (3.27)
x→0 x
ln (1 + ax)
lim = 1 (3.28)
x→0 ax
lim+ ln x = −∞ (3.29)
x→0
lim ln x = ∞ (3.30)
x→∞
lim loga x = −∞ (3.31)
x→0+
lim loga x = ∞ (3.32)
x→∞
π
lim tan−1 (x) = ± . (3.33)
x→±∞ 2

Proof. m

54 MATH 122: Calculus I - Kojoga Aziedu


Example 3.31. Find the value which should be assigned to the function
ln (1 + ax) − ln (1 − bx)
f (x) =
x
x = 0 so that it is continuous at x = 0.

Solution. We know for the function f to be continuous we must have


f (0) = lim f (x)
x→0
ln (1 + ax) − ln (1 − bx)
= lim
x→0 x
a ln (1 + ax) b ln (1 − bx)
= lim + lim
x→0 ax x→0 −bx
= a · 1 + b · 1 = a + b.
Therefore, the value that must be assigned for f (0) = a + b.

3.1.7 Infinite Limits: Vertical Asymptotes


1
Consider the function f (x) = .
x

Figure 3.6: Infinite limits

55 MATH 122: Calculus I - Kojoga Aziedu


As x → 0+ , f (x) becomes
  very large and positive, and as x → 0 , f (x) becomes very large and

1
negative. Thus lim± do not exist.
x→0 x
 
1
Similarly, lim does not exist.
x→0 x2
Notation: We use the notations
 
1
lim+ = +∞
x→0 x
 
1
lim− = −∞.
x→0 x
Definition 3.32. Vertical Asymptotes.
Suppose f (x) is a function that is defined at both sides of the point a except possibly at a. Then
 
1
lim = ∞
x→a x

means the values of f (x) become larger and positive as x → a.


Similarly,
 
1
lim = −∞
x→a x

means that the values of f (x) become larger and negative as x → a.


The line x = a is called the vertical asymptote of the curve y = f (x) if at least one of the following is
true:

(i) lim f (x) = ∞ (ii) lim f (x) = ∞ (iii) lim f (x) = ∞.


x→a x→a+ x→a−

• In simple terms terms vertical asymptotes of the funtionf (x) are those values of x for which f is
undefined.

Example 3.33. Evaluate the following limits:


6 x−1
(a) lim− (b) lim +
x→5 x−5 x→−2 x2 (x + 2)

Solution.

6
(a) Let h = x − 5 so that . Now, as x → 5− , h → 0− . Therefore,
x−5
6 6 1
lim− = lim− = 6 lim− = −∞.
x→5 x − 5 h→0 h h→0 h

(b) Notice that, for some constants A, B and C,


x−1 A B C
≡ + + .
x2 (x + 2) x x2 x + 2
Thus,
x − 1 = Ax (x + 2) + B (x + 2) + Cx2 ⇒ B = − 21 , C = − 43 , A = 34 .

56 MATH 122: Calculus I - Kojoga Aziedu


Therefore,
x−1 3 1 3
≡ − 2− .
x2 (x + 2) 4x 2x 4 (x + 2)
Hence,
x−1 3 1 3
lim + = lim + − lim + 2 − lim +
x→−2 x2 (x + 2) x→−2 4x x→−2 2x x→−2 4 (x + 2)
3 1 3 1
= − − − lim + .
8 8 4 x→−2 x + 2

Let h = x + 2. Then as x → −2+ ⇒ h → 0+ .


x−1 1 3 1
∴ lim + = − − lim
x→−2 x2 (x + 2) 2 4 h→0+ h
1 3
= − − (∞) = −∞.
2 4

3.1.7.1 Sign Analysis of Limits.

.
.
.
See Limits analytically and DUMMIES in Calculus I Folder on Desktop
.
.
.
.

Example 3.34. Find the vertical asymptotes of the following functions:


x 3 (x + 1)
(a) f (x) = (b) f (x) =
x2 −x−2 x2− 3x − 4

Solution.

x x
(a) f (x) = = . f (x) is not defined at x = −1, 2. Therefore, the lines
x2 − x − 2 (x + 1) (x − 2)
x = −1 and x = 2 are the vertical asymptotes. The graph below shows these asymptotes.

57 MATH 122: Calculus I - Kojoga Aziedu


(b) Notice that
3 (x + 1) 3 (x + 1) 3
f (x) = = = .
x2 − 3x − 4 (x + 1) (x − 4) x−4
Thus the f (x) is undefined when
x − 4 = 0 ⇒ x = 4.
Thus the vertical asymptote of f is x = 4 as shown the graph below.

58 MATH 122: Calculus I - Kojoga Aziedu


f (x)
3.1.8 Limits at Infinity: General Rule for Evaluating lim .
x→∞ g (x)

Consider the limit


f (x)
lim
x→∞ g (x)
where f (x) and g (x) are polynomials in x. That is

f (x) = a0 xn + a1 xn−1 + a2 xn−2 + · · · + an−1 x + an

and
g (x) = b0 xm + b1 xm−1 + b2 xm−2 + · · · + bm−1 x + am .
1
Let x = = z −1 . Then as x → ∞, z → 0, so that
z
f (x) a0 xn + a1 xn−1 + a2 xn−2 + · · · + an−1 x + an
=
g (x) b0 xm + b1 xm−1 + b2 xm−2 + · · · + bm−1 x + am
a0 z −n + a1 z −n+1 + a2 z −n+2 + · · · + an−1 z −1 + an
=
b0 z −m + b1 z −m+1 + b2 z −m+2 + · · · + bm−1 z −1 + am
(a0 + a1 z + a2 z 2 + · · · + an−1 z n−1 + an z n ) z −n
=
(b0 + b1 z + b2 z 2 + · · · + bm−1 z m−1 + am z n ) z −m
a0 + a1 z + a2 z 2 + · · · + an−1 z n−1 + an z n m−n
= z
b0 + b1 z + b2 z 2 + · · · + bm−1 z m−1 + am z n
Therefore,

f (x) a0 + a1 z + a2 z 2 + · · · + an−1 z n−1 + an z n m−n


lim = lim z
x→∞ g (x) z→0 b0 + b1 z + b2 z 2 + · · · + bm−1 z m−1 + am z n

a0 + a1 z + a2 z 2 + · · · + an−1 z n−1 + an z n
= lim · lim z m−n
z→0 b0 + b1 z + b2 z 2 + · · · + bm−1 z m−1 + am z n z→0
a0
= lim z m−n .
b0 z→0

Deductions.

f (x) a0
• n < m , then z m−n → 0 as z → 0 ⇒ lim = lim z m−n = 0.
x→∞ g (x) b0 z→0

f (x) a0
• n > m , then z m−n → ∞ as z → 0 ⇒ lim = lim z m−n = ∞.
x→∞ g (x) b0 z→0
f (x) a0 a0
• n = m , then z m−n → 1 as z → 0 ⇒ lim = lim z m−n = .
x→∞ g (x) b0 z→0 b0
Hence,

if

0,
 n<m
 a0 , if

f (x) n=m
lim = b0
(3.34)
x→∞ g (x) 

 ∞, if n > m, a0 b0 > 0
−∞, if

n > m, a0 b0 < 0.

59 MATH 122: Calculus I - Kojoga Aziedu


Definition 3.35. Horizontal Asymptote.
Let f (x) be a function defined on some interval (a, ∞). Then

lim f (x) = L
x→∞

means that the value of f (x) can be arbitrarily close to L by taking x sufficiently large. Similarly, for

lim f (x) = L.
x→−∞

The line y = L is called the horizontal asymptote of the curve y = f (x) if either

lim f (x) = L
x→∞

or
lim f (x) = L.
x→−∞

1 1
Now, suppose f (x) = . If we let z = , then as x → ∞, z → 0.
x x
1
lim = lim z = 0.
x→∞ x z→0
1
In general, if n ∈ Q, then z n = , then as x → ∞, z → 0, and
xn
1
lim = lim z n = 0.
x→∞ xn z→0

That is,
Theorem 3.36. For any rational number n,
 
1
lim = 0. (3.35)
x→±∞ xn
Example 3.37. Evaluate the following:
x2 + 4 2x2 + 3x + 4
(a) lim (b) lim
x→∞ x2 − 1 x→∞ 5x2 + 2x + 1
√ 
(c) lim x2 + 3x + 1 − x
x→∞

Solution.

x2 + 4 x2 − 1 + 5 5
(a) Notice that 2
= 2
=1+ 2 . (You can also use long division)
x −1 x −1 x −1
x2 + 4
 
5
lim = lim 1 + 2
x→∞ x2 − 1 x→∞ x −1
= 1 + 0 = 1.
1
Alternatively, Let z = . then z → 0 as x → ∞.
x
1 2
 ! 1
x2 + 4 + 4 z2

z
+4 z2
∴ lim 2 = lim = lim 1 ×
x→∞ x − 1 1 2 − 1 z2

z→0 −1 z→0
z z2
1 + 4z 2 1+0
= lim 2
=
z→0 1 − z 1−0
= 1.

60 MATH 122: Calculus I - Kojoga Aziedu


(b)
 
3 4
lim 2 + + 2
2x2 + 3x + 4 x→∞ x x
lim 2
=  
x→∞ 5x + 2x + 1 2 1
lim 5 + + 2
x→∞ x x
2+0+0 2
= = .
5+0+0 5

(c) Notice that

√ √  √x2 + 3x + 1 + x
x2 + 3x + 1 − x = x2 + 3x + 1 − x · √
x2 + 3x + 1 + x
x2 + 3x + 1 − x2 3x + 1
= √ =√
2
x + 3x + 1 + x 2
x + 3x + 1 + x
Therefore,
 
1
√  3x + 1 3+
lim x + 3x + 1 − x = lim √
2

= lim  r
 x 

x→∞ x→∞ x2 + 3x + 1 + x x→∞ 3 1 
1+ + 2 +1
x x
   
1 1
lim 3 + lim 3 +
x→∞ x x→∞ x
= r !=s
3 1
 
3 1
lim 1+ + 2 +1 lim 1 + + 2 + 1
x→∞ x x x→∞ x x
3 3
= = .
1+1 2

61 MATH 122: Calculus I - Kojoga Aziedu


3.2 Exercise on Limits.
1. Given that lim f (x) = 2, lim g (x) = 5 and lim h (x) = 0. Find the following limits, if they exist.
x→2 x→2 x→2

2f (x) + h (x) f (x) 2f (x) h (x)


(a) lim (b) lim (c) lim
x→2 g (x) x→2 h (x) x→2 g (x)
(
3x + 1 if x < 1
2. Evaluate the limits lim f (x) and lim |x|, where f (x) = .
x→1 x→0 5 − x2 if x ≥ 1

3. Let √
 −x
 if x < 0
f (x) = 3 − x if 0 ≤ x < 3
(x − 3)2 if x > 3.

Evaluate the following limits, if they exist.

(a) lim f (x) (c) lim f (x) (e) lim f (x)


x→0+ x→0 x→3−

(b) lim f (x) (d) lim f (x) (f) lim f (x)


x→0− x→3+ x→3

4. Consider the function 





1
x2
if x < −1
if − 1 ≤ x < 1

2
f (x) =
3
 if 1 < x ≤ 2

− 1 2

x > 2.
(x−2)

By sketching the graph of f determine the following limits.

(a) lim f (x) (d) lim f (x) (g) lim f (x) (j) lim f (x)
x→−1+ x→1+ x→2+ x→−3

(b) lim f (x) (e) lim f (x) (h) lim f (x) (k) lim f (x)
x→−1− x→1− x→2− x→5

(c) lim f (x) (f) lim f (x) (i) lim f (x) (l) lim f (x)
x→−1 x→1 x→2 x→ 32

5. Evaluate the following limits.

(a) lim (x2 − 4) x2 − 2x + 1 x−2


x→2 (d) lim (g) lim
x→1 x3 − x x→2 x2
− 3x + 2
x3 − 4x x3 + 3x2 + x 3x − 4x3 + 1
4
(b) lim (e) lim (h) lim
x→0 2x2 + 3x x→−2 x2 − x − 6 x→1 (x − 1)2
x−2 3x + 2x−1
(c) lim 2 (f) lim
x→2 x − 3x + 2 x→0 x + 4x−1

6. Evaluate the following limits numerically.



sin x 3 − 2x4 x3 − 8
(a) lim± (d) lim . (g) lim
x→0 x x→±∞ 1 + x2 + x4 x→4 x−4
1 − cos x (1 + x)2 − 1 (h) lim x2 sec x
(b) lim± (e) lim x→0
x→0 x x→0 x
x x4 − 1 2
(c) lim √ (f) lim (i) lim
x→0 x+1−1 x→1 x − 1 x→3 (x − 3)2

7. Evaluate the following limits.

62 MATH 122: Calculus I - Kojoga Aziedu



x−1
(a) lim
x→1 x − 1
xn − 1
(b) lim m , m, n ∈ N
x→1 x − 1
p √
(a + bx) (c + dx) − ac
(c) lim , a, b, c, d ∈ R+
x→0 x
8. Show that
1 − cos x 1
lim 2
= .
x→0 x 2
Use this result to show that
2 sin2 x
lim = 1.
x→0 x3 csc x

9. Let a > 0 and n ∈ Q. Show that


x n − an
lim = nan−1 .
x→a x − a

Hence, find the positive integer n so that


xn − 3n
lim = 108.
x→3 x − 3

10. By using the principle of mathematical induction, show that, ∀n ∈ N


" k # k
X X
lim fi (x) = lim fi (x)
x→a x→a
i=1 i=1

and h in
lim [f (x)]n = lim f (x) .
x→a x→a

11. Let (
3x + a, x ≤ 2
f (x) =
x − 2, x > 2.
Find the real constant a for which the limit of x exists as x approaches 2 and state the value of
the limit if possible.

12. If 2x ≤ g (x) ≤ x2 − x + 2 for all x, evaluate lim g (x).


x→0

13. Find lim f (x), the limit of the given function, in the following.
x→0
√ √
(a) 25 − 2x2 ≤ f (x) < 5 − x2 . 1 x2 1 − cos x 1
(c) − < 2
<
x2 x sin x 2 24 x 2
(b) 1 − < <1 (d) m
6 2 − cos x

14. By using the Squeeze theorem, find lim f (x) of the following functions.
x→0

|x| (c) m
(a) f (x) =
1 + x2

x+1−1
(b) f (x) = (d) Add more.
5x2 + 2

15. Evaluate the following limits.


2 − cos x
 
sin x 1 1
(i) lim lim − (ii) lim
x→∞ x x→0 x |x| x→∞ x+3

63 MATH 122: Calculus I - Kojoga Aziedu


x2 2 + sin2 x
  
1
(iii) lim (x − 1) sin (viii) lim
x→1 x−1 x→∞ x + 100
5x2 − sin 3x
(iv) lim 1 + (x − 5)2 sin (csc πx)
 
(ix) lim
x→5 x→−∞ x2 + 10
π i x2 cos x + sin3 x
h  
(v) lim (x + 1) + (x − 2) sin sec 2
(x) lim
x→2 x x→−∞ (x2 + 1) (x − 3)

cos2 2x
 
2 1
(vi) lim (xi) lim √ −
x→∞ 3 − 2x h→0 h 4+h h
  √
2 8−x−2
(vii) 3
lim− x cos (xii) lim √
x→0 x x→4 5−x−1

16. Find the constant b so that lim f (x) exists given that
x→1
(
x2 − 2, x < 1
f (x) =
bx − 4, x ≥ 1.

17. Compute the following


√   √
x+1−1 1 1 1 x− x
(a) lim (d) lim √ − (g) lim √
x→0 x x→0 x x+9 3 x→0 x
 √  1
− 14 x2 − 4
(b) lim x + x − x
2
(e) lim x
(h) lim
x→∞ x→4 x − 4 x→2 |x − 2|
2
1 √ √  + x+4 |x|
(c) lim 1+x− 1−x (f) lim x−3 5
(i) lim
x→0 x x→−2 x2 + 5x + 6 x→0 x2 + x + 10

18. Compute the following limits.

(a) lim cos e−x


 sin x tan x − sin x
x→∞ (e) lim (i) lim+
x→0 |x| x→0 x2
sin (x − 2) x2 − 9
(b)
1
lim 2
x→2 x − x − 2 (f) lim √ (j) lim− [4x + 3x ] x
h→−3 4 − 13 − x x→0
sec x − 1 sin (x − 3) 1
(c) lim (g) lim 2 (k) lim [4x + 3x ] x
x→π sin x x→3 x − 5x + 6 x→∞

x2 − π 2 cos x sin y cos x sin x − tan x


(d) lim (h) lim lim (l) lim .
x→π sin x y→0 x→0 x−y x→0 x2 sin x
f (x)
19. Let f be an invertible function and that f (0) = 0, f is continuous at x = 0 and lim exists.
x→0 x
f (x)
Given that lim = L, show that
x→0 x
x
lim = L.
x→0 f −1 (x)

(
f (x)
, if x 6= 0
[Hint: Define h (x) = x
and apply Theorem 3.29 to the composition h ◦ f −1 .]
L, if x = 0
Hence, or otherwise, evaluate the following limits.
x sin−1 5x
(a) lim (c) lim
x→0 sin−1 x
x→0 x
−1
tan−1 x sin (x − 1)
(b) lim (d) lim
x→0 x x→0 x2 − 1

64 MATH 122: Calculus I - Kojoga Aziedu


20. Find the vertical and horizontal asymptotes (if any) of the following functions. Justify your
answer.
x+2 (d) f (x) = tan x
(a) f (x) =
x−2
(e) f (x) = cot x
x2 − 2x − 8
(b) f (x) = (f) f (x) = sec x.
x+2
x+5
(c) f (x) = 3
x + x2 − 2x

21. Vertical asymptotes

22. m

23. m

65 MATH 122: Calculus I - Kojoga Aziedu


3.3 Continuity of a Function

3.3.1 The Intuitive Notion of the Continuity of a Function


Definition 3.38. Continuity of a Function
Let f be a function. Then f is said to be continuous at the point x = a if

• f (a) is defined, that is, a ∈ Df ,

• lim f (x) exists, and


x→a

• lim f (x) = f (a).


x→a

Symbolically, we write  
lim f (x) = f lim x = f (a)
x→a x→a

to denote the notion of the continuity of f at x = a.


In simple terms, a function is said to be continuous at a point if there is no “breakage” in the function
at that point.

Definition 3.39. Left- and Right-continuous Functions.


The function f is said to be left-continuous at the point x = a if

lim f (x) = f (a)


x→a−

and right-continuous when


lim f (x) = f (a) .
x→a+

Note that a piecewise (spliced) function can only be continuous at a point x = a if it is both left- and
right-continuous at a; i.e. if
lim− f (x) = lim+ f (x) = f (a) .
x→a x→a

Figure 3.7: Continuity of a function at a point

Definition 3.40. Discontinuity of a Function


A function f is said to be discontinuous (or not continuous; or have a discontinuity) at the point x = a
if it is not continuous at that point; that is one of those three conditions was not satisfied.

66 MATH 122: Calculus I - Kojoga Aziedu


Figure 3.8: Discontinuity of a function at a point

Example 3.41. Consider the function


x2 − 1
f (x) = .
x+1
(−1)2 − 1 0
It is clearly undefined at x = −1 since f (−1) = = . However,
−1 + 1 0
x2 − 1 (x + 1) (x − 1)
lim f (x) = lim = lim .
x→−1 x→−1 x + 1 x→−1 x+1
= lim (x − 1) = −1
x→−1

x2 − 1
exists, but lim f (x) 6= f (−1). Therefore, f (x) = is not continuous at x = −1.
x→−1 x+1

Example 3.42. Explain why the function f (x) defined by


 2
 x − 2x − 8
if x 6= 4
f (x) = x−4
3 if x = 4.

is discontinuous at x = 4.

Solution. By definition, f (4) = 3. Now

x2 − 2x − 8 (x + 2) (x − 4)
lim f (x) = lim = lim
x→4 x→4 x−4 x→4 x−4
= lim (x + 2) = 4 + 2 = 6
x→4
6= 3 = f (4) .

Thus the value of the function at x = 4 is not equal to the limit as x → 4. It follows that f is
discontinuous at x = 4.

67 MATH 122: Calculus I - Kojoga Aziedu


Example 3.43. Let (
1
if x 6= 0

x4 sin x
f (x) =
0 if x = 0.
Show that f is continuous at x = 0.

Solution. First note that it is given that f (0) = 0. It is now left for us to establish that x → 0 ⇒
f (x) → 0. Now,    
1 4 4 1
−1 ≤ sin ≤ 1 ⇒ −x ≤ x sin ≤ x4
x x
for all real x. And since
lim −x4 = lim x4 = 0,
 
x→0 x→0

it follows from the Squeeze Principle that


 
4 1
lim f (x) = lim x sin = 0 = f (0) .
x→0 x→0 x
Hence, f is continuous at x = 0.

3.3.1.1 Test of Continuity of a Function at a Point.

Testing for the continuity or otherwise of a function at a point is to verify whether the function satisfies
or violates any or all of those three conditions for continuity. So to prove that a function is not
continuous at a point x = a, it is sufficient to show that one of the three conditions for continuity,
namely
lim− f (x) = lim+ f (x) = f (a)
x→a x→a
is not met.
Example 3.44. Test the continuity or otherwise of the following functions at the point x = 1.
 2
(a) f (x) = 7x3 + 3x2 − 2  x + 2x − 3
if x 6= 1
(c) h (x) = x−1
4 if x = 1

x2 + 2x − 3
(b) g (x) = (d)
x−1

Solution.

(a) f (x) = 7x3 + 3x2 − 2.


 f (1) = 7 (1)3 + 3 (1)2 − 2 = 7 + 3 − 2 = 8
 lim f (x) = lim (7x3 + 3x2 − 2) = 7lim x3 + 3lim x2 − lim 2 = 7 + 3 − 2 = 8.
x→1 x→1 x→1 x→1 x→1

 Since lim f (1) = 8 = f (1), f (x) = 7x + 3x − 2 is continuous at x = 1.


3 2
x→1

x2 + 2x − 3
(b) For g (x) = , g (1) is not defined. Therefore, g (x) is discontinuous at x = 1.
x−1
 2
 x + 2x − 1
if x =
6 1
(c) For h (x) = x−1 ,
4 if x = 1

x2 + 2x − 3 (x + 3) (x − 1)
= = x + 3, ∵ x 6= 1.
x−1 x−1
Therefore,

68 MATH 122: Calculus I - Kojoga Aziedu


 h (1) = 1 + 3 = 4
 lim h (x) = lim (x + 3) = 1 + 3 = 4.
x→1 x→1

 And lim h (1) = 4 = h (1). Hence, h is continuous at x = 1.


x→1

Example 3.45. Find the value of p such that the function


(
x − 3p if x < 3
f (x) =
1 − 2px if x ≥ 3

is continuous at x = 3.

Solution. For f to be continuous at x = 3, we must have that

lim f (x) = f (3) .


x→3

However, lim f (x) exists provided


x→3
lim f (x) = lim+ f (x) .
x→3− x→3

Now
lim f (x) = lim− (x − 3p) = 3 − 3p
x→3− x→3

and
lim f (x) = lim+ (1 − 2px) = 1 − 6p.
x→3+ x→3

Thus
2
3 − 3p = 1 − 6p ⇒ 3p = −2 ⇒ p = − .
3
Hence, with this value of p,
  
2
lim− f (x) = lim− x − 3 − = lim− (x + 2) = 3 + 2 = 5,
x→3 x→3 3 x→3

     
2 4 4
lim+ f (x) = lim+ 1 − 2 − x = lim− 1 + x = 1 + (3) = 5,
x→3 x→3 3 x→3 3 3
and  
4 4
f (3) = 1+ x =1+ (3) = 5
3 3
confirming the continuity of f at x = 3.

Example 3.46. Find the number c so that the function


q
 (c − x)2 − 1 if x < 1

f (x) = cx2 + x − 2

 2 if x ≥ 1
x −x+3
is continuous at the point x = 1.

Solution. Again, for f to be continuous, the left-hand and right-hand limits must be equal and their
value equal to the value of f at x = 1. Let us compute them as follows.
q q
lim− f (x) = lim− (c − x) − 1 = (c − 1)2 − 1
2
x→1 x→1

69 MATH 122: Calculus I - Kojoga Aziedu


and
cx2 + x − 2 c−1
lim+ f (x) = lim+ 2 = .
x→1 x→1 x − x + 3 3
This limit exist if
c−1 (c − 1)2
q
(c − 1)2 − 1 = ⇒ (c − 1)2 − 1 =
3 9
or
9
9 (c − 1)2 − 9 = (c − 1)2 ⇒ (c − 1)2 =
8
and so
√r
9 3 3 2
c−1 = ± =± √ =±
8 2 2 4

3 2
∴ c = 1± .
4

Now, when c = 1 + 3 2
4
,
√ u √ !2
v !2 v
u
3 2 u 3 2
q u
lim− f (x) = (c − 1)2 − 1 = t 1 + −1 −1=t −1
x→1 4 4
r r √
18 2 2
= −1= =
16 16 4
and
√ √ √
c−1 1 + 342 − 1 3 2
2
lim+ f (x) = = = 4 = .
x→1 3 3 3 4

Thus when c = 1 + 3 2
4
, lim− f (x) = lim+ f (x).
x→1 x→1

We do same for c = 1 − 3 2
4
as well.
√ √ !2
v !2 v
u u
3 2 3 2
q u u
lim− f (x) = (c − 1)2 − 1 = t 1 − −1 −1=t − −1
x→1 4 4
r r √
18 2 2
= −1= =
16 16 4
and
√ √ √
c−1 1 − 342 − 1 −342 2
lim+ f (x) = = = =− .
x→1 3 3 3 4
√ √ √ √
Thus for c = 1 − 3 2
4
, lim− f (x) = 4
2
6= − 4
2
= lim+ f (x), so we reject c = 1 − 3 2
4
.
x→1 x→1

Therefore, the value of c for which the limit of f exists is c = 1 + 3 2
4
.
Next, we verify whether the value of the function at x = 1 equals the limit with this value of c. Thus
√ 

 √ √
1 + 3 4 2 (1)2 + 1 − 2 1 + 342 − 1 3 2
2
4
f (1) = 2 = = = .
(1) − 1 + 3 3 3 4
i.e. √
2
f (1) = = lim f (x) .
4 x→1

Hence, f is continuous at x = 1 if c = 1 + 3 2
4
.

70 MATH 122: Calculus I - Kojoga Aziedu


Example 3.47. Show that the function
 3
x −1
if x < 1


 2
g (x) = rx −1
x+k
if x ≥ 1



2
is continuous at x = 1 provided k = 72 .

Solution. Just like before, g is continuous at x = 1 if lim g (x) = g (1). However, lim g (x) exists
x→1 x→1
provided lim− g (x) = lim+ g (x). Now
x→1 x→1

x3 − 1 (x − 1) (x2 + x + 1)
lim− g (x) = lim− = lim
x→1 x→1 x2 − 1 x→1− (x − 1) (x + 1)
2
x +x+1 3
= lim− =
x→1 x+1 2
and
r r
x+k 1+k
lim+ g (x) = lim+ = .
x→1 x→1 2 2
For the limit to exist, r
1+k 3 1+k 9 9
= ⇒ = ⇒1+k =
2 2 2 4 2
or
9 9−2 7
k= −1= =
2 2 2
an requied. And since s
7
r
1+ 2 9 3
g (1) = = = = lim g (x) .
2 4 2 x→1
Hence, with k = 7/2, g is continuous at x = 1.

Example 3.48. For what value of λ is the function defined by


(
λ (x2 − 2x) x ≤ 0
f (x) =
4x + 1 x>0

continuous at x = 0? What about the continuity at x = 1?

Solution. For f to be continuous at x = 0,

lim f (x) = lim+ f (x) = f (0) .


x→0− x→0

Thus, since 0 ∈ (−∞, 0], we have

lim− λ x2 − 2x = lim+ (4x + 1) = λ (0)2 − 2 (0)


  
x→0 x→0
2 
⇒ λ (0) − 2 (0) = 4 (0) + 1 = 0

i.e.
0 = 1 = 0,
an impossiblity. Therefore, there is no value of λ for which f is continuous at x = 0.

71 MATH 122: Calculus I - Kojoga Aziedu


At x = 1, since 1 ∈ (0, ∞), we have

f (1) = 4 (1) + 1 = 5; lim (4x + 1) = 4 (1) + 1 = 5.


x→1

Thus
lim (4x + 1) = f (1) .
x→1

Hence at x = 1, f is continuous for any value of λ.

Example 3.49. Find the number z that makes


(
x−z
z+1
, x≤0
k (x) = 2
x + z, x > 0

continuous for every x.

Solution. Note that k (x) is continuous for every and so is continuous at x = 0 provided

k (0) = lim+ k (x) = lim− k (x) .


x→0 x→0

Now
0−z z
k (0) = =−
z+1 z+1
2
lim+ k (x) = 0 + z = z
x→0
0−z z
lim− k (x) = =− .
x→0 z+1 z+1
Thus
z
− = z ⇒ z (z + 1) = −z
z+1
or
z (z + 2) = 0 ⇒ z = 0, z = −2.
Hence the values of z for which k (x) is continuous everwhere is z = −2 and z = 0.

3.3.1.2 Continuity of Some Standard Functions.

Theorem 3.50. Let f be an algebraic function. If a ∈ Df , then f is continuous at a.

The table below shows some common functions (relevant to this course) that are continuous on their
domains.

72 MATH 122: Calculus I - Kojoga Aziedu


No Function, f (x) Example Interval of Continuity
1. Constant f (x) = c R ≡ (−∞, ∞)
2. Identity f (x) = x R ≡ (−∞, ∞)
3. Polynomial f (x) = an xn + an−1 xn−1 + · · · + a1 x + a0 R ≡ (−∞, ∞)
N (x)
4. Rational f (x) = , D (x) 6= 0 R\ {x : D (x) = 0}
D (x)
5. Absolute-value f (x) = |x| R ≡ (−∞, ∞)
6. Power f (x) = ax , a > 0 R ≡ (−∞, ∞)

7. Radical f (x) = x R≥0 ≡ [0, ∞)
8. Exponential f (x) = ex R ≡ (−∞, ∞)
9. Logarithmic f (x) = ln x R+ ≡ (0, ∞)
(d) 10 Sine f (x) = sin x R ≡ (−∞, ∞)
11. Cosine f (x) = cos x R ≡ (−∞, ∞)
12. Tangent R\ (2n + 1) π2 : n ∈ Z

f (x) = tan x
13. Secant f (x) = sec x R\ (2n + 1) π2 : n ∈ Z
14. Cosecant f (x) = csc x R\ {nπ : n ∈ Z}
15. Cotangent f (x) = cot x R\ {nπ : n ∈ Z}
16. Inverse sine f (x) = sin−1 x [−1, 1]
17. Inverse cosine f (x) = cos−1 x [−1, 1]
18. Inverse tangent f (x) = tan−1 x R
19. Inverse secant f (x) = sec−1 x (−∞, −1] ∪ [1, ∞)
20. Inverse cosecant f (x) = csc−1 x (−∞, −1] ∪ [1, ∞)
21. Inverse cotangent f (x) = cot−1 x R

The proof of these asertions of continuity follows from the property of limits. For example, let

p (x) = an xn + an−1 xn−1 + · · · + a1 x + a0

be a polynomial with a ∈ Dp = R. Then p (a) is finite and so is defined. Also, from the laws of limits,

lim p (x) = p (a) .


x→a

Hence, p is continuous on R.

Theorem 3.51. Properties of Continuous Functions.


Suppose k is a real number and f and g are continuous functions at x = a. Then the following functions
are also continuous at x = a:

• Scalar multiple: kf . • f ◦ g.
• Sum or difference: f ± g. • |f |.
• Product: f g.
• Existence of extreme values.
f
• Quotient: provided g (a) 6= 0. • Takes every value of points within its domain.
g

Proof. Follows from the laws of limits.

3.3.2 Types of Discontinuity.


There are four (4) types of the discontinuity of a function.

73 MATH 122: Calculus I - Kojoga Aziedu


3.3.2.1 Removable Discontinuity at a Point.

• A function is said to have a removable discontinuity at the point x = a when both f (a) and
lim f (x) = L but
x→a
lim f (x) = L 6= f (a) .
x→a

• A function that has a removable discontinuity at a point will have a “hole” in its graph at that
point.

• This class of discontinuities typically occur when rational functions have a (some) factor(s) from
their numerator and denominator, or through piecewise functions.

 Particularly, f (x) has a removable discontinuity at x = a whenever


0
f (a) = ;
0
 and if the multiplicity (the exponent of the common factor) of the factor (x − a) in the
numerator is greater or equal to the multiplicity of the factor in the denominator;
 otherwise, i.e. if multiplicity of the factor (x − a) in the numerator is less than the multipli-
city of the factor in the denominator, then x = a is a vertical asyptote.

• Revovable discontinuities can be removed! To remove this discontinuity, redefine a new function
F as follows: (
f (x) , if x 6= a
F (x) =
L, if x = a.
For all x other than a, we see that F (x) = f (x), and

lim F (x) = L = F (a) .


x→a

So the new function F is continuous at x = a.

Example 3.52. Suppose


x4 − 1
f (x) = , x ∈ R.
x−1

• Then f is not continuous at the point x = 1 since f is not defined at x = 1.

• Thus there is a “hole” at the point (1, 4) (since lim f (x) = 4) as shown left in the diagram below.
x→1

74 MATH 122: Calculus I - Kojoga Aziedu


• To remove this discontinuity, we redefine f as a piecewise function as follows;
 4
x − 1
, x 6= 1
f (x) = x − 1
4, x = 1.

Then
x4 − 1
= lim x2 + (x + 1)
  
lim f (x) = lim
x→1 x→1 x − 1 x→1
= 2 + 2 = 4 = f (1) .
It follows that f becomes a continuous function at x = 1 (right graph).
• The point x = 1 is a removable discontinuity.

Example 3.53. Discuss the continuity of the following function.



2
x , x < 2

f (x) = 2x, x > 2

1, x = 2.

Solution. By definition of f ,
f (2) = 1; lim− f (x) = 4 = lim+ f (x) ⇒ lim− f (x) = 4 6= 1 = f (2) .
x→2 x→2 x→2

Therefore, f (x) has a removable discontinuity at x = 2.

To remove this discontinuity, set f (2) = 4, so that f (2) = lim f (x) and define a new function F as
x→2

2
x , x < 2

F (x) = 2x, x > 2

4, x = 2.

Then F is continuous for all x.

75 MATH 122: Calculus I - Kojoga Aziedu


3.3.2.2 Jump Discontinuity.

• A function f is said to have a jump (or finite) discontinuity when the right- and left-hand limits both exi
Thus, if f has a jump discontinuity at x = a, then

lim f (x) 6= lim+ f (x) .


x→a− x→a

• The size of the jump in the discontinuity is the difference between the right- and left-hand limits,
i.e.
size of jump = lim− f (x) − lim+ f (x) .


x→a x→a

• A function possessing a finite number of jumps in a given finite interval is called a piecewise
continuous function (see sub-subsection 1.2.6).

Example 3.54. Consider the function


(
x, x ≤ 2
f (x) =
x2 , x > 2.

• f (2) = 2.

• lim f (x) = lim− x = 2.


x→2− x→2

• lim f (x) = lim+ x2 = 22 = 4.


x→2+ x→2

• Thus both lim− f (x) and lim+ f (x) but lim− f (x) 6= lim+ f (x). Therefore, there is a jump
x→2 x→2 x→2 x→2
discontinuity at x = 2 (see graph below).

• The jump of the discontinuity is




lim f (x) − lim f (x) = 4 − 2 = 2.
+x→2 −
x→2

Example 3.55. Graph the function f (x) = tan−1 1


and discuss its continuity.

x

76 MATH 122: Calculus I - Kojoga Aziedu


Solution. The graph of the function is show below.

From the graph, f is continuous everywhere except x = 0. At this point,


π π
lim− f (x) = − 6= = lim+ f (x) .
x→0 2 2 x→0
π  π
Thus there is a jump discontinuity at the point x = 0 with the jump size − − = π.
2 2

3.3.2.3 Infinite Discontinuity.

• A function f is said to possess an infinite discontinuity at the point x = a if one of the one-sided limits
of the function is infinite.

• That is whenever either lim− f (x) = ±∞ or lim+ f (x) = ±∞.


x→a x→a

Example 3.56. m

Solution. m

3.3.2.4 Essential Discontinuity.

• A function f is said to have an essential (or oscilliatory) discontinuity when the values of the
function appear to be approaching two or more values simultaneously.

Example 3.57. m

Solution. m

Example 3.58. Discuss the continuity of the following functions.

(x − 1) (2x + 1)
(a) f (x)
x (x − 1)

77 MATH 122: Calculus I - Kojoga Aziedu


(b)
(c) f (x) = {
(d)

Solution. m

3.3.3 Continuity on an Interval


Definition 3.59. Continuity on an Open Interval (a, b).
A function f is said to be continuous on an interval (a, b) if ∀x ∈ (a, b), f is continuous.
That is f is continuous on (a, b) if it is continuous at every point in (a, b).


Example 3.60. Show that f (x) = 1 − 1 − x2 is continuous on (−1, 1).

Solution. Let a ∈ (−1, 1). Then √


f (a) = 1 − 1 − a2 ,
which is defined. Also,
 √  √
lim f (x) = lim 1 − 1 − x2 = lim 1 − lim 1 − x2
x→a x→a x→a x→a
q √
= 1 − lim (1 − x ) = 1 − 1 − a2
2
x→a
= f (a) .
That is f is continuous at x = a if −1 < a < 1. Since a is arbitrary, we have that f is continuous at
every point in (−1, 1) and so continuous on (−1, 1).
x  π π
Example 3.61. Show that the function f (x) = is continuous on the interval − , .
cos x 2 2
x  π π  π π
Solution. Given f (x) = and the interval − , , let a ∈ − , . Then
cos x 2 2 2 2
a
f (a) =
cos a
and
 x  lim x a
lim f (x) = lim = x→a = = f (a) .
x→a x→a cos x lim cos x cos a
x→a
 π π
Since a is arbitrary, we conclude that f is continuous at x for all x ∈ − , . Hence, f is continuous
 π π 2 2
on − , .
2 2

Definition 3.62. Continuity on a Closed Finite Interval [a, b].


The function f is continuous on [a, b] if it is continuous on (a, b) and
lim f (x) = f (a) ; lim f (x) = f (b) .
x→a+ x→b−

That is f is continuous on (a, b) and at the points x = a, x = b.

78 MATH 122: Calculus I - Kojoga Aziedu



Example 3.63. We have seen from from Example 3.60 that the function f (x) = 1 − 1 − x2 is
continuous on (−1, 1). We want to establish here that f is indeed continuous on [−1, 1].
Now, q  √ 
f (−1) = 1 − 1 − (−1)2 = 1; lim f (x) = lim + 2
1 − 1 − x = 1,
x→−1+ x→−1
q  √ 
f (1) = 1 − 1 − (1)2 = 1; lim− f (x) = lim− 1 − 1 − x2 = 1.
x→1 x→1

Thus
lim f (x) = f (−1) = 1
x→−1+

and
lim f (x) = f (1) = 1.
x→1−

Hence, f is continuous on [−1, 1] as claimed.

Example 3.64. Study the continuity of the function


(
x2 , x < 2
f (x) =
4, x ≥ 2

on the interval [0, 4].

Solution. Clearly, f is defined on R and so on [0, 4]. Let c be a real number. Then

• if c < 2, then f (c) = c2 , and

lim f (x) = lim− x2 = c2 = f (c) .


x→c− x→c

Thus f is continuous for all x < 2.

• if c ≥ 2, then f (c) = 4, and


lim f (x) = lim+ 4 = 4 = f (c) .
x→c+ x→c

So f is continuous for all x ≥ 2.

• From the above observations, we can conclude that f is continuous at every point x ∈ R.

Example 3.65. Show that the function f (z) = (z − 1) (3 − z) is continuous on [1, 3].
p

Solution. To show that f is continuous on [1, 3], we first show that it is continuous on (1, 3) aand at
the end points x = 1, x = 3.

Let a ∈ (1, 3). Then f (a) = (a − 1) (3 − a) and


p

p q
lim f (z) = lim (z − 1) (3 − z) = lim (z − 1) (3 − z)
z→a z→a z→a
p
= (a − 1) (3 − a) = f (a) .

Thus f is continuous on (1, 3).

• For continuity at the end points,

79 MATH 122: Calculus I - Kojoga Aziedu


p √
 f (1) = (1 − 1) (3 − 1) = 0 = 0;
p q
lim+ f (z) = lim+ (z − 1) (3 − z) = lim (z − 1) (3 − z)
z→1 z→1 z→1+
p
= (1 − 1) (3 − 1) = 0 = f (1) ,

and
p √
 f (3) = (3 − 1) (3 − 3) = 0 = 0;
p q
lim− f (z) = lim− (z − 1) (3 − z) = lim (z − 1) (3 − z)
z→3 z→3 z→3−
p
= (3 − 1) (3 − 3) = 0 = f (3) .

Thus f is continuous at x = 1 and x = 3. Hence f is continuous on [1, 3].

Theorem 3.66. Extreme-value Theorem (Maximum-minimum Principle).


Let f (x) be a continuous function on the closed, finite interval [a, b]. Then ∃m, n ∈ [a, b] such that

f (m) ≤ f (x) ≤ f (n) .

The number M = f (m) is called the absolute minimum value of f and it occurs at x = m, and
N = f (n) is called the absolute maximum value of f and it occurs at x = n.

Example 3.67. Let us verify the Extreme-value Theorem for function


(
x2 , x<2
f (x) =
x + 2, x ≥ 2

on the interval [−1, 4].

• First of all we need to establish the continuity of f on [−1, 4]. Since both pieces of f are polyno-
mials, they are continuous on [−1, 4] except possibly at x = 2. We check that one too.

 f (2) = x + 2 = 4.
 lim f (x) = 22 = 4; lim f (x) = 2 + 2 = 4. Thus
x→2− x→2+

 lim f (x) = lim+ f (x) = f (2) = 2. Hence, f is continuous at x = 2 and so on [−1, 4].
x→2− x→2

• Let x ∈[−1, 4]. Then 0 ≤ f (x) ≤ 6.

• Hence, on the inteval [−1, 4], the absolute minimum value of f is 0 and the absolute maximum
value is 6, as shown in the graph below.

• These values occur at m, n ∈ [−1, 4] such that

f (n) = 0 ⇒ n2 = 0 ⇒ n = 0 or n + 2 = 0 ⇒ n = −2

and √
f (m) = 6 ⇒ m2 = 6 ⇒ m = ± 6 or m + 2 = 6 ⇒ m = 4.
√ √
• We reject n = −2 and m = − 6 since − 6, −2 ∈
6 / [−1, 4]. We choose m = 4 and n = 0.

80 MATH 122: Calculus I - Kojoga Aziedu


The Extreme-value Theorem has many applications in the real world, especially in business, where the
making the right decision in the right degree is absolutely crucial to maximizing profit. F or example,
it can help:

• the businessman to calculate the maximum and minimum prices that his business should charge
for its goods and services;
• a manager can calculate maximum and minimum overtime hours or productivity rates;
• a salesman can figure out how many sales he or she has to make in a year;
• a pharmacist to estimate the smallest dosage they can give to someone to still be effective, or the
largest dosage that they can give without causing harm; etc.

3.3.4 Intervals of Continuity.


.
.
.
.
.
.
.
Example 3.68. Let
x+2
f (x) = .
x3 − 7x + 6
Where is f continuous and where is it discontinuous?

81 MATH 122: Calculus I - Kojoga Aziedu


Solution. Since f is a rational function, it is continuous at those points in its domain and discontinuous
at those where it is undefined. That is, it discontinuous for those values of x for which x3 − 7x + 6 = 0.
Let g (x) = x3 − 7x + 6. Then g (1) = 0 and so (x − 1) is a factor of g (x), by the factor theorem. Diving
g (x) by (x − 1) via long division gives us

g (x) = x3 − 7x + 6 = (x − 1) x2 + x − 6


= (x − 1) (x − 2) (x + 3) .

Hence, g (x) = 0 ⇒ x = −3, x = 1, x = 2. Thus f is continuous everywhere except at x = –3, x = 1,


and x = 2 where it is discontinuous. In otherwords, f is continuous on (−∞, −3) ∪ (1, 2) ∪ (2, ∞). The
discontinuities here are infinite discontinuities.

Figure 3.9: Graph of f showing interval of continuity and points of discontinuity.

Example 3.69. Let f be defined by


x2 − 1
f (x) = .
|x2 − 1|
Where is f continuous and where is it discontinuous? At points of discontinuity if any, explain why it
is discontinuous.

Solution. By definition,
(
2
x − 1 = x2 − 1 if x2 − 1 > 0 ⇒ |x| > 1
− (x2 − 1) if x2 − 1 < 0 ⇒ |x| < 1.

If x2 − 1 > 0 ⇒ |x| > 1, then |x2 − 1| = x2 − 1, so


x2 − 1
f (x) = =1
x2 − 1
and if x2 − 1 < 0 ⇒ |x| < 1, then |x2 − 1| = − (x2 − 1), so
x2 − 1
f (x) = = −1.
− (x2 − 1)
But f is not defined at x = 1 or x = –1. Hence, f is continuous everywhere except at x = 1 and
x = –1. It is discontinuous at these two points because it’s not defined at any of them. These are jump
discontinuities with jump size 2.

82 MATH 122: Calculus I - Kojoga Aziedu


Figure 3.10: Graph of f showing interval of continuity and points of discontinuity.

Example 3.70. Let f be defined by


 2
x − x − 2
if x 6= 1
f (x) = x+1
−3 if x = 1.

Where is f continuous and where is it discontinuous? At points of discontinuity if any, explain why it
is discontinuous.
x2 − x − 2
Solution. The rational function is continuous for all real values of except x = −1, ∀x ∈
x+1
R\ {−1}, it is continuous.
At x = −1, we have
x2 − x − 2 (x + 1) (x − 2)
lim f (x) = lim = lim
x→−1 x→−1 x+1 x→−1 x+1
= lim (x − 2) = −1 − 2 = −3 = f (−1) .
x→−1

Thus, f is also continuous at x = –1. Hence, f is continuous everywhere on the real line. The graph of
f is the same as the line y = x–2. Thus there exists a removed discontinuity at x = −1.

Figure 3.11: Graph of f showing interval of continuity and points of discontinuity.

Example 3.71. Given the function


1
f (x) = ,
x−1
find the points of discontinuity of the composite function y = f (f (x)).

83 MATH 122: Calculus I - Kojoga Aziedu


Solution. Clearly, f is discontinuous at x = 1. Now, for x 6= 1, we have
 
1 1 1 x−1
y=f = 1 = 2−x =
x−1 x−1
−1 x−1
2−x

which is discontinuous at x = 2. Hence, the points of discontinuity are x = 1 and x = 2.

3.3.5 Intermediate Value Theorem (IVT)


Theorem 3.72. The IVT.
Suppose f is a continuous function on [a, b] and k ∈ (f (a) , f (b)). Then there is at least a number
c ∈ (a, b) such that f (c) = k.

That is, every continuous function f on [a, b] takes every intermediate between the function values f (a)
and f (b).

Figure 3.12: The intermediate-value theorem

Example 3.73. Consider the function f (x) = x2 − 1. Then clearly, f is continuous on R, and for
that matter on [0, 4], since it is a polynomial, by Theorem 3.3.1.2. Now, f (0) = (0)2 − 1 = −1 and
f (4) = (4)2 − 1 = 15.
The IVT is saying that since f is continuous on [0, 4] ⊂ R, then ∀k ∈ (−1, 15), ∃c ∈ (0, 4) such that
k = f (c).
Choose, for example, 8 ∈ (−1, 15) or −1 = f (0) < 8 < 15 = f (4). Then 3 ∈ (0, 4) and f (3) = 8.

84 MATH 122: Calculus I - Kojoga Aziedu


Figure 3.13: Verifying the IVT

Example 3.74. Prove that 2 is the image of an element in − π2 , π2 under the function f (x) =


x (sin x + 1).

Solution. Notice that f is the product of two continuous functions on R and so is continuous on R.
Now, at the extremes of the interval, we have
 π π   π  π
f − =− sin − + 1 = − (−1 + 1) = 0 < 2
2 2 2 2
and π  π  π   π
f = sin + 1 = (1 + 1) = π > 2.
2 2 2 2
i.e.  π π 
f − <2<f .
2 2
Hence, by the IVT,  π π
∃c ∈ − , : f (c) = 2.
2 2
Therefore 2 is the image of an element in − π2 , π2 under f .


Note: The IVT does not indicate the value of the c ∈ [a, b]; it only determines its existence.

3.3.6 Application of the IVT: Locating Roots of Equations


Theorem 3.75. Bolzano’s Theorem (Root-location Theorem).
Suppose f is a continuous function on [a, b] and that

f (a) · f (b) < 0, (3.36)

i.e. f (a) and f (b) have opposite signs. Then ∃c ∈ (a, b) such that f (c) = 0.

85 MATH 122: Calculus I - Kojoga Aziedu


The number c ∈ (a, b) is called a root of f or a solution to the equation f (x) = 0 on (a, b).

Figure 3.14: Bolzano’s theorem

Example 3.76. Let us verify that the equation x3 + x − 1 = 0 has at least one real solution in the
interval [0, 1].

Set f (x) = x3 + x − 1. Then clearly, f is continuous on on [0, 1] since it is a polynomial. Now

f (0) = (0)3 + 0 − 1 = −1 < 0

and
f (1) = (1)3 + 1 − 1 = 1 > 0.
Thus
f (0) · f (1) = −1 · 1 = −1 < 0.
Hence, by Bolzano’s Theorem, there is at least a number λ ∈ (0, 1) such that f (λ) = 0 with x = λ be
the root of the equation x3 + x − 1 = 0 in [0, 1].

Example 3.77. Show that x3 − x2 − 1 = 0 ha at least one root in (1, 2).

Solution. Let f (x) = x3 − x2 − 1. Then since f (x) is a polynomial, it is continuous on R, and so on


[1, 2]. Now,
f (1) = (1)3 − (1)2 − 1 = −1 < 0
and
f (2) = (2)3 − (2)2 − 1 = 3 > 0.
Thus f (1) · f (2) = (−1) (3) = −3 < 0. Therefore, by the IVT, ∃c ∈(1, 3) : f (c) = 0.

86 MATH 122: Calculus I - Kojoga Aziedu


Figure 3.15: Graph of f showing root in (1, 2)

Example 3.78. Let f (x) = x3 − x2 + x. Show that there exists a solution to the equation f (x) = 10.

Solution. Let g (x) = x3 − x2 + x − 10 = f (x) − 10. Then g is a polynomial, and so is continuous on


R. Also, by inspection,
g (2) = (2)3 − (2)2 + 2 − 10 = −4
and
g (3) = (3)3 − (3)2 + 2 − 10 = 11.
Hence, by the IVT, ∃ε ∈ (2, 3) ⊂ R such that g (ε) = 0. Hence, the equation f (x) = 10 has a solution
in R as

Figure 3.16: Graph of f showing root in (1, 2)

required.

87 MATH 122: Calculus I - Kojoga Aziedu


3.3.6.1 The Interval Bisection Method.

3.3.6.2 The Newton-Raphson Method

Ach, Single
.
.
.
.

88 MATH 122: Calculus I - Kojoga Aziedu


3.4 Exercises on Continuity.
1. ........Continuity at points .....
2. .......Continuity on intervals ......
3. Let x ∈ R. Show that the functions f (x) = cos x and g (x) = sin x are continuous on R.
4. Determine the interval of continuity of each of the following functions.
3
(a) f (x) = 12 x + 3 |x + 2|
(c) f (x) =
2−x
2 1
(b) f (x) = (d) f (x) =
2x − 3 1 + cos x

5. Let 
x + 2 if x < 2
 3

f (x) = 5 if x = 2
x + 6 if x > 2.

 2

Is f continuous at x = 2? Give reasons to support your claim. Draw the graph of f .


6. Explain why the function f defined by
(
x2 −2x−8
x−4
if x 6= 4
f (x) =
3 if x = 4

is discontinuous at x = 4. Define a new function F (x) such that F is continuous everywhere.


7. Show that the function (
2x−6
x−3
if x 6= 3
f (x) =
2 if x = 3
is continuous at x = 3.
8. Find the relationship between the constants a and b so that the function f defined by
(
ax + 1 x ≤ 3
k (x) =
bx + 3 x > 3

is continuous at x = 3.
9. Find the values of h and k to that the following function f is continuous for all x.

x + h, if x < −1
 2

f (x) = x + 4, if − 1 < x ≤ 1
if x > 1.

 x
x+k
,

10. Find the numbers k so that the following functions are all continuous for all x.
( (
x2 , x≤2 x2 − x + 1, x ≤ 1
(a) f (x) = (c) f (x) =
k − x2 , x>2 kx2 + 1, x>1
(d) m
(
x − k, x<3
(b) f (x) =
1 − kx, x ≥ 3.

11. Find the value(s) of the constant a of that the given function is continuous at the designated
point.

89 MATH 122: Calculus I - Kojoga Aziedu


 4 (
x − 1 3x2 − 11x − 4, x ≤ 4
, x 6= 1 (e) f (x) = ,x = 4
(a) f (x) = x − 1 ,x = 1 kx2 − 2x − 1, x > 4
a, x=1


−6x − 12, x < −3
 3 2
x + x − 16x + 20 
, x 6
= 2
, x = (f) f (x) = k − 5k,
 2
(b) f (x) = (x − 2)2 x = −3 , x = −3
6, x > −3

a, x=2

2 (
  , x
x sin 1 , x 6= 0 (g) f (x) =

(c) f (x) = x , x=0 , x


a, x=0 (h) m

(
ax + 1, x ≤ 5
(d) f (x) = ,x=5
3x − 5, x > 5

12. find the value of a for which f is continuous at x = 1.

13. Determine the intervals of continuity for the function


(
x2 + 1, if x ≤ 0
f (x) =
3x + 5, if x > 0.

14. Let the function f be defined as



sin ax
 x
 if x < 0
f (x) = 5 if x = 0
x + b if x > 0.

Find the constants a and b so that f is continuous everywhere.

15. For what values of the constant c is the function f defined by


(
cx + 1 if x ≤ 3
f (x) =
cx2 − 1 if x > 0

continuous on R?

16. Suppose f is defined as


(
0 if x is an rational number
f (x) =
1 if x is an irrational number.

Prove that f is discontinuous at every point p ∈ R.

17. Sketch the graph of the function


x2 + 5x − 24
f (x) = .
x2 − x − 6
(a) At what point is the removable discontinuity for the function?
(b) With this graph, find

i. lim f (x) iv. lim+ f (x)


x→−∞ x→3
ii. lim f (x) v. f (3)
x→∞
iii. lim− f (x) vi. any discontinuities.
x→3

90 MATH 122: Calculus I - Kojoga Aziedu


18. Prove the following claims.

(a) The function f (x) = 2 sin (x + 3) cos 1 2


takes the number 10 in the interval [0, 5].

2
x
(b) The function p (x) = 2x3 − 5x2 − 10x + 5 has a root somewhere between −1 and 2.
(c) There are some xi ∈ [−4, 4] , i = 1, 2, 3 such that equation f (xi ) = 0 where f (x) = x3 −
12x + 2.
(d) The equations x3 − 6x  π+ 11x − 6 = 0 and x = cos x have at least one root each on the
2

intervals [2.5, 4] and 0, 2 respectively.


(e) The equation ln x = e−x has a root between 1 and 2.
(f) The equation tan−1 x + x = 1 has at least one real root [Hint: Carefully choose your own
interval].

19. Show that there exist at least one root of the equation x3 − 3x + 1 = 0 between 0 and 1.

20. Does each of the following equations have solutions on the stated intervals?

(a) x2 − 5 = 0 on [2, 3] (c) x4 + 2x = 1 on [1, 4]


(b) x3 − 3x2 + 1 = 0 on [0, 1] (d) x5 − 5x3 + 3 = 0 on [−3, −2]

21. m

91 MATH 122: Calculus I - Kojoga Aziedu


Chapter 4

Differentiation.

4.1 The Concept of the Derivative of a Function

4.1.1 The Gradient of a Straight Line


The gradient of a straight line passing through the points P (x1 , y1 ) and Q (x2 , y2 ) is given by

∆y y2 − y1
Gradient = = .
∆x x2 − x1

Figure 4.1: Gradient of a straight line through two points.

The equation of such a line is


y2 − y1
y= (x − x1 ) + y1
x2 − x1
where (x, y) denotes any point on the line.

4.1.2 The Gradient to a Curve at a Point and the Derivative


Consider a curve whose equation is given by y = f (x) and whose graph is shown below:

92
Figure 4.2: Graph of the curve y = f (x) showing the gradient of chord P Q.

Then gradient of chord P Q is


f (x + h) − f (x) f (x + h) − f (x)
Gradient = = .
(x + h) − x h
Now, as h → 0, x + h → x, Q → P and eventually, the chord P Q becomes a tangent to the curve at P .
Therefore, the gradient of the curve y = f (x) at the point P (x, y) ≡ P (x, f (x)) is
f (x + h) − f (x)
lim
h→0 h
provided this limit exists.
Definition 4.1. The Derivative (Derived Function) of a Function.
The function
f (x + h) − f (x)
f 0 (x) = lim (4.1)
h→0 h
is called the derivative (or the derived function) of f with respect to x at the point P (x, f (x)), provided
the limit exists.
Also, the derivative of f at the point x = a is given by
f (x) − f (a)
f 0 (a) = lim . (4.2)
x→a x−a
The process of finding the derivative of a function is called differentiation.

Notations:
The notation f 0 (x) is called the derivative of f at x. Other notations are
dy df df (x)
= y 0 (x) = y 0 = Dx y = = = Dx f (x) = Df (x) ,
dx dx dx
etc, where y = f (x).

4.1.3 Differentiation from First Principles.


Definition 4.2. The process of differentiation a function using the defintion of the derivative (using
limits) is called differentiation from first principles. That is, differentiating a function from first prin-
ciples (or using the definition of the derivative) is the process of finding the derivative of the function
using the formula
f (x + h) − f (x)
f 0 (x) = lim
h→0 h

93 MATH 122: Calculus I - Kojoga Aziedu


or
f (x) − f (a)
f 0 (a) = lim . (4.3)
x→a x−a

Example 4.3. Differentiate the following functions from first principles


√ √
(a) f (x) = x3 (c) f (x) = x (e) f (x) = x3 − 1

1 x+3
(b) f (x) = (d) f (x) = 3x − x2 (f) f (x) = √ .
x2 1+ x

Solution.

(a) f (x) = x3 ⇒ f (x + h) = (x + h)3 = x3 + 3x2 h + 3xh2 + h3 .


Therefore, from first principles, we have

f (x + h) − f (x) x3 + 3x2 h + 3xh2 + h3 − x3


f 0 (x) = lim = lim
h→0 h h→0 h
3x2 h + 3xh2 + h3 h (3x2 + 3xh + h2 )
= lim = lim
h→0 h h→0 h
= lim 3x + 3xh + h = 3x + 3x (0) + (0)2
2 2 2

h→0
2
= 3x .

1 1
(b) f (x) = ⇒ f (x + h) = .
x 2
(x + h)2

1 1
2 − 2
f (x + h) − f (x) (x + h) x
∴ f 0 (x) = lim = lim
h→0 h ! h→0 h
2
1 x2 − (x + h) 1 x2 − (x2 + 2xh + h2 )
 
= lim = lim
h→0 h x2 (x + h)2 h→0 h x2 (x + h)2
1 −2xh − h2
   
1 h (−2x − h)
= lim = lim ·
h→0 h x2 (x + h)2 h→0 h x2 (x + h)2
−2x − h lim (−2x − h)
h→0
= lim =
h→0 x2 (x + h)2 lim x2 (x + h)2
h→0
−2x − 0 −2x 2
= 2 = 4 = − 3.
x2 (x + 0) x x
√ √
(c) f (x) = x ⇒ f (x + h) = x + h.

√ √
f (x + h) − f (x) x+h− x
∴ f 0 (x) = lim = lim
h→0 h h→0 h
√ √ √ √  !
x+h− x x+h+ x x+h−x
= lim ×√ √ = lim √ √ 
h→0 h x+h+ x h→0 h x+h+ x
!
h 1
= lim √ √  = lim √ √
h→0 h x+h+ x h→0 x+h+ x
1 1 1
= √ √ =√ √ = √ .
x+0+ x x+ x 2 x

94 MATH 122: Calculus I - Kojoga Aziedu


(d) Given that f (x) = 3x2 − x, we have f (x + h) = 3 (x + h)2 − (x + h).
2 2

f (x + h) − f (x) 3 (x + h) − (x + h) − 3x − x
f 0 (x) = lim = lim
h→0 h h→0 h
2 2 2
3x + 6hx + 3h − x − h − 3x + x 6hx + 3h2 − h
= lim = lim
h→0 h h→0 h
= lim (6x + 3h − 1) = 6x + 0 − 1 = 6x − 1.
h→0

√ q
(e) f (x) = x3 − 1 ⇒ f (x + h) = (x + h)3 − 1.
q √
f (x + h) − f (x) (x + h)3 − 1 − x3 − 1 0
∴ f 0 (x) = lim = lim = ,
h→0 h h→0 h 0
an indeterminacy. Rationalising with the conjugate of the binomial term containing the radical,
we have
q
3
√ q
3
√ 
(x + h) − 1 − x 3−1 (x + h) − 1 + x 3−1
f 0 (x) = lim  ×q √

h→0 h 3
(x + h) − 1 + x3 − 1
(x + h)3 − 1 − (x3 − 1) x3 + 3x2 h + 3xh2 + h3 − 1 − x3 + 1
= lim q  = lim
√ √
q 
h→0 3 h→0 3
h 3
(x + h) − 1 + x − 1 h 3
(x + h) − 1 + x − 1

3x2 h + 3xh2 + h3 3x2 + 3xh + h2


= lim  = lim q √

q
h→0 h→0
h (x + h)3 − 1 + x3 − 1 (x + h)3 − 1 + x3 − 1

3x2 + 3x (0) + (0)2 3x2


= q √ = √ .
(x + (0))3 − 1 + x3 − 1 2 x3 − 1

x+3 x+h+3
(f) f (x) = √ ⇒ f (x + h) = √ . Therefore,
1+ x 1+ x+h
x+h+3 x+3
√ − √
0 f (x + h) − f (x) 1+ x+h 1+ x 0
f (x) = lim = lim = ,
h→0 h h→0 h 0
so,
√ √  !
1 (x + h + 3) (1 + x) − 1 + x + h (x + 3)
f 0 (x) = lim √  √
h→0 h 1 + x + h (1 + x)
" √ √ #
1 (x + 3) h (1 + x) − 1 − x + h
= lim √  √ =
h→0 h 1 + x + h (1 + x)
.
.
. √
x+2 x−3
= √ 2√ .
2 (1 + x) x

95 MATH 122: Calculus I - Kojoga Aziedu


4.1.4 Differentiability of a Function at a Point or on an Interval.
Definition 4.4. Differentiable Functions.
.
.
.
.

Definition 4.5. Non-differentiable Functions.


A f (x) is said to be non-differentiable (or has no derivative) at the point x = a if
lim f (x) 6= lim+ f (x) .
x→a− x→a

That is a function has no derivative at the point x = a if its limit as x → a does not exist.

Example 4.6. Show that the function f defined by


f (x) = |x|
is not differentiable at the point x = 0.

Solution. By definition, (
x if x ≥ 0
f (x) = |x| =
−x if x < 0.
Also, provided this limit exists,
f (x + h) − f (x)
f 0 (x) = lim .
h→0 h
Hence,
f (0 + h) − f (0) f (h) − 0 f (h) |h|
f 0 (0) = lim = lim = lim = lim .
h→0 h h→0 h h→0 h h→0 h

since f (0) = |0| = 0. But


|h| −h
lim− = lim = lim (−1) = −1
h→0 h h→0 h h→0

and
|h| h
lim+ = lim = lim (1) = 1.
h→0 h h→0 h h→0

Thus,
lim f (x) 6= lim+ f (x)
h→0− h→0

and so lim f (x) does not exist. Hence, f (0) does not exist or f is non-differentiable at x = 0
0
h→0

Example 4.7. Find the values of the constants a and b so the function f defined by
(
ax + b if x < 0
f (x) =
2 sin x + 3 cos x if x ≥ 0

is differentiable at the point x = 0.

96 MATH 122: Calculus I - Kojoga Aziedu


Solution. First of all, for f (x) to be differentiable at x = 0, it must be continuous there too. Hence

lim f (x) = f (0) ⇒ a (0) + b = 2 sin (0) + 3 cos (0) ⇒ b = 3.


x→0−

Second, since f (x) is differentiable at x = 0. i.e.

lim f 0 (x) = f 0 (0) .


x→0−

But (
a if x < 0
f 0 (x) =
2 cos x − 3 sin x if x ≥ 0.
Thus
a = 2 cos (0) − 3 sin (0) ⇒ a = 2.
Hence, the values of a and b for which f is differentiable at x = 0 are a = 2, b = 3.

Example 4.8. Let f (x) = x |x|, for all x ∈ R. Discuss the derivability of f (x) at x = 0.

Solution. Note that a function is derivable at at point x = a if the left- and right-hand derivatives
exist and are equal. i.e.
f−0 (a) = f+0 (a) .
Let us write f (x) as (
x2 , x≥0
f (x) = x |x| = 2
−x , x < 0.
Then
f (0 + h) − f (0) −h2 − 0
f−0 (a) = lim− = lim− = − lim− h = 0
h→0 h h→0 h h→0

and
f (0 + h) − f (0) h2 − 0
f+0 (a) = lim+ = lim+ = lim+ h = 0.
h→0 h h→0 h h→0

And since f− (0) = 0 = f+ (0), f is differentiable at x = 0.


0 0

Example 4.9. Let  √


2 + x, x ≥ 1
f (x) = 1 5
 x + , x < 1.
2 2
Show that f is differentiable at x = 1.

Solution. By definition,
f (x + h) − f (x) f (1 + h) − f (1)
f 0 (x) = lim ⇒ f 0 (1) = lim .
h→0 h h→0 h
Note that h can be either positive or negative (i.e h can be taken to the right or left of 1), and so
(
1 + h > 1 when h > 0;
h is
1 + h < 1 when h < 0.

Thus,  √
2 + 1 + h, if h > 1
f (1 + h) = 1 5
 (1 + h) + , if h < 1.
2 2

97 MATH 122: Calculus I - Kojoga Aziedu


Now,
√ √ √
f (1 + h) − f (1) 2+ 1+h−2+ 1 1+h−1
f+0 (1) = lim+ = lim+ = lim+
h→0 h h→0 h" h→0 h
√ √  #
1+h−1 1+h+1 1+h−1 h
= lim+ ·√ = lim+ √  = lim √ 
h→0 h 1+h+1 h→0 h 1+h+1 h→0 h
+
1+h+1
1 1 1
= lim+ √ =√ = .
h→0 1+h+1 1+0+1 2

Similarly,
1
(1 + h) + 52 − 12 (1) + 5

0 f (1 + h) − f (1) 2 2
f− (1) = lim− = lim−
h→0 h h→0 h
1 1 5 1 5 1
+ h+ 2 − 2 − 2 h 1 1
= lim− 2 2 = lim− 2 = lim− = .
h→0 h h→0 h h→0 2 2
Thus, both one-sided limits exist and are equal, so that f is differentiable at x = 1 with

f (1 + h) − f (1) 1
f 0 (1) = lim = .
h→0 h 2

Example 4.10. Let (


1
, if x 6= 0

x2 sin x
f (x) =
0, if x = 0.

Show that f is differentiable at x = 0.

Example 4.11. Determine points of non-differentiability of the following functions.


1
(a) f (x) = (x − 1) 3 (b) f (x) = |x + 2| x2
(c) f (x) =
x−1

Solution.

(a) By the power rule of differentiating functions (see Theorem 4.21), we have that the derivative
of f is
1 1 1 2 1
f 0 (x) = (1) (x − 1) 3 −1 = (x − 1)− 3 = q .
3 3
3 (x − 1)3
q
Thus f is defined everywhere except when 3 (x − 1)3 = 0 ⇒ x = 1. Hence f is differentible for
0

all x except when x = 1. Indeed


1 1

0 f (1 + h) − f (1) (1 + h − 1) 3 − (1 − 1) 3
f (1) = lim = lim
h→0 h h→0 h
1
h 3 1
= lim = lim 2 = ∞,
h→0 h h→0 h 3

confirming that f is not differentiable at x = 1.

98 MATH 122: Calculus I - Kojoga Aziedu


(b) Note that (
x+2 if x ≥ −2
f (x) = |x + 2| =
− (x + 2) if x < −2

and that both pieces of f are polynomials and so are differentiable on R. The only point where
something can go wrong is when x = −2. Computing the the left- and right-hand limits of f at
this point, we have

f (−2 + h) − f (−2) −h
f−0 (−2) = lim − = lim − = −1
h→−2 h h→−2 h
and
f (−2 + h) − f (−2) h
f+0 (−2) = lim + = lim + = 1.
h→−2 h h→−2 h

Thus f (−2) does not exist since f−0 (−2) 6= f+0 (−2). Therefore, f is differentiable at every point
0

except x = −2.
x2
(c) By the quotient rule, f (x) = is differentiable everywhere except at x = 1. However, x = 1
x−1
is not in the domain of f , and so f is differentiable at every point of its domain.

Example 4.12. Let f be the piecewise function defined as



2
−x , x < 0

f (x) = 0, x=0

sin x, x > 0.

Evaluate f 0 (0).

Solution. For the given function f ,

f (0 + h) − f (0) (−h)2 − 02 h2
f−0 (0) = lim− = lim− = lim−
h→0 h h→0 h h→0 h
= lim− h = 0
h→0

and
f (0 + h) − f (0) sin (h) − sin (0) sin h
f+0 (0) = lim+ = lim+ = lim+ = 1.
h→0 h h→0 h h→0 h
Since f−0 (0) = 0 6== 1 = f+0 (0), we conclude that f 0 (0) does not exist.

Example 4.13. A function f satisfies the following equation

f (4h) + f (2h) + f (h) + f 12 h + f 41 h + f 1


  
h + ···
lim 8
= 64. (4.4)
h→0 h

Given that f (0) = 0 and that f 0 (0) exists, determine f 0 (0).

99 MATH 122: Calculus I - Kojoga Aziedu


Solution. By definition,

f (0 + h) − f (0) f (h) − 0 f (h)


f 0 (0) = lim = lim = lim .
h→0 h h→0 h h→0 h
Consider
f (nh)
lim , n∈R (4.5)
h→0 h
and let z = nh. Then as h → 0, z → 0. Thus (4.5) becomes

f (z) f (z)
lim h
= n lim .
z→0
n
z→0 z

Hence, rewriting (4.4) as


!
f (4h) f (2h) f (h) f 12 h f 41 h f 18 h
  
lim + + + + + + · · · = 64,
h→0 h h h h h h

we have
f (z) f (z) f (z) 1 f (z) 1 f (z) 1 f (z)
64 = 4 lim + 2 lim + lim + lim + lim + lim + ···
z→0 z z→0 z z→0 z 2 z→0 z 4 z→0 z 8 z→0 z
1 1 1
= 4f 0 (0) + 2f 0 (0) + f 0 (0) + f 0 (0) + f 0 (0) + f 0 (0) + · · ·
2 4 8

  !
1 1 1 X 1
4 + 2 + 1 + + + + · · · f 0 (0) = f 0 (0) 7 + i
.
2 4 8 i=1
2


X 1
The term i
is a geometric series with first term 1
2
and common ratio 1
2
< 1, whose value is
i=1
2

∞ 1
X 1 2
i
= 1 = 1.
i=1
2 1 − 2

Thus
64
64 = f 0 (0) (7 + 1) ⇒ f 0 (0) = = 8.
8
Therefore, the value of f 0 (0) is 8.

Example 4.14. Let f (x) = x4 + ax2 + bx be a function such that

f (x) − f (2)
lim =4
x→2 x−2
and
f (x) − f (1)
lim = 9.
x→1 x2 − 1
Show that the value of (b − a) is 77.

Solution. Given f (x) = x4 + ax2 + bx, we have

f (2) = (2)4 + a (2)2 + b (2) = 16 + 4a + 2b

and
f (1) = (1)4 + a (1)2 + b (1) = 1 + a + 2.

100 MATH 122: Calculus I - Kojoga Aziedu


Thus
f (x) − f (2) x4 + ax2 + bx − (16 + 4a + 2b)
4 = lim = lim
x→2 x−2 x→2 x−2
4 2
x − 16 + a (x − 4) + b (x − 2)
= lim
x→2 x−2
(x − 2) (x + 2) (x2 + 4) + a (x − 2) (x + 2) + b (x − 2)
= lim
x→2 x−2 
2
= lim (x + 2) x + 4 + a (x + 2) + b = (2 + 2) 22 + 4 + a (2 + 2) + b
  
x→2
= 32 + 4a + b
⇒ 4a + b = −28. (4.6)

Similarly,

f (x) − f (1) x4 + ax2 + bx − (1 + a + b)


9 = lim = lim
x→1 x2 − 1 x→1 x2 − 1
4 2
x − 1 + a (x − 1) + b (x − 1)
= lim
x→1 x2 − 1
(x − 1) (x + x2 + x + 1) + a (x − 1) (x + 1) + b (x − 1)
3
= lim
x→1 (x − 1) (x + 1)
3 2
x + x + x + 1 + a (x + 1) + b 1 + 1 + 1 + 1 + a (1 + 1) + b 4 + 2a + b
= lim = =
x→2 x+1 1+1 2
⇒ 2a + b = 14. (4.7)

We solve (4.6) and (4.7) simultanueously:


(4.6) − (4.7) ⇒ 2a = −42 ⇒ a = −21 and so b = 14 − 2 (−21) = 56.

∴ b − a = 56 − (−21) = 77.

Example 4.15. A function f satisfies the following relation:

f (ab) = f (a) + f (b) ∀a, b ∈ R+ .

Given that f 0 (1) = k < ∞, find f (x).

Solution. Let f 0 (1) = z. Then by definition,

f (1 + h) − f (1)
f 0 (1) = lim := k.
h→0 h
Let a = 1 ∈ R+ . Then
f (b) = f (1 · b) = f (1) + f (b) ⇒ f (1) = 0.
Let x ∈ R+ . Then x + h = x 1 + hx so that


    
h h
f (x + h) = f x 1 + = f (x) + f 1 + .
x x

Therefore,
h

0 f (x + h) − f (x) f (x) + f 1 + x
− f (x)
f (x) = lim = lim
h→0 h h→0 h
h
f 1+ x
= lim .
h→0 h

101 MATH 122: Calculus I - Kojoga Aziedu


h

f 1 +
Suppose f 0 (x) exists. Then lim x
also exists. Let z = hx so that z → 0 and h → 0. Then
h→0 h
f 1 + hx f 1 + hx − 0
 
f (1 + z) − f (1)
lim = lim = lim
h→0 h h→0 h z→0 xz
1 f (1 + z) − f (1) 1 0 k
= lim = f (1) = .
x z→0 z x x
Thus
k
f 0 (x) =
,
x
a standard first order differential equation. The solution (i.e. the function whhose derivative is xk , see
theorem 4.45, subsection (iii) and subsection 8.7 for more) is

f (x) = k ln x,

for some constant k to be determined. Since f (1) = 0 (i.e. set a = b = 1 into the given equation), we
have that k = 1 and
f (x) = ln x.

Example 4.16. By interpreting the expressions as appropriate derivatives, evaluate the following limits.

e3x − 1 10i − 1
(a) lim (c) lim
x→0 x i→0 i
h √ i2
−1 3
9 sin 2
+h − π2 2
ex − 1
(b) lim (d) lim
h→0 h x→0 x

Solution.

(a) Comparing
e3x − 1 e3x − e0
lim = lim 3
x→0 x x→0 3x − 0
with
f (z) − f (0)
3 lim ,
z→0 z−0
we have f (z) = ez where z = 3x and z → 0 as x → 0. Thus

e3x − 1 f (z) − f (0)


lim = 3 lim
x→0 x z→0 z−0
0
= 3f (0) = 3e0 = 3.

e3x − 1
∴ lim = 3.
x→0 x
h √ i2
9 sin−1 23 + h − π2
(b) lim
h→0 h
10i − 1
(c) lim
i→0 i
2
ex − 1
(d) lim
x→0 x

102 MATH 122: Calculus I - Kojoga Aziedu


Example 4.17. m

Solution.

Theorem 4.18. Differentiability Implying Continuity.


If f is a differentiable at the point a, then f is continuous at a.

That is if f is differentiable at a point, then it is continuous at that point too.


Contrapositively, this theorem says that if f (x) is not continuous at the point x = a then it is also not
differentiable there.

Proof. Recall that f is continuous at x = a if a ∈ Df , lim f (x) exists and lim f (x) = f (a).
x→a x→a

So suppose x ∈ Df and that x 6= a. Suppose also that f is differentiable at x = a. Then


f (x) − f (a)
f 0 (a) = lim .
x→a x−a
Now, notice that
f (x) − f (a)
f (x) − f (a) = · (x − a) ,
x−a
and so
 
f (x) − f (a)
lim (f (x) − f (a)) = lim · (x − a)
x→a x→a x−a
f (x) − f (a)
= lim · lim (x − a)
x→a x−a x→a
0
= f (a) (a − a) = 0

since f is differentiable at a. Thus,

lim (f (x) − f (a)) = lim f (x) − lim f (a) = 0.


x→a x→a x→a

Therefore,

lim f (x) = lim f (a) = f (a) .


x→a x→a

That is, f is continuous at x = a, completing the proof.

Caution: The converse of this statement is not always true. That is continuity does not necessarily
imply differentiability. For example, the function f (x) = |x| is continuous at x = 0 but not differentiable
at x = 0.

Example 4.19. m

Solution.

103 MATH 122: Calculus I - Kojoga Aziedu


Example 4.20. m

Solution.

Theorem 4.21. Derivative of a Power Function.


Let f (x) = xn , n ∈ R the power function. Then its derivative is given by

f 0 (x) = nxn−1 . (4.8)

Proof. We shall prove this in three folds; first for n ∈ {0} ∪ Z+ , then for n ∈ Z− (see Theorem 4.26)
and later for n ∈ Q. (see Theorem 4.58)
Case I: n ∈ R≥0 .
Given that f (x) = xn ⇒ f (x + h) = (x + h)n . But
n  
n
X n
(x + h) = xn−k hk
k
k=0
n (n − 1) xn−2 h2
= xn + nxn−1 h + + · · · + hn .
2!
Hence, by the definition of the derivative, we have

0 f (x + h) − f (x) (x + h)n − xn
f (x) = lim = lim
h→0 h h→0 h
n−2 2
n (n − 1) x h
xn + nxn−1 h + + · · · + hn − xn
= lim 2!
h→0 h
n−2 2
n (n − 1) x h
nxn−1 h + + · · · + hn
= lim 2!
h→0
 h 
n−1 1 n−2 n−1
h nx + n (n − 1) x h + · · · + h
2
= lim
h→0 h
n−2
 
n−1 n (n − 1) x h n−1
= lim nx + + ··· + h
h→0 2
n (n − 1) xn−2
= nx n−1
+ (0) + · · · + (0)n−1
2
= nxn−1

as claimed.

Example 4.22. Let us use the above theorem to differentiate the following functions:

(a) f (x) = x3 (b) f (x) = x.

Solution.

(a)

f (x) = x3 ⇒ n = 3.
∴ f 0 (x) = nxn−1 = 3x3−1
= 3x2 .

104 MATH 122: Calculus I - Kojoga Aziedu



(b) f (x) = x = x1/2 ⇒ n = 21 .

1 1 −1 1 − 1
∴ f 0 (x) = x2 = x 2
2 2
1 1 1 1
= × 1 = ×√
2 x2 2 x
1
= √
2 x
as before.

4.2 Algebraic Rules of Differentiation.


Theorem 4.23. Let f (x) and g (x) be differentiable functions at the point x. Let also k ∈ R. Then

1. Derivative of a Constant: If f (x) = k is a constant function, then


df (x) d
= (k) = 0. (4.9)
dx dx

2. Sum Rule:
d df (x) dg (x)
[f (x) ± g (x)] = ± . (4.10)
dx dx dx
3. Product Rule:
d df (x) dg (x)
[f (x) · g (x)] = g (x) + f (x) . (4.11)
dx dx dx
And if k is a constant, then
d df (x)
[kf (x)] = k . (4.12)
dx dx

4. Quotient Rule:
  g (x) df (x) − f (x) dg (x)
d f (x) dx dx .
= (4.13)
dx g (x) [g (x)]2

Proof.

1. Suppose f (x) = k, a constant. Then f (x + h) = k (see Figure ?? below.)

Figure 4.3: Finding the derivative of a constant function.

105 MATH 122: Calculus I - Kojoga Aziedu


Thus
df f (x + h) − f (x) k−k
= lim = lim
dx h→0 h h→0 h
= lim (0) = 0.
h→0

Therefore,
d
(k) = 0
dx
for all constants k.

2. Let H (x) = f (x) + g (x). Then


dH H (x + h) − H (x)
= lim
dx h→0 h
f (x + h) + g (x + h) − (f (x) + g (x))
= lim
h→0 h
f (x + h) − f (x) + g (x + h) − g (x)
= lim
h→0
 h 
f (x + h) − f (x) g (x + h) − g (x)
= lim +
h→0 h h
f (x + h) − f (x) g (x + h) − g (x) df dg
= lim + lim = + .
h→0 h h→0 h dx dx
Similarly, if we let G (x) = f (x) − g (x), then
dG f (x + h) − g (x + h) − (f (x) − g (x))
= lim
dx h→0 h
f (x + h) − f (x) − g (x + h) + g (x)
= lim
h→0
 h 
f (x + h) − f (x) g (x + h) − g (x)
= lim −
h→0 h h
f (x + h) − f (x) g (x + h) − g (x) df dg
= lim − lim = − .
h→0 h h→0 h dx dx
Therefore,
d df dg
(f ± g) = ±
dx dx dx
3. Let H (x) = f (x) · g (x). Then
dH H (x + h) − H (x)
= lim
dx h→0 h
f (x + h) · g (x + h) − f (x) · g (x)
= lim .
h→0 h
Add 0 in the form 0 = −f (x) g (x + h) + f (x) g (x + h) to the RHS to get
dH f (x + h) · g (x + h) − f (x) g (x + h) + f (x) g (x + h) − f (x) · g (x)
= lim .
dx h→0 h
g (x + h) (f (x + h) − f (x)) + f (x) (g (x + h) − g (x))
= lim
h→0
 h 
g (x + h) (f (x + h) − f (x)) f (x) (g (x + h) − g (x))
= lim +
h→0 h h
g (x + h) (f (x + h) − f (x)) f (x) (g (x + h) − g (x))
= lim + lim
h→0 h h→0 h
f (x + h) − f (x) (g (x + h) − g (x))
= lim g (x + h) · lim + f (x) lim
h→0 h→0 h h→0 h
df dg
= g (x) + f (x) .
dx dx
106 MATH 122: Calculus I - Kojoga Aziedu
Therefore,
d df dg
(f (x) · g (x)) = g (x) + f (x) .
dx dx dx

Now, suppose g (x) = k, a constant. Then

d df (x) d
[kf (x)] = k + f (x) (k)
dx dx dx
df (x) df (x)
= k + f (x) (0) = k .
dx dx

4. Method I.
Suppose g (x) 6= 0. Then by definition

f (x + h) f (x)
  −
d f (x) g (x + h) g (x)
= lim
dx g (x) h→0 h
 
1 f (x + h) g (x) − f (x) g (x + h)
= lim · .
h→0 h g (x + h) g (x)

Now, let us add the term 0 ≡ −f (x) g (x) + f (x) g (x) to the numerator, to get

   
d f (x) 1 f (x + h) g (x) − f (x) g (x) + f (x) g (x) − f (x) g (x + h)
= lim ·
dx g (x) h→0 h g (x + h) g (x)
  
1 f (x + h) g (x) − f (x) g (x) f (x) g (x + h) − f (x) g (x)
= lim −
h→0 h g (x + h) g (x) g (x + h) g (x)
  
1 g (x) (f (x + h) − f (x)) f (x) (g (x + h) − g (x))
= lim −
h→0 h g (x + h) g (x) g (x + h) g (x)
   
1 (f (x + h) − f (x)) f (x) (g (x + h) − g (x))
= lim · − lim ·
h→0 g (x + h) h h→0 g (x + h) g (x) h
1 (f (x + h) − f (x)) f (x) (g (x + h) − g (x))
= lim · lim − lim · lim
h→0 g (x + h) h→0 h h→0 g (x + h) g (x) h→0 h
1 df f (x) dg
= · − ·
g (x) dx [g (x)]2 dx
df dg
g (x) − f (x)
= dx dx .
[g (x)]2

Method II. In Jots


.
.
.

107 MATH 122: Calculus I - Kojoga Aziedu


Corollary 4.24. Derivative of the Reciprocal of a Function.
1
If f is a differentiable function at x and f (x) 6= 0, then is also differentiable at x, and
f

f 0 (x)
 
d 1
=− . (4.14)
dx f (x) [f (x)]2

Proof. By definition,
1 1
  −
d 1 f (x + h) f (x)
= lim
dx f (x) h→0 h
f (x) − f (x + h)
= lim
h→0 hf (x + h) f (x)

−1 f (x + h) − f (x)
= lim · lim
h→0 f (x + h) f (x) h→0 h
0
f (x)
= − .
[f (x)]2

ALT: Use Quotient Rule.

Example 4.25. Let us use the above theorem to differentiate the following functions:

(a) f (x) = 2x5 − 4x4 + 3x2 + 1

(b) f (x) = (x2 − 4x + 1) (x6 + x4 − 3x2 )


4x − 7
(c) f (x) =
3 − x2

Solution.

(a) f (x) = 2x5 − 4x4 + 3x2 + 1.

d d d d
f 0 (x) = 2x5 − 4x4 + 3x2 +
  
(1)
dx  dx  dx dx
= 2 5x4 − 4 4x3 + 3 (2x)
= 10x4 − 16x3 + 6x.

(b) f (x) = (x2 − 4x + 1) (x6 + x4 − 3x2 ).


 d  d
f 0 (x) = x2 − 4x + 1 x6 + x4 − 3x2 + x6 + x4 − 3x2 x2 − 4x + 1
 
 dx  dx
x2 − 4x + 1 6x5 + 4x3 − 6x + x6 + x4 − 3x2 (2x − 4)

=

108 MATH 122: Calculus I - Kojoga Aziedu


4x − 7
(c) f (x) = .
3 − x2
d d
(3 − x2 ) (4x − 7) − (4x − 7) (3 − x2 )
0
f (x) = dx dx
(3 − x2 )2
4 (3 − x2 ) + 2x (4x − 7) 12 − 4x2 + 8x2 − 14x
= =
(3 − x2 )2 (3 − x2 )2
4x2 − 14x + 12 2 (2x − 3) (x − 2)
= 2 = .
(3 − x )2 (3 − x2 )2

Theorem 4.26. Derivative of a Power Function for Negative Exponents.


Let n ∈ N− . Then
d n
(x ) = nxn−1
dx

Proof. Case II: n ∈ N− .


Suppose n ∈ N− . Then n = −m for some m ∈ N+ . Therefore,
1
f (x) = xn = x−m = .
xm
Hence, applying the quotient rule, we have
 
0 d 1
f (x) =
dx xm
d d m
xm (1) − (1) (x ) 0 − mxm−1
= dx dx =
(xm )2 x2m
= −mxm−1−2m = −mx−m−1
= nxn−1 .

Example 4.27. Differentiate the following with respect to x:


1 1
(a) f (x) = (b) f (x) = √ .
x2 x

Solution.

1 2
(a) f (x) = 2
= x−2 ⇒ f 0 (x) = −2x−2−1 = −2x−3 = − 3 .
x x

1 1 1 3 1 1
(b) f (x) = √ = x− 2 ⇒ f 0 (x) = − 12 x− 2 −1 = − 21 x− 2 = − 3 = − √ .
x 2x 2 2 x3

109 MATH 122: Calculus I - Kojoga Aziedu


4.3 Differentiation of Composite Functions: The Chain Rule.
Suppose y = f (u) and u = g (x). Then y is the composite function
y = (f ◦ g) (x) = f (g (x)) .
The rule for differentiating composite functions is called the Chain Rule.

Theorem 4.28. The Chain Rule.


Let y = f (u) and u = g (x) differentiable functions of u and x respectively. Then y = f (g (x)) is also
a differentiable function of x and
d df (u) du (x)
[f (g (x))] = ·
dx du dx
or simply
dy dy du
= · .
dx du dx
Also, if y = f (u), u = g (v) and v = h (x), and f , g and h are differentiable functions of u, v and x
respectively, then
dy dy du dv
= · · .
dx du dv dx
In general,

Proof. On phone

Figure 4.4:

Example 4.29. Find the derivative of the following functions with respect to x:

110 MATH 122: Calculus I - Kojoga Aziedu


(a) f (x) = (x2 + 3x + 2)
7
(x − 1)2
(c) f (x) =
r (x + 3)3
2x + 3
(b) f (x) = 4
(d) f (x) = (2x2 + 4) (x2 + 2x + 3)
2
x+2

Solution.

(a) Let u = x2 + 3x + 2. Then f (x) = [u (x)]7 = g (u (x)) = (g ◦ u) (x) .

∴ f 0 (x) = f 0 (u) · u0 (x)


= 7u6 · (2x + 3)
6
= 7 x2 + 3x + 2 (2x + 3) .

2x + 3 p 
(b) Let u = so that f (x) = f u (x) . Now,
x+2
df d √  1
= u = √
du dx 2 u

and
d d
du (x + 2) (2x + 3) − (2x + 3) (x + 2)
= dx dx
dx (x + 2)2
2 (x + 2) − (2x + 3) 1
= 2 =− .
(x + 2) (x + 2)2

Alternatively, since
2x + 3 2 (x + 2) − 1 1
u= = =2− ,
x+2 x+2 x+2
we have that
 
du d 1
= 2−
dx dx x+2
1
= − ,
(x + 2)2

by Corollary 4.24. Hence,


df df du
= ·
dx du dx 
1 1 1 1
= √ · − 2 =− r ·
2 u (x + 2) 2x + 3 (x + 2)2
2
x+2
r √
1 x+2 1 1 x+2 1
= − · 2 = −
√ ·q
2 2x + 3 (x + 2) 2 2x + 3
(x + 2)4
1 1 1 1 1 1
= − √ ·q =− √ · p
2 2x + 3 2 2x + 3 (x + 2) (x + 2)
(x + 2)3
1
= − p .
2 (x + 2) (2x + 3) (x + 2)

111 MATH 122: Calculus I - Kojoga Aziedu


(x − 1)2 u2
(c) For f (x) = , let u = x − 1 and v = x + 3 so that f (x) = and
(x + 3)3 v3

du2 2 dv
3
3 du
2
du 2 dv
3
dv
df v3 − u v · − u ·
= dx dx = du dx dv dx
2 6
dx 3
(v ) v
du dv
2uv 3 − 3u2 v 2
= dx dx .
v6
using the quotient and the chain rules. But
du dv
=1= .
dx dx

df 2uv 3 (1) − 3u2 v 2 (1) 2 (x − 1) (x + 3)3 − 3 (x − 1)2 (x + 3)2


∴ = =
dx v6 (x + 3)6
(x − 1) (x + 3)2 [2 (x + 3) − 3 (x − 1)]
=
(x + 3)6
(x − 1) (x + 3)2 [−x + 9] (x − 1) (x − 9)
= 6 =− .
(x + 3) (x + 3)4

du dv
(d) Let u = 2x2 + 4 ⇒ = 4x and v = x2 + 2x + 3 ⇒ = 2x + 2. Then
dx dx
4 2
f (x) = 2x2 + 4 x2 + 2x + 3 = u4 v 2 .

Therefore,
d d d 2  dv d  du
f 0 (x) = u4 v2 + v2 u4 = u4 + v2 u4
 
v
dx dx dv dx du dx
dv du
= 2u4 v + 4v 2 u3
dx dx
4 2 2 3
= 2 2x + 4 x + 2x + 3 (2x + 2) + 4 x2 + 2x + 3 2x2 + 4 (4x)
2

3
= 4 x2 + 4 x2 + 2x + 3 (x + 1) 2x2 + 4 + 4x x2 + 2x + 3 2x2 + 4 .
   

4.4 Derivative of Some Standard Functions.

4.4.1 Derivative Exponential Functions.


4.4.1.1 The Exponential Function and its Derivative.

Let a > 0. Then the exponential function f of x to the base a is given by

f (x) = ax .

112 MATH 122: Calculus I - Kojoga Aziedu


Figure 4.5: Graph of f (x) = ax

So, given f (x) = ax ,

f (x + h) − f (x) ax+h − ax ax · ah − ax
f 0 (x) = lim = lim = lim
h→0 h  h→0 h h→0 h
x h h
a a −1 a −1
= lim = ax lim .
h→0 h h→0 h

Now, notice that f (x) = ax , we have


f (0 + h) − f (0) f (h) − f (0)
f 0 (0) = lim = lim
h→0 h h→0 h
h 0 h
a −a a −1
lim = lim .
h→0 h h→0 h

Thus
f 0 (x) = ax f 0 (0) ;
ah − 1
implying that f 0 (0) is a constant. That is lim is finite. So
h→0 h
f (x) = ax ⇒ f 0 (x) = Kax

ah − 1
where K = lim is a constant. That is
h→0 h
f 0 (x) = Kf (x) .

Definition 4.30. The Number e.


The value of a for which K = 1 is denoted e, and is

e = 2.718281828459045235360287471352662497757247 · · ·
≈ 2.718,

an irrational number. Thus


eh − 1
lim = 1. (4.15)
h→0 h

113 MATH 122: Calculus I - Kojoga Aziedu


Hence, with
ah − 1
lim = 1,
h→0 h
we have that for f (x) = ax ,
f 0 (x) = f (x)
or
d x
(e ) = ex . (4.16)
dx
The function
f (x) := exp (x) = ex
is called the (natural) exponential function.

Deductions.
• If f is a differentiable function of x and y = ef (x) , then
dy deu deu du
= = ·
dx dx du dx
du
= eu · = f 0 (x) ef (x)
dx
where u = f (x) .
• Thus if f (x) = k, a constant, then
d kx
e = kekx .
dx
Example 4.31. Differentiate the following with respect to x.

(a) e2x (b) e2x x2 −1

Solution. Using the chain and product rules, we have

(a)
d  d
e2x (2x) = 2e2x .
d (2x) dx
(b)
Using the chain rule here

d  2x√x2 −1  d  √ 2
z }| √ {
2
e = 2x x − 1 e2x x −1
dx dx
Using the product rule here
 
z }| {
d √ d√ 2  2x√x2 −1
= 2 (x) · x 2−1+x x − 1 e
 dx dx 

 
Using the chain rule here
z  }| {
√ 1 1
 √
− 2
x2 − 1 2  e2x x −1

= 2  x2 − 1 + x (2x)
 
 2 

√ x2
 2
x − 1 + x2 2x√x2 −1
  √ 
2x x2 −1
= 2 −1+ √
x2 e =2 √ e
x2 − 1 x2 − 1
 2  √
2x − 1 2
= 2 √ e2x x −1 .
x2 − 1

114 MATH 122: Calculus I - Kojoga Aziedu


4.4.2 Derivatives of Trigonometric Functions.
Theorem 4.32. Let x be an angle measured in radians. Then the derivative of the six trigonometric
ratios are as follows:
d d d
(a) (sin x) = cos x (c) (tan x) = sec2 x (e) (cot x) = − csc2 x
dx dx dx
d d d
(b) (cos x) = − sin x (d) (sec x) = sec x tan x (f ) (csc x) = − csc x cot x.
dx dx dx

Proof. (a) Let f (x) = sin x so that f (x + h) = sin (x + h). Hence, from first principles, we have
d sin (x + h) − sin x
(sin x) = lim . (4.17)
dx h→0 h
Method I. Using the trigonometric identity
sin (A + B) = sin A cos B + cos A sin B,
we have
d sin (x + h) − sin x
(sin x) = lim
dx h→0 h
sin x cos x + sin h cos x − sin x
= lim
h→0 h
sin x (cos h − 1) + sin h cos x
= lim
h→0 h
sin x (cos h − 1) sin h cos x
= lim + lim
h→0 h h→0 h
(cos h − 1) sin h
= sin x lim + cos x lim .
h→0 h h→0 h

sin x cos x − 1
Now, recall that lim = 1 and lim = 0.
x→0 x x→0 x
d
∴ (sin x) = sin x · 0 + cos x · 1
dx
= cos x.

Method II. Recall the trignogometric identity


sin (A + B) − sin (A − B) = 2 cos A sin B.
2x + h h h
Set A + B = x + h and A − B = x so that A = = x + and B = . Then (4.17) becomes
2 2 2
2 cos x + h2 sin h2
 
d d sin (x + h) − sin x
(sin x) = (sin x) lim = lim
dx dx h→0 h h→0 h
h h
sin h2
    
cos x + 2 sin 2 h
= lim h
= lim cos x + · lim h
h→0
2
h→0 2 h→0
2
cos is continuous
z}| {
sin h2

h
= cos lim x + · lim h
= cos (x + 0) · 1 = cos x.
h→0 2 h→0
2

115 MATH 122: Calculus I - Kojoga Aziedu


(b) Similarly, let f (x) = cos x. Then

d cos (x + h) − cos x
(cos x) = lim . (4.18)
dx h→0 h
Method I. From the trigonometric identity

cos (A + B) = cos A cos B − sin A sin B,

d cos (x + h) − cos x
(cos x) = lim
dx h→0 h
cos x cos h − sin x sin h − cos x
= lim
h→0 h
cos x (cos h − 1) sin x sin h
= lim − lim
h→0 h h→0 h
(cos h − 1) sin h
= cos x lim − sin x lim
h→0 h h→0 h
= cos x · 0 − sin x · 1
= − sin x.

Method II.

(c) Method I. From (a) and (b) above, and applying the quotient rule, we have

  cos x d (sin x) − sin x d (cos x)


d d sin x dx dx
(tan x) = =
dx dx cos x cos2 x
cos x (cos x) − sin x (− sin x) cos2 x + sin2 x
= =
cos2 x cos2 x
1
= = sec2 x.
cos2 x

Method II. From first principles

(d) Method I.
 
d d 1 cos x · 0 − 1 · (− sin x)
(sec x) = =
dx dx cos x cos2 x
sin x 1 sin x
= 2
= ·
cos x cos x cos x
= sec x tan x.

Method II. From first principles

116 MATH 122: Calculus I - Kojoga Aziedu


(e) Method I.

tan x · 0 − 1 · (sec2 x)
 
d d 1
(cot x) = =
dx dx tan x tan2 x
− sec2 x 1 1 cos2 x 1
= 2
= − 2
· = − 2 ·
tan x tan x cos2 x sin x cos2 x
1
= − 2 = − csc2 x.
sin x

Method II.
d d
d d  cos x  sin x dx (cos x) − cos x dx (sin x)
(cot x) = =
dx dx sin x sin2 x
sin x (− sin x) − cos x (cos x) sin2 x + cos2 x
= =−
sin2 x sin2 x
1
= − 2 = − csc2 x
sin x
as before.

Method III. From first principles

(f) Method I.
 
d d 1 sin x · 0 − 1 · (cos x)
(csc x) = =
dx dx sin x sin2 x
− cos x 1 cos x
= 2 =− · = − csc x cot x.
sin x sin x sin x

Method II. From first principles

Again, note that x is in radians!

Example 4.33. Differentiate the following with respect to x:

(a) sin 2x (d) sin x◦


(b) cos2 5x
p √
(c) x2 sin 3x (e) sin x2 + 1.

Solution.

117 MATH 122: Calculus I - Kojoga Aziedu


du
(a) Let u = 2x ⇒ = 2.
dx
d d du
∴ (sin 2x) = (sin u)
dx du dx
= cos u · 2 = 2 cos 2x.

(b) Let y = cos2 5x and u = 5x. Then y = cos2 u and


d d  du
cos2 5x = cos2 u ·

dx du dx
= −2 cos u sin u · 5.
= −10 cos 5x sin 5x.

(c)
d d d
x2 sin 3x = x2 (sin 3x) + sin 3x x2
 
dx dx dx
2
= 3x cos 3x + 2x sin 3x
= x (3x cos 3x + 2 sin 3x) .

(d) Recall that

180◦ = π rad ⇒ 1◦ = π/180 rad.

Therefore,
d ◦ d
  π 
(sin x ) sin x .
dx dx 180
π π
Let u = x ⇒ u0 (x) = .
180 180
d d π
∴ (sin x◦ ) = (sin u (x)) = .
dx dx 180
d du π
= (sin u) · = cos u ·
du dx 180
π π ◦
= cos u = cos x .
180 180

p √ √
(e) Let y = sin x2 + 1, v = x2 + 1 and u = sin v. Then

y = sin u.

Hence,
dy dy du dv
= · ·
dx du dv dx
1 1 1 − 1
= (cos u)− 2 · cos v · (2x) x2 + 1 2
2 √ 2
x cos x + 12
= q √ .
2
2 (x + 1) cos sin x + 1 2

118 MATH 122: Calculus I - Kojoga Aziedu


4.5 Derivatives of Inverse Functions.
Here, we seek differentiate the various inverse functions we learnt in ..........

4.5.1 Differentiating General Inverse Functions.


Theorem 4.34. Differentiating an Inverse Function
Let f be a differentiable function at every point of an interval I and that f 0 (x) 6= 0 at any point of I.
Then f −1 is differentiable on f (I) and for any a ∈ I,

df −1

1
=
dx x=a df
dy −1 y=f (a)

or
0 1
f −1 (a) = . (4.19)
f0 (f −1 (a))
In short,
0 1
f −1 = . (4.20)
f0 (f −1 )

Proof.
Method I.
Suppose y = f −1 (x),. Then f (y) = x. Set so that b = f −1 (a) f (b) = a . Hence, by definition,

df −1 f −1 (x) − f −1 (a)

= lim .
dx x=a x→a x−a

Now, since f is differentiable on I, it is also continuous, so f −1 is continuous. Therefore, as x → a so


does f −1 (x) = y → f −1 (a) = b. Hence,

df −1

y−b 1
= lim =
dx x=a y→b f (y) − f (b) limy→b f (y)−f (b)

y−b
1 1
= = .
df df
dy dy
y=b y=f −1 (a)

Method II.
Let f −1 be the inverse of f on I. Then f (f −1 (x)) = f −1 (f (x)) = x ∀x ∈ I. By the chain rule, we
have
 d d 1
f 0 f −1 (x) · f −1 (x) = 1 ⇒ f −1 (x) = 0 −1
 
.
dx dx f (f (x))

119 MATH 122: Calculus I - Kojoga Aziedu


4.5.1.1 Steps to Follow When Finding the Derivative of the Inverse of a Function.

• Establish the existence of the inverse f −1 of f .

• Compute f 0 (x)

• To compute f −1 (a), suppose f −1 (a) = x ∈ Df so that f (x) = a.

 Solve this equation for x.


 Thus f −1 (a) = x.

• Use the formula


0 1
f −1 (a) =
f0 (f −1 (a))
to compute the required derivative at x = a.

0
Example 4.35. Find (f −1 ) (1) if f (x) = x3 − x2 + 1, x > 32 .

Solution. First of all, we have to establish the existence or not of f . Now,

f (x) = x3 − x2 + 1 ⇒ f (x) = 3x2 − 2x


 
2 2
= 3x x − > 0 ∀x > .
3 3

That is, f is increasing on 23 , ∞ and hence is 1 − 1. Therefore, f −1 exists.




Now, by definition,
−1
0 1
f (x) =
x=a f0 (f −1 (a))
and so,
0 1
f −1 (x) = .
x=1 f0 (f −1 (1))
Let f −1 (1) = x ⇒ f (x) = 1. To get the value of f −1 (1), we solve for x in the equation f (x) = 1, that
is,

x3 − x2 + 1 = 1
⇒ x2 (x − 1) = 0
⇒ x = 0 or x = 1.

But 0 ∈ 2
, so we choose x = 1 so that f −1 (1) = 1. Therefore,

/ 3
,∞
0 1
f −1 (1) =
f0 (f −1 (1))
1
=
f0
(x)|x=1
1
= 2
3x − 2x|x=1
1
= = 1.
3−2

120 MATH 122: Calculus I - Kojoga Aziedu


√ 0
Example 4.36. Show that f (x) = x 3 + x2 is invertible when x > 0 and hence find (f −1 ) (2).
Solution.
√ − 1 √ x2
f 0 (x) = 3 + x2 + x2 3 + x2 2 = 3 + x2 + √
3 + x2
√ x2
 
= 3 + x2 1 +
3 + x2

3 + x2 2

= 2x + 3 .
3 + x2

Since x2 > 0 for all x ∈ R, we have 3 + x2 > 0 and 2x2 +3 > 0 for all x ∈ R, and so f 0 (x) > 0 ∀x > 0.
Hence, f strictly increases for all real numbers and so is 1 − 1, and is thus invertible. Hence,

0 1
f −1 (2) = .
f0 (f −1 (2))
Now, let f −1 (2) = x so that f (x) = 2. That is,

x 3 + x2 = 2.

Squaring both sides, we have

x2 x2 + 3

= 4
⇒ x4 + 3x2 − 4 = 0
⇒ (x − 1) x3 + x2 + 4x + 4

= 0
⇒ (x − 1) (x + 1) x2 + 4

= 0,

so that x = 1 for x > 0, i.e. f −1 (2) = 1. Therefore,


0 1 1
f −1 (2) = = h√ i
f 0 (f −1 (2)) 3 + x2 + √x
2

3+x2 x=1
1 1 4
= √ = 1 = .
3+1+ √1 2+ 9
3+1 4

ˆ g(x)
Example 4.37. Let f (x) = f (u) du. Then f 0 (x) is given by
a

f 0 (x) = g 0 (x) f (x) ∀a ∈ R.

Show that the function f defined by


ˆ x √
f (x) = 1 + t2 dt
2
0
is one-to-one for x > 0 and find (f −1 ) (2).

Solution. By the definition above,



f 0 (x) = 1 + x2 > 0 ∀x > 0.

Thus f is strictly increasing (see ..........) on (0, ∞) and so is one-to-one. Therefore, f has an inverse.
Hence
0 1
f −1 (2) = 0 −1
.
f (f (2))

121 MATH 122: Calculus I - Kojoga Aziedu


Now, let us find f −1 (2). Notice that if f −1 (2) = x, then f (x) = 2, i.e.
ˆ x√
f (x) = 1 + t2 dt = 2.
2
or
ˆ x √ √ ˆ 2
1+ t2 dt
dt ⇒ 1 + x2 = 2
=
2 2

⇒ x = ± 3.
√ √
For x > 0, we have x = 3, i.e. f −1 (2) = 3.
0 1 1
∴ f −1 (2) = √ = √
f 0 3 1 + x2 x=√3
1 1
= √ = .
1+3 2
Example 4.38. Suppose the function f defined by
4x3
f (x) =
x2 + 1
0
has an inverse. Find (f −1 ) (2).
Solution. Given that f is invertible, we have
0 1
f −1 (2) = .
f0 (f −1 (2))
Let f −1 (2) = x ⇒ f (x) = 2. Then
4x3
= 2 ⇒ 4x3 = 2 x2 + 1

f (x) = 2
x +1
or
 2
3 2 21 
0 = 4x − 2x − 2 = (x − 1) 4x + 2x + 2 = 4 (x − 1) x + .
2
That is, x = 1 for x > 0 and so f −1 (2) = 1. Thus
0 1 1 4 1
f −1 (2) = = 2 2 = = .
f 0 (1) 2
12(1) (1 +1)−8(1)
2
12 (2) − 8 4
(12 +1)

Example 4.39. Let f (x) = x3 + 2ex . Show that f is one-to-one and confirm that f (0) = 2. Hence,
0
find (f −1 ) (2).
Solution.

4.5.2 Derivatives of Inverse Trigonometric Fuctions.


4.5.2.1 Derivative of the Inverse Sine Function.
h π πi
Let y = sin−1 x so that x = sin y, y ∈ − , . Differentiating both sides with respect to x, we have
2 2
dx d dy
= (sin y) ⇒ 1 = cos y ,
dx dx dx
or
dy 1 1 1
= =p =√ .
dx cos y 2
1 − sin y 1 − x2
Therefore,
d 1
sin−1 x = √ (4.21)

.
dx 1 − x2

122 MATH 122: Calculus I - Kojoga Aziedu


4.5.2.2 Derivative of the Inverse Cosine Function.

Let y = cos−1 x so that x = cos y, y ∈ [0, π]. Differentiating both sides with respect to x, we have

dx d dy
= (cos y) ⇒ 1 = − sin y ,
dx dx dx
or
dy 1 1 1
=− = −p = −√ .
dx cos y 1 − cos2 y 1 − x2
Therefore,
d 1
cos−1 x = − √ (4.22)

.
dx 1 − x2

4.5.2.3 Derivative of the Inverse Tangent Function.

Let y = tan−1 x ⇒ x = tan y, y ∈ R. Then


dx d dy
= (tan y) ⇒ 1 = sec2 y .
dx dx dx
dy 1 1 1
∴ = 2
= 2
= .
dx sec y 1 + tan y 1 + x2
Hence,
d 1
tan−1 x = (4.23)

.
dx 1 + x2

4.5.2.4 Derivative of the Inverse Secant Function.

Method I.
Let y = sec−1 x for |x| > 1. Then

sec y = x
dy
⇒ sec y tan y = 1
dx
dy 1
⇒ =
dx sec y tan y

Since, |x| > 1, y ∈ 0, π2 ∪ π


and so sec y tan y 6= 0. Therefore,
 
2

dy 1
=
dx sec y tan y
1
= √ .
±x x2 − 1

Alternative Analysis:
Observe from the graph of y = sec−1 x that its slope is always positive. Hence,
(
d −1
 x
√1
x2 −1
, if x > 1
sec x =
dx √1
−x x2 −1
, if x < −1
1
= √ if |x| > 1.
|x| x2 − 1

123 MATH 122: Calculus I - Kojoga Aziedu


Method II.
We can also define the sec−1 as
 
−1 −1 1
sec x = cos , |x| ≥ 1.
x

Therefore,
  
d d 1
sec−1 x = cos −1
dx dx x
 
1 1
= −q − 2
1 − x21 x
r
1 x2 1 ±x
= 2 = √
x x2 − 1 x2 x2 − 1
1 |x| 1
= 2√ = √ .
x x2 − 1 |x| x2 − 1

4.5.2.5 Derivative of the Inverse Cosecant Function.

Let y = csc−1 x for |x| > 1. Then

d 1
csc−1 x = − √
dx x x2 − 1

4.5.2.6 Derivative of the Inverse Cotangent Function.

Let y = cot−1 x. Then

cot y = x
dy 1
⇒ = − 2
dx csc y
1
= − 2
x +1
d 1
∴ cot−1 x = − 2 , x ∈ R. (4.24)
dx x +1

Summary of the Derivatives of the Inverse Trigonometric Functions.


d −1 1 h π πi d 1
tan−1 x =
 
• sin x = √ , x∈ − , • , x∈R
dx 1−x 2 2 2 dx 1 + x2

d 1 h π πi d 1
−1 sec−1 x = √ , |x| ≥ 1

• cos x = − √ ,, x ∈ − , •
dx 1 − x2 2 2 dx |x| x2 − 1

124 MATH 122: Calculus I - Kojoga Aziedu


d 1 d 1
• csc−1 x = − √ , |x| ≥ 1 • cot−1 x = − 2 , x ∈ R.
dx |x| x2 − 1 dx x +1

Example 4.40. Differentiate the following functions with respect to x.



(a) tan−1 x3 (d) x2 sin−1 x 3
   
x − 1 − x2
(g) −1
tan √
1 + 1 + x2
(e)
 
1 sec−1 x + csc−1 x
(b) sin−1 s


x2  √ 2
1 + x + 1
(h) cos−1 

2x 1 − x2 √
(c) ln cos−1 x
 (f) tan −1
2 2 1 + x2
1 − 2x

Solution.

(a)

(b)

(c)

(d)

(e)

(f)

(g)

(h)

Example 4.41. m

Solution.

 
x−1 π
Example 4.42. Prove that tan −1
= tan−1 x − for x > −1.
x+1 4
 
x−1
Solution. Let f (x) = tan −1
− tan−1 x. Then for x > −1, we have
x+1
 
0 1 d x−1 1
f (x) =  2 − 2
x−1 dx x + 1 x +1
+1
x+1
 
1 x − 1 − (x + 1) 1
=  2 2 − 2
x−1 (x + 1) x +1
+1
x+1
2 1
= 2
− 2 = 0.
2x + 2 x + 1

125 MATH 122: Calculus I - Kojoga Aziedu


Hence, f (x) = C, a constant on (−1, ∞). When x = 0 ∈ (−1, ∞), we have
π
C = f (0) = tan−1 (−1) − tan−1 (0) = − .
4

Note: The choice of x = 0 ∈ (−1, ∞) is arbitrary. Any choice of x in the given interval will work. For
example, when we choose x = 1, we have
π
C = f (0) = tan−1 (0) − tan−1 (1) = − .
4

Example 4.43. Show that


 √ 
d −1 x−1 1
tan √ = √ .
dx 1+ x 2 x (x + 1)

Solution.
 √  √ 
d −1 x−1 1 d x−1
tan √ =  √ 2 · √
dx 1+ x x−1 dx 1+ x
1+ √
1+ x
" √
d √ √ d √
#
1 ( x + 1) dx ( x − 1) − ( x − 1) dx ( x + 1)
= √ 2 √ 2 · √ 2
( x+1) +( x−1) ( x + 1)
√ 2
( x+1)
√ " √ √
( x + 1) 2√1 x − ( x − 1) 2√1 x
2
#
( x + 1)
= √ 2 √ 2 · √ 2
( x + 1) + ( x − 1) ( x + 1)
1 √ √
 
1 
= √ √ · √ x+1− x+1
x+2 x+1+x−2 x+1 2 x
 
1 1 1
= √ = √ .
x 2x + 2 2 x (x + 1)

4.5.3 Derivative of the Logarithmic Function.


Definition 4.44. Natural or Napieran Logarithm.

Suppose y = ex . Then the logarithm to base e is x, i.e. loge y = x. The logarithm of a number x to
the base e is called the natural (or Napierian) logarithm of x, and is denoted by ln x, i.e.

ln x = loge x.

Thus the function ln is the inverse of the function exp, and so

exp (ln x) ≡ eln x = x.

Therefore, given
y = ex ⇒ ln y = x.
In general,
y = ax ⇒ loga y = x
and is called the logarithm of y to the base a > 0.

126 MATH 122: Calculus I - Kojoga Aziedu


Figure 4.6: Graph of f (x) = ax

Theorem 4.45. Derivative of the Logarithmic Function.


Given that y = ln x, then
dy d 1
= (ln x) = . (4.25)
dx dx x
Proof. Given that y = ln x, then x = ey . Differentiating both sides with respect to x, we have
d d y dey dy
(x) = (e ) = ·
dx dx dy dx
dy
⇒ 1 = ey .
dx
Therefore,
dy 1 1
= y = .
dx e x

Corollary 4.46. If f is a differentiable function of x, then


d f 0 (x)
[ln (f (x))] = . (4.26)
dx f (x)

Proof. Let y = ln (f (x)) and u = f (x) so that y = ln u. Then from the Chain Rule, we have
dy dy du
= ·
dx du dx
d d
= (ln u) · (f (x))
du dx
1 0 f 0 (x)
= · f (x) = .
u f (x)

127 MATH 122: Calculus I - Kojoga Aziedu


Therefore,
d f 0 (x)
[ln (f (x))] = .
dx f (x)

Theorem 4.47. Derivative of the General Exponential and Logarithmic Functions.

Let a > 0. Then


d x
(a ) = ex ln a ln a (4.27)
dx
= ax ln a (4.28)

and

d 1
(loga x) = . (4.29)
dx x ln a

Proof. Let y = ax for a > 0. Then

ln y = x ln a ⇒ y = ax = ex ln a .

Differentiating both sides with respect to x by letting u = x ln a, we have


d x d x ln a  d u
(a ) = e = (e )
dx dx dx
d u du d
= (e ) · = eu (x ln a)
du dx dx
= eu ln a = ex ln a ln a
x
= eln a ln a = ax ln a.

Also, since
loge x ln x
loga x = = ,
loge a ln a
we have
 
d d ln x 1 d
(loga x) = = (ln x)
dx dx ln a ln a dx
1 1 1
= · = .
ln a x x ln a

Example 4.48. Differentiate the following with respect to x.

(a) 2x (c) ex log10 (sin x2 )


3

(b) eln x (d) ln (x cos 3x)


2

Solution.

(a)
d x
(2 ) = ex ln 2 ln 2
dx
= 2x ln 2.

128 MATH 122: Calculus I - Kojoga Aziedu


(b) Let u = ln x2 . Then

d  ln x2  dy du d u d
ln x2

e = · = (e ) ·
dx du dx du dx
2x 2 2
= eu · 2 = eln x ·
x x
2 ln x2 2 2
= e = · x = 2x.
x x

Alternatively, since eln x = x2 , we have


2

d  ln x2  d
x2 = 2x.

e =
dx dx

(c)

d  x3 2
 2
 d  x3  3 d
+ ex log10 sin x2

e log10 sin x = log10 sin x e
dx dx dx
2
  2 x3  x3 1 2

= log10 sin x 3x e +e 2x cos x
sin x2 ln 10
3  2x cot x2
= 3x2 ex log10 sin x2 + .
ln 10

(d)

d cos 3x − 3x sin 3x
(ln (x cos 3x)) =
dx x cos 3x
1
= − 3 tan 3x.
x

18
Example 4.49. If f (x) = , find that value of
x
f 0 (x) + 18
lim .
x→1 x−1

18 18
Solution. f (x) = ⇒ f 0 (x) = f 0 (x) = − 2 .
x x
f 0 (x) + 18 − x182 + 18 − x12 + 1
lim = lim = 18 lim
x→1 x−1 x→1 x−1 x→1 x − 1
x2 −1  2 
x2 x −1 1
= 18 lim = 18 lim ·
x→1 x − 1 x→1 x2 x−1
   
(x + 1) (x − 1) 1 x+1
= 18 lim · = 18 lim
x→1 x2 x−1 x→1 x2
 
1+1
= 18 = 18 (2) = 36.
12

129 MATH 122: Calculus I - Kojoga Aziedu


4.6 Deivatives of Implicit Functions.
4.6.1 Definition and Examples.
Definition 4.50. Implicit Equations.
Let y be a variable dependent on the variable x and consider an equation of the form f (x, y) = k where
f is a function of x and y and k is a constant. Then variable y, which depends on the variable x, is
said to be defined implicitly as a function of x.
Example 4.51. Examples of implicit functions are

• x2 y + xy 2 = 1 • x3 + y + cos y = 4

• x2 + y 2 + xy + 3 = 0 • xy 3 + ex = y

Note that a function f in the variables x and y is said be explicitly defined if


y = f (x) ;
that is if y can be explicitly made as the subject of the relation connecting x and y.

Theorem 4.52. Let z (y) = y n . If z is a differentiable function of y and y is a differentiable function


of x, then
dz d n dy
= (y ) = ny n−1 .
dx dx dx
Proof. Suppose z and y are differentiable functions of y and x respectively, and that
z (y (x)) = (y (x))n ≡ y n (x) .
Then by the Chain Rule, we have
dz dz dy d n dy
= · = (y ) ·
dx dy dx dy dx
dy
= ny n−1 .
dx

Example 4.53. If f (x) = y 3 (x), then


df df dy d 3  dy
= · = y ·
dx dy dx dy dx
dy dy
= 3y 3−1 = 3y 2 .
dx dx

4.6.2 Procedure for Differentiating Implicit Functions.


dy
Suppose an equation defines y explicitly as a function of x. Then to find ,
dx
• differentiate both of the equation with respect to x;
• because y is a function of x, use the chain rule to differentiate terms containing powers of y;
dy
• solve the differentiated equation for .
dx
dy
Example 4.54. Find if:
dx

130 MATH 122: Calculus I - Kojoga Aziedu


(a) x2 + y 2 = 1 (c) sin x + cos y = x + y

(b) x2 + 2xy + x − 2y 2 = 2 (d) sin (x + y) = y 2 cos x.

Solution.

(a)

d d d dy
x2 + y2 =
 
(1) ⇒ 2x + 2y =0
dx dx dx dx
dy x
∴ = − , y 6= 0.
dx y

(b)

d d d d d
x2 + 2 (xy) + y2
 
(x) − 2 = (2)
dx dx dx dx dx
dy dy
2x + 2y + 2x + 1 − 4y = 0
dx dx
dy
(2x − 4y) = −2x − 2y − 1
dx
dy −2x − 2y − 1 2x + 2y + 1
= = .
dx (2x − 4y) (4y − 2x)

(c)

dy dy
cos x − sin y = 1+
dx dx
dy
(1 + sin y) = cos x − 1
dx
dy cos x − 1
= .
dx sin y + 1

(d)
 
dy dy
cos (x + y) 1 + = 2y cos x − y 2 sin x
dx dx
dy
(cos (x + y) − 2y cos x) = − cos (x + y) − y 2 sin x
dx
dy − cos (x + y) − y 2 sin x
=
dx cos (x + y) − 2y cos x
cos (x + y) + y 2 sin x
= .
2y cos x + cos (x + y)

dy √
Example 4.55. Find if 1 − x6 + 1 − y 6 = a3 (x3 − y 3 ).
p
dx

131 MATH 122: Calculus I - Kojoga Aziedu



Solution. We differentiate implicitly both sides of 1 − x6 + 1 − y 6 = a3 (x3 − y 3 ) with respect to
p

x as follows
d h√ p i d  3 3
a x − y3

1 − x6 + 1 − y 6 =
dx dx
d h 1 1
i d  3 3
1 − x6 2 + 1 − y 6 2 = a x − y3
  
dx dx 
1 5
 6 −2
 1 1 5
 6 −2
 1 dy
3 2 2 dy
−6x 1 − x + −6y 1−y = a 3x − 3y
2 2 dx dx
5 5
3x 3y dy dy
−√ −p = 3a3 x2 − 3a3 y 2
1−x 6 1 − y dx
6 dx

i.e
!
3 2 3y 5 dy 3x5
3a y − p = 3a3 x2 + √
1 − y6 dx 1 − x6
√ p
dy 3a3 x2 1 − x6 + 3x5 3a3 y 2 1 − y 6 − 3y 5
= √ ÷ p
dx 1 − x6 1 − y6
√ p
3a3 x2 1 − x6 + 3x5 1 − y6
= √ × p
1 − x6 3a3 y 2 1 − y 6 + 3y 5
√ p
3a3 x2 1 − x6 + 3x5 1 − y6
= √ × p
1 − x6 3a3 y 2 1 − y 6 + 3y 5
 2√ p !
x 1 − x6 + x5

1 − y6
= √ p
1 − x6 y2 1 − y6 + y5
√ r
x2 1 − x6 + x5 1 − y 6
= p , x, y 6= ±1.
y 2 1 − y 6 + y 5 1 − x6

4.7 Logarithmic Differentiation.


Suppose we want to differentiate functions expressed as products and quotients of many factors, such
as
f (x) g (x)
(f (x))n and .
h (x) k (x)
Then taking logarithms of such expressions reduces the products and quotients to sums and differences.

Definition 4.56. The technique of using logarithm to differentiate functions is called logarithmic
differentiation.

4.7.1 Logarithmic Differentiation of y = (f (x))n and y = (f (x))g(x) .


• y = (f (x))n .
y = (f (x))n ⇒ ln y = n ln f (x) .

132 MATH 122: Calculus I - Kojoga Aziedu


Differentiating both sides with respect to x gives us
y0 f 0 (x)
= n
y f (x)
0 f 0 (x)
⇒ y = ny .
f (x)
d f 0 (x)
∴ ((f (x))n ) = n (f (x))n . (4.30)
dx f (x)

• y = (f (x))g(x) .
Suppose f and g are differentiable functions of x and

y = (f (x))g(x) ,

then
ln y = g (x) ln f (x) .
Differentiating both sides with respect to x, we have
y0 f 0 (x) g (x) f 0 (x)
 
0 0 0
= g (x) ln x + g (x) · ⇒ y = y g (x) ln x + .
y f (x) f (x)
g (x) f 0 (x)
 
d  
∴ (f (x))g(x)
= (f (x)) g(x) 0
g (x) ln x + . (4.31)
dx f (x)

f (x) g (x)
4.7.2 Logarithmic Differentiation of y = .
h (x) k (x)
• Given
f (x) g (x)
y= ⇒ ln y = ln (f (x) g (x)) − ln (h (x) k (x)) = ln f (x) + ln g (x) − ln h (x) − ln k (x) .
h (x) k (x)
When we differentiate both sides with respect to x, we have
y0 f 0 (x) g 0 (x) h0 (x) k 0 (x)
= + − −
y f (x) g (x) h (x) k (x)
 0
f (x) g (x) h (x) k 0 (x)
0 0

0
⇒y = y + − −
f (x) g (x) h (x) k (x)
or
f 0 (x) g 0 (x) h0 (x) k 0 (x)
   
d f (x) g (x) f (x) g (x)
= + − − . (4.32)
dx h (x) k (x) h (x) k (x) f (x) g (x) h (x) k (x)

• In general, if
n
Y
fi (x)
i=1
y = m
Y
gj (x)
j=1

133 MATH 122: Calculus I - Kojoga Aziedu


so that n m
X X
ln y = ln (fi (x)) − ln (gj (x)) ,
i=1 j=1

then
 n  n  n m
X 
gj0
Y Y X
 fi (x)  fi (x)  fi0 (x) (x) 
d  i=1  = i=1  i=1 j=1
(4.33)
   
m m n − m
.
dx  Y  Y X X 
gj (x) gj (x) fi (x) gj (x)
   
j=1 j=1 i=1 j=1

Example 4.57. Find f 0 (x) if


f (x) = xx , x > 0
and
(x + 1) (x + 2) (x + 3)
f (x) = .
x+4
Solution.

• For f (x) = xx , taking the logarithm of both sides, we have

ln f (x) = x ln x.

Differentiating both sides with respect to x, we get


f 0 (x)
 
1
= ln x + x
f (x) x
0
⇒ f (x) = f (x) (ln x + 1)
= xx (ln x + 1)
= xx (ln x + ln e)
= xx ln ex.

(x + 1) (x + 2) (x + 3)
• Also, for f (x) = , we have
x+4
ln |f (x)| = ln |x + 1| + ln |x + 2| + ln |x + 3| − ln |x + 4|
 
0 1 1 1 1
⇒ f (x) = f (x) + + −
x+1 x+2 x+3 x+4
 
(x + 1) (x + 2) (x + 3) 1 1 1 1
= + + −
x+4 x+1 x+2 x+3 x+4
 
1 (x + 1) (x + 2) (x + 3)
≡ (x + 2) (x + 3) + (x + 1) (x + 3) + (x + 1) (x + 2) −
x+4 (x + 4)
 
1 (x + 1) (x + 2)
= (x + 3) (2x + 3) + .
x+4 (x + 4)

Theorem 4.58. Differentiating Power Functions for rational Exponents.


mmm
Look for appropriate place for this ................Acn

Proof. mmm

134 MATH 122: Calculus I - Kojoga Aziedu


4.7.3 Derivatives of Functions in Parametric Forms.
4.7.3.1 Definition and Examples.

Definition 4.59. Parametric Equation of a Curve.


Let P (x, y) denote a point on the curve
y = f (x) , (4.34)
and suppose that the variables x and y are functions of a third variable t, i.e.
x = g (t) , y = h (t) . (4.35)
Then the system of equations (4.35) is called the parametric form of the equation (4.34).
The variable t is called the parameter.

Example 4.60. The parabola y 2 = 4ax has parametric equation x = at2 , y = 2at; and note that
y = 2at ⇒ y 2 = 4a2 t2 = 4a at2 = 4ax.


Figure 4.7: The parabola x = at2 , y = 2at with a = 1.

Also, the ellipse


x2 y 2
+ 2 =1
a2 b
can be parametrised with the parameter θ as
x = a cos θ, y = b sin θ.

Figure 4.8: The ellipse x = a cos θ, y = b sin θ with a = 1, b = 2.

135 MATH 122: Calculus I - Kojoga Aziedu


4.7.3.2 Differentiating Parametric Functions.

Let a parametric curve be defined by

x = f (t) , y = g (t)

for some parameter t at the point P (t) ≡ P (x, y) ≡ P (f (t) , g (t)) where f (t) and g (t) are differentiable
functions of t; i.e.
dx dy
= f 0 (t) , = g 0 (t)
dt dt
dx
exist. If = f 0 (t) 6= 0, then
dt
dy
dy dy dt
= · = dt
dx dt dx dx
dt
g 0 (t)
= .
f 0 (t)

dy
Example 4.61. Find if
dx
(a) x = t3 − t, y = 4 − t2 (c) x = cos 2t, y = tan 2t

(b) x = ln t, y = t − ln t (d) x = cos3 θ, y = 3 sin θ − sin3 θ.

Solution.

(a) x = t3 − t, y = 4 − t2 , we have

dx dy
= ẋ = 3t2 − 1, = ẏ = −2t
dt dt
and so
dy ẏ 2t
= =− 2 , t 6= ±1.
dx ẋ 3 (t − 1)

(b) x = ln t, y = y = t − ln t
1 1 t−1
ẋ = , ẏ = 1 − = .
t t t
t−1
dy ẏ t
∴ = = 1 = t − 1.
dx ẋ t

(c) x = cos 2t, y = tan 2t

ẋ = −2 sin 2t, ẏ = 2 sec2 2t.

dy ẏ 2 sec2 2t sec2 2t
∴ = = =
dx ẋ −2 sin 2t sin 2t
= sec2 2t csc 2t.

136 MATH 122: Calculus I - Kojoga Aziedu


(d) x = cos3 θ, y = 3 sin θ − sin3 θ.

ẋ = −3 cos2 θ sin θ
ẏ = 3 cos θ − 3 sin2 θ cos θ

3 cos θ 1 − sin2 θ

dy ẏ 3 cos θ − 3 sin2 θ cos θ
∴ = = =
dx ẋ −3 cos2 θ sin θ −3 cos2 θ sin θ
2
3 cos θ cos θ cos θ
= = − = − cot θ.
−3 cos2 θ sin θ sin θ

Example 4.62. Prove the following claims.


√ √
(a) Let x = and y = ecos−1 t . Show that
−1
esin t

dy y
=− .
dx x

(b) Let f (t) is a differentiable function of t. Show that


 
0 1 dx dy
f (t) = + − x + y sec t
2 dt dt

if x = f (t) cos t − f 0 (t) sin t and y = f (t) cos t + f 0 (t) sin t, and that
 2  2
dx dy 2
+ = [f (t) + f 00 (t)]
dt dt

if x = f (t) cos t − f 0 (t) sin t and y = f (t) sin t + f 0 (t) cos t.

(c) If x = a (cos θ + θ sin θ) and y = a (sin θ − θ cos θ), where 0 < θ < π2 , show that

d2 y

= .............
dx2 x= π
4

(d) Given that x = sec t − cos t and y = secn t − cosn t, show that

dy
= n.
dx (x,y)=(0,0)

(e) 26 in jots Q47

(f)

Solution.

√ 
−1
 21
(a) Given x = = esin t , we have
−1
esin t

dx d  sin−1 t  12 1d   −1  12 −1 sin−1 t
= e = sin−1 t · esin t ·e
dt dt 2 dt
1 1  −1 − 12 1  −1  12
sin t sin−1 t
= √ e ·e = √ esin t
2 1 − t2 2 1 − t2
√ −1
1 p
−1 esin t
= √ esin t = √ .
2 1 − t2 2 1 − t2

137 MATH 122: Calculus I - Kojoga Aziedu


Similarly,
√ −1
dy ecos t
= − √ .
dt 2 1 − t2
√ −1 √ √ −1
dy dy dt ecos t 2 1 − t2 ecos t y
∴ = · =− √ × √ −1 = − √ −1 = − .
dx dt dx 2 1 − t2 esin t esin t x

(b) Q49
(c)
(d) Q
(e) Q47
(f)

4.8 Higher-order Derivatives and the Formation of Differential


Equations.
Definition 4.63. Higher-order Derivatives.
Given the function f , we form a new function f 0 from it; i.e.
f → f 0.
Also, given f 0 another function f 00 can be formed, etc. The value of this new function f 00 at x is denoted
by
d2 f
 
00 d 0 d df
f (x) = (f (x)) = = 2.
dx dx dx dx
This new function f is called the second derivative of f with respect to x.
00

Similarly, given f 00 we can have f 000 , and


d2 f d3 f
 
000 d d
f (x) = (f 00 (x)) = =
dx dx dx2 dx3
and is called the third derivative of f with respect to x.
Again, if f 000 is differentiable at x, then
d3 f d4 f
 
d d
(f 000 (x)) = = = f (4) (x)
dx dx dx3 dx4
is the fourth derivative of f with respect to x.
In general,
dn f dn−1 f
 
(n) d
f (x) = n =
dx dx dxn−1
dn−1 f
provided is differentiable at x.
dxn−1
The functions f (n) for n ≥ 2 are called the higher derivatives of f .
Note that f (n) = Dxn f = Dn f .
d2 y d3 y
Example 4.64. Find and , given that
dx2 dx3

138 MATH 122: Calculus I - Kojoga Aziedu


(a) y = x4 + 4x2 + 3x + 1 x3 (c) x = t2 et , y = sin (t2 )
(b) y 2 =
2−x

Solution.

(a) Given y = x4 + 4x2 + 3x + 1,

dy d
x4 + 4x2 + 3x + 1 = 4x3 + 8x + 3

=
dx dx
d2 y d
4x3 + 8x + 3 = 12x2 + 8

2
=
dx dx
3
dy d 2

= 12x + 8 = 24x.
dx3 dx

x3
(b) Differentiating y 2 = implicitly, we have
2−x
 3 
d 2
 d x dy (2 − x) (3x2 ) − (−1) x3 6x2 − 2x3
y = ⇒ 2y = = .
dx dx 2 − x dx (2 − x)2 (2 − x)2

Differentiating again with respect to x, we have

d 6x2 − 2x3
   
d dy
2y =
dx dx dx (2 − x)2
dy dy d2 y (2 − x)2 (12x − 6x2 ) − (6x2 − 2x3 ) (−2) (2 − x)
⇒2 · +2 2 =
dx dx dx (2 − x)4
(2 − x) (12x − 6x2 ) + 2 (6x2 − 2x3 ) x (x2 − 6x + 12)
= = 2 .
(2 − x)3 (2 − x)3

x3
Thus the second derivative of the implicit function y 2 = is given by
2−x
 2 2
d2 y x (x2 − 6x + 12) x (x2 − 6x + 12)
 2
dy 3x − x3
= − = −
dx2 (2 − x)3 dx (2 − x)3 y (2 − x)2
x (x2 − 6x + 12) y 2 − (2 − x) (3x2 − x3 )
= .
(2 − x)3 y 2

(c)

x = t2 et ⇒ ẋ = 2tet + t2 et = tet (2 + t)
y = sin t2 ⇒ ẏ = 2t cos t2 .
 

Therefore,
dy ẏ tet (2 + t)
= (t + 2) et sec t2 .

= = 2
dx ẋ 2t cos (t )

139 MATH 122: Calculus I - Kojoga Aziedu


Differentiating again with respect to x, we have
d2 y d  d   dt
(t + 2) et sec t2 = (t + 2) et sec t2

2
=
dx dx
 dt dx 
Using the product and chain rules
z  }| { 1
= et sec t2 + (t + 2) et sec t2 + (t + 2) et (2t) sec t2 tan t2 
  
(t + 2) tet
 
Factorising and cancelling out et
z }| { 1
= sec t2 + (t + 2) sec t2 + t (t + 2) sec t2 tan t2 
    
(t + 2) t

  sec (t2 )
2
= t + 3 + t (t + 2) tan t
(t + 2) t
 
t+3
+ tan t2 sec t2 .
 
=
t (t + 2)
Hence,
d3 y
     
d t+3 2
 2
 d t+3 2
 2
 dt
= + tan t sec t = + tan t sec t
dx3 dx t (t + 2) dt t (t + 2) dx
     
d t+3 t+3 d 1
+ tan t2 · sec t2 + + tan t2 sec t2
   
=
dt t (t + 2) t (t + 2) dt (t + 2) tet

t (t + 2) − 2 (t + 3) (t + 1)
+ 2t sec2 t2 · sec t2
 
= 2
t2 (t + 2)
  
t+3 2
 2
 2
 1
+2t + tan t sec t tan t
t (t + 2) (t + 2) tet
 t2 + 6t + 6
 
3 2
 2t + 6 2
 2
 2 1
= 2t sec t + + 2t tan t sec t tan t − 2 .
(t + 2) t (t + 2) (t + 2) tet
2

Example 4.65. Let f (x) = xeax+b where a and b are constants. Find the value of a + b so that
f 0 (0) = 3 and f 00 (0) = 6.

Solution. Given
f (x) = xeax+b
⇒ f 0 (x) = eax+b + axeax+b = (1 + ax) eax+b ;
f 00 (x) = aeax+b + aeax+b + a2 xeax+b = 2a + a2 x eax+b .


Thus
f 0 (0) = 3 ⇒ eb = 3 ⇒ b = ln 3
and
f 00 (0) = 6 ⇒ 2aeb = 6 ⇒ 2a eln 3 = 6a = 6 ⇒ a = 1.


a + b = 1 + ln 3.


g (x)
Example 4.66. Let f (x) = ln . Given g (1) = h (1) = 1 and g 0 (x) = h0 (x) = 2, show that
h (x)
g 0 (x) h0 (x)
f 0 (x) = −
g (x) h (x)
and find the value of f 0 (1). Given that g 00 (1) = λ and h00 (1) = µ, show that
f 00 (1) = λ − µ.

140 MATH 122: Calculus I - Kojoga Aziedu


Solution. By definition,
h i
  d g(x) h(x)g 0 (x)−g(x)h0 (x)
0 d g (x) dx h(x) [h(x)]2
f (x) = = g(x)
= g(x)
dx h (x)
h(x) h(x)
h (x) h (x) g 0 (x) − g (x) h (x) h (x) g (x) − g (x) h0 (x)
0 0
= · =
g (x) [h (x)]2 g (x) h (x)
0 0 0 0
h (x) g (x) g (x) h (x) g (x) h (x)
= − = − .
g (x) h (x) g (x) h (x) g (x) h (x)
g 0 (2) h0 (2) 2 2
∴ f 0 (1) = − = − = 0.
g (1) h (1) 1 1
Also,
d g 0 (x) h0 (x) d g 0 (x) d h0 (x)
     
00
f (x) = − = −
dx g (x) h (x) dx g (x) dx h (x)
0 0
d d
g (x) dx [g (x)] − g (x) dx [g (x)] h (x) dx d
[h0 (x)] − h0 (x) dx
d
[h (x)]
= 2 − 2
[g (x)] [h (x)]
00 0 2 00 0 2
g (x) g (x) − [g (x)] h (x) h (x) − [h (x)]
= 2 −
[g (x)] [h (x)]2
[h (x)]2 g (x) g 00 (x) − [g 0 (x)]2 − [g (x)]2 h (x) h00 (x) − [h0 (x)]2
   
=
[g (x)]2 [h (x)]2
g (x) g 00 (x) − [g 0 (x)]2 h (x) h00 (x) − [h0 (x)]2
= −
[g (x)]2 [h (x)]2
g (x) g 00 (x) [g 0 (x)]2 h (x) h00 (x) [h0 (x)]2
= − − +
[g (x)]2 [g (x)]2 [h (x)]2 [h (x)]2
g 00 (x) [g 0 (x)]2 h00 (x) [h0 (x)]2
= − − + .
g (x) [g (x)]2 [h (x)] [h (x)]2

g 00 (1) [g 0 (1)]2 h00 (1) [h0 (1)]2 λ 22 µ 22


∴ f 00 (1) = − − + = − − + 2
g (1) [g (1)]2 [h (1)] [h (1)]2 1 12 1 1
= λ − 4 − µ + 4 = λ − µ.

Example 4.67. m
Solution. m

Example 4.68. m
Solution. m

4.8.1 Higher Derivatives for Some Standard Functions.


4.8.1.1 Partern Identification.

.
.
.
.

141 MATH 122: Calculus I - Kojoga Aziedu


4.8.1.2 Polynomial Functions.

• Functions of the Form f (x) = xm , m ∈ R.


Given the function f (x) = xm , m ∈ R, we have

f 0 (x) = mxm−1
f 00 (x) = m (m − 1) xm−2
f 000 (x) = m (m − 1) (m − 2) xm−3
f (4) (x) = m (m − 1) (m − 2) (m − 3) xm−4 .

By inspection, we have

f (n) (x) = m (m − 1) (m − 2) · · · (m − (n − 1)) xm−n


= m (m − 1) (m − 2) · · · (m − n + 1) xm−n
m (m − 1) (m − 2) · · · (m − n + 1) (m − n)! m−n
= x
(m − n)!
m!
= xm−n .
(m − n)!

Therefore, if f (x) = xm , m ∈ R, then

m!
f (n) (x) = xm−n . (4.36)
(m − n)!

In particular, if:

 m ∈ Z+ , then
, if m > n,

m! m−n
 (m−n)! x

(n)
f (x) = m!, if m = n,
if m < n.

0,

• Functions of the Form f (x) = (ax + b)m , a, b, m ∈ R.


Let f (x) = (ax + b)m , a, b, m ∈ R, then

d
f 0 (x) = m (ax + b)m−1 · (ax + b) = am (ax + b)m−1
dx
f (x) = a m (m − 1) (ax + b)m−2
00 2

f 000 (x) = a3 m (m − 1) (m − 2) (ax + b)m−3


f (4) (x) = a4 m (m − 1) (m − 2) (m − 3) (ax + b)m−4 .

Therefore,

f (n) (x) = an m (m − 1) (m − 2) (m − 3) · · · (m − n + 1) (ax + b)m−n


m (m − 1) (m − 2) (m − 3) · · · (m − n + 1) (m − n)!
= an (ax + b)m−n .
(m − n)!

Hence,
m!
f (n) (x) = an (ax + b)m−n . (4.37)
(m − n)!

142 MATH 122: Calculus I - Kojoga Aziedu


4.8.1.3 Rational Functions.
1 b
• Functions of the Form f (x) = , a, b ∈ R, x 6= − .
ax + b a
1
If f (x) = = (ax + b)−1 , then
ax + b
f 0 (x) = a (−1) (ax + b)−2
f 00 (x) = a2 (−1) (−2) (ax + b)−3
= a2 (−1)2 (1) (2) (ax + b)−3
f 000 (x) = a3 (−1)3 (1) (2) (3) (ax + b)−4
= a3 (−1)3 3! (ax + b)−4
f (4) (x) = a4 (−1)4 4! (ax + b)−5 .
Thus, in general, we have that
f (n) (x) = (−1)n an n! (ax + b)−(n+1)
(−1)n an n!
= . (4.38)
(ax + b)n+1

In particular, if
1
 f (x) = = a = −1, b = 1, then
1−x
n n 2 n
(−1) (−1) n! (−1) n!
f (n) (x) = n+1 = n+1
(1 − x) (1 − x)
n
(1) n! n!
= n+1 = . (4.39)
(1 − x) (1 − x)n+1
1
 f (x) = = a = b = 1, then
1+x
(−1)n (1)n n! (−1)n n!
f (n) (x) = = . (4.40)
(1 + x)n+1 (1 + x)n+1

4.8.1.4 Trigonometric Functions.

Recall that  π
cos x = sin x +
2
and  π
sin x = − cos x + .
2
• The nth Derivative of the Sine Function.

 Let f (x) = sin x. Then


0
 π
f (x) = cos x = sin x +
2  
 π  π π 2π
f 00 (x) = cos x + = sin x + + = sin x +
2 2 2 2
      
2π 2π π 3π
f 000 (x) = cos x + = sin x+ + = sin x +
2 2 2 2
      
3π 3π π 4π
f (4) (x) = cos x + = sin x+ + = sin x +
2 2 2 2
      
4π 4π π 5π
f (5) (x) = cos x + = sin x+ + = sin x + .
2 2 2 2

143 MATH 122: Calculus I - Kojoga Aziedu


Continuing with the trend, we have that, if f (x) = sin x, then
 nπ 
f (n) (x) = sin x + . (4.41)
2

 Similarly, for f (x) = sin (ax + b), we have


 π
f 0 (x) = a cos (ax + b) = a sin ax + b +
2  
00 2
 π 
2
 π π 2 2π
f (x) = a cos ax + b + = a sin ax + b + + = a sin ax + b +
2 2 2 2
    
000 3
 π 
3 2π π 3 3π
f (x) = a cos ax + b + = a sin ax + b + + = a sin ax + b + .
2 2 2 2

Hence, if f (x) = sin (ax + b), then


 nπ 
f (n) (x) = an sin ax + b + . (4.42)
2
• The nth Derivative of the Cosine Function.

 Let f (x) = sin x. Then




0
 π
f (x) = − sin x = cos x +
2  
 π  π π 2π
f 00 (x) = − sin x + = cos x + + = cos x +
2 2 2 2
      
2π 2π π 3π
f 000 (x) = − sin x + = cos x+ + = cos x +
2 2 2 2
      
3π 3π π 4π
f (4) (x) = − sin x + = cos x+ + = cos x +
2 2 2 2
      
4π 4π π 5π
f (5) (x) = − sin x + = cos x+ + = cos x + .
2 2 2 2

Continuing with the trend, we have that, if f (x) = cos x, then


 nπ 
f (n) (x) = cos x + . (4.43)
2

 Again, when f (x) = cos (ax + b), then


 π
f 0 (x) = −a sin (ax + b) = a cos ax + b +
 2 
 π  2π
f 00 (x) = −a2 sin ax + b + = a2 cos ax + b +
2 2
 
000 3
 π 3 3π
f (x) = −a sin ax + b + = a cos ax + b + .
2 2

Hence, if f (x) = cos (ax + b), then


 nπ 
f (n) n
(x) = a cos ax + b + . (4.44)
2

144 MATH 122: Calculus I - Kojoga Aziedu


4.8.1.5 Exponential and Logarithmic Functions.

• Given f (x) = ex , then obviously


f (n) (x) = ex . (4.45)

• Given f (x) = eax , we have

f 0 (x) = aeax
f 00 (x) = a2 eax
f 000 (x) = a3 eax .

Therefore,
f (n) (x) = an eax . (4.46)

• Let f (x) = ln x. Then


1
f 0 (x) = = x−1
x
f 00 (x) = (−1) x−2
f 000 (x) = (−1) (−2) x−3 = (−1)2 (1) (2) x−3
f (4) (x) = (−1) (−2) (−3) x−4 = (−1)3 (1) (2) (3) x−4
f (5) (x) = (−1)4 (1) (2) (3) (4) x−5 .

Hence, by inspection, we have that

f (n) (x) = (−1)n−1 (n − 1)!x−n , n ∈ N.

Hence, when f (x) = ln x, then

(−1)n−1 (n − 1)!
f (n) (x) = , n ∈ N. (4.47)
xn

• Given f (x) = ln (ax + b),


a
f 0 (x) = = a (ax + b)−1
ax + b
f 00 (x) = (−1) a2 (ax + b)−2
f 000 (x) = (−1) (−2) a3 (ax + b)−3
= (−1)2 (1) (2) a3 (ax + b)−3
(4)
f (x) = (−1)3 (1) (2) (3) a4 (ax + b)−4
= (−1)3 3!a4 (ax + b)−4
(5)
f (x) = (−1)4 (1) (2) (3) (4) a5 (ax + b)−5
= (−1)4 4!a5 (ax + b)−5 .

Therefore,

f (n) (x) = (−1)n−1 (n − 1)!an (ax + b)−n


(−1)n−1 (n − 1)!an
= , n ≥ 1.
(ax + b)n

145 MATH 122: Calculus I - Kojoga Aziedu


4.8.1.6 Second Derivatives for Parametrically Defined Functions.

Let
x = f (t) , y = g (t)
define a function with parameter t. Then

dy g 0 (t)
= = F (t) .
dx f 0 (t)

Now,

d2 y
 
d dy d
2
= = (F (t))
dx dx dx dx
d g 0 (t)
 
d dt dt
= (F (t)) · = ·
dt dx dt f 0 (t) dx
 0   0 
d g (t) dt 1 d g (t)
= · = .
dt f 0 (t) dx f 0 (t) dt f 0 (t)

d2 y
Example 4.69. Find if
dx2
2 − 3t 3 + 2t (b) x = cos3 θ, y = 3 sin θ − sin3 θ.
(a) x = ,y=
1+t 1+t

Solution.

2 − 3t
(a) For x = ,
1+t
(1 + t) (−3) − (2 − 3t) −3 − 3t − 2 + 3t
x0 (t) = 2 =
(1 + t) (1 + t)2
−5
=
(1 + t)2
3 + 2t
and when y = ,
1+t
(1 + t) (2) − (3 + 2t) −1
y 0 (t) = 2 = .
(1 + t) (1 + t)2

dy y 0 (t) −1 −5 1
⇒ = 0 = 2 ÷ 2 = .
dx x (t) (1 + t) (1 + t) 5

Therefore,

d2 y
 
dy 1
2
= = 0.
dx dx 5

(b) x = cos3 θ, y = 3 sin θ − sin3 θ.

ẋ = −3 cos2 θ sin θ
ẏ = 3 cos θ − 3 sin2 θ cos θ

146 MATH 122: Calculus I - Kojoga Aziedu


3 cos θ 1 − sin2 θ

dy ẏ 3 cos θ − 3 sin2 θ cos θ
∴ = = =
dx ẋ −3 cos2 θ sin θ −3 cos2 θ sin θ
2
3 cos θ cos θ cos θ
= = − = − cot θ.
−3 cos2 θ sin θ sin θ
Hence,
d2 y d d dθ
2
= − (cot θ) = − (cot θ)
dx dx dθ dx
1 1
= csc2 θ · =−
2
−3 cos θ sin θ 3 cos θ sin3 θ
2

= −3 sec2 θ csc3 θ.

Example 4.70. Find the nth derivatives of the following functions. Evaluate these derivatives for the
following values of a and b.
1 (b) f (x) = eax sin bx, a = 1, b = 1.
(a) f (x) = , a = 2, b = 1.
(x − a) (x − b)

Solution.

(a) Expressing f (x) in partial fractions, i.e.


1 Y Z
f (x) = = f (x) = + ,
(x − a) (x − b) x−a x−b
we have
1 1
Y = , Z=− .
a−b a−b
Therefore,
dn
 
(n) 1 1 1 1
f (x) = · − ·
dxn a − b x − a a − b x − b
1 dn 1 dn
   
1 1
= −
a − b dxn x − a a − b dxn x − b
1 dn  −1  1 dn 
(x − b)−1

= n
(x − a) − n
a − b dx a −b dx
n
(−1)n n!
  
1 (−1) n! 1
= − ,
a − b (x − a)n+1 a − b (x − b)n+1
using equation (4.38), i.e.
dn  −1  (−1)n an n!
(ax + b) = .
dxn (ax + b)n+1

(−1)n n! (−1)n n!
   
(n) 1 1
∴ f (x) = −
a − b (x − a)n+1 a − b (x − b)n+1
(−1)n n!
 
1 1
= − .
a−b (x − a)n+1 (x − b)n+1
Hence, when a = 2 and b = 1, we have
dn dn
   
1 1 1
− =
dxn x − 2 x − 1 dxn (x − 1) (x − 2)
 
n 1 1
= (−1) n! −
(x − 1)n+1 (x − 2)n+1
" #
n+1 n+1
(x − 2) − (x − 1)
= (−1)n n!
[(x − 1) (x − 2)]n+1

147 MATH 122: Calculus I - Kojoga Aziedu


(b) Given f (x) = eax sin bx, we have

f 0 (x) = beax cos bx + aeax sin bx


= eax (b cos bx + a sin bx) .

But
b cos bx + a sin bx = R sin (bx + ε)

 
b
where R = a + b and ε = tan
2 2 −1
.
a

∴ f 0 (x) = Reax sin (bx + ε)


1
= a2 + b2 2 eax sin (bx + ε) .

So,

∴ f 00 (x) = R (aeax sin (bx + ε) + beax cos (bx + ε))


= Reax (a sin (bx + ε) + b cos (bx + ε)) .

Again,
a sin (bx + ε) + b cos (bx + ε) = Q sin (bx + ε + β)

 
b
where Q = a + b = R and β = tan
2 2 −1
= ε.
a

∴ f 00 (x) = Reax (R sin (bx + ε + ε))


2
= a2 + b2 2 eax sin (bx + 2ε) .

Similarly,
 23
f 000 (x) = a2 + b 2 eax sin (bx + 3ε) .

Hence, in general
 n2
f (n) (x) = a2 + b 2 eax sin (bx + nε)
  
2
n
2 2 ax −1 b
= a +b e sin bx + n tan .
a

When a = b = 1, then

dn x √ x √ x
  
n e sin x + n tan −1 1 n e sin x + n tan−1 (1)

(e sin x) = 2 = 2
dxn 1
√ x  nπ 
= 2n e sin x + .
4

4.8.1.7 The nth Derivative of a Product of Two Functions.

Theorem 4.71. Leibniz Theorem.


Let f and g are nth order differentiable functions of x. Then
n 
dn

X n
[f (x) g (x)] = f (n−r) (x) g (r) (x) . (4.48)
dxn r
r=0

148 MATH 122: Calculus I - Kojoga Aziedu


Proof. We prove this by the principle of mathematical induction.
Let n = 1. Then
d
LHS = [f (x) g (x)] = f 0 (x) g (x) + f (x) g 0 (x) .
dx
1      
X 1 1 1
RHS = f (1−r) (r)
(x) g (x) = 0
f (x) g (x) + f (x) g 0 (x)
r 0 1
r=0
= f (x) g (x) + f (x) g 0 (x) = LHS.
0

Therefore, the theorem is true when n = 1.


Suppose the theorem is true for n = k > 1. Then
k
dk
 
X k
[f (x) g (x)] = f (k−r) (x) g (r) (x) .
dxk r
r=0

Differentiating both sides with respect to x, we have


" k   #
d dk
 
d X k
[f (x) g (x)] = f (k−r) (x) g (r) (x)
dx dxk dx r=0 r
k 
dk+1

X k d  (k−r)
(x) g (r) (x)

[f (x) g (x)] = f
dx k+1 r dx
r=0
k  
X k  (k−r+1)
(x) g (r) (x) + f (k−r) (x) g (r+1) (x)

= f
r
r=0
k   k  
X k (k−r+1) (r)
X k
= f (x) g (x) + f (k−r) (x) g (r+1) (x) .
r r
r=0 r=0

k  
X k
For the term f (k−r) (x) g (r+1) (x), let s = r + 1. Then when r = 0, s = 1 and when r = k,
r
r=0
s = k + 1.
k   k+1  
X k (k−r) (r+1)
X k
∴ f (x) g (x) = f (k−(s−1)) (x) g ((s−1)+1) (x)
r s−1
r=0 s=1
k+1
X k 
= f (k−s+1) (x) g (s) (x)
s−1
s=1
k+1
X k 
= f (k−r+1) (x) g (r) (x)
r−1
r=1

149 MATH 122: Calculus I - Kojoga Aziedu


for some dummy variable r.
k  k+1 
dk+1
 
X k (k−r+1) (r)
X k
∴ [f (x) g (x)] = f (x) g (x) + f (k−r+1) (x) g (r) (x)
dxk+1 r r−1
r=0 r=1
k  
X k
= f (k+1) (x) g (x) + f (k−r+1) (x) g (r) (x)
r
r=0
k  
X k
+ f (k−r+1) (x) g (r) (x) + f (x) g (k+1) (x)
r−1
r=1
k    
(k+1) (k+1)
X k k
= f (x) g (x) + f (x) g (x) + + f (k−r+1) (x) g (r) (x)
r r−1
r=0
k  
(k+1) (k+1)
X k+1
= f (x) g (x) + f (x) g (x) + f (k−r+1) (x) g (r) (x)
r
r=0
k+1  
X k+1
= f (k−r+1) (x) g (r) (x)
r
r=0

since      
k k k+1
+ = .
r r−1 r
That is the theorem being true for n = k ⇒ n = k + 1 is true. Therefore, by the principle of
mathematical induction, the theorem is true ∀n ∈ N. Hence,
k
dk
 
X k
[f (x) g (x)] = f (k−r) (x) g (r) (x) .
dxk r
r=0

In particular,
1  
d X 1
[f (x) g (x)] = f (1−r) (x) g (r) (x)
dx r
r=0
   
1 0 1
= f (x) g (x) + f (x) g 0 (x)
0 1
= f 0 (x) g (x) + f (x) g 0 (x) (4.49)

which is the product rule, and


2 
d2

X 2
(f (x) g (x)) = f (2−r) (x) g (r) (x)
dx2 r
r=0
     
2 (2) (0) 2 (1) (1) 2
= f (x) g (x) + f (x) g (x) + f (0) (x) g (2) (x)
0 1 2
= f 00 (x) g (x) + 2f 0 (x) g 0 (x) + f (x) g 00 (x) . (4.50)

Example 4.72. Use Leibniz theorem to find the nth derivative of each of the following functions.

150 MATH 122: Calculus I - Kojoga Aziedu


(a) y = x2 eax , a ∈ R (b) z = x3 sin 2x.

Solution. By Leibniz theorem, if y = f (x) g (x), then


n 
dn

X n
[f (x) g (x)] = f (n−r) (x) g (r) (x) .
dxn r
r=0

(a) For y = x2 eax , let g (x) = x2 and f (x) = eax . Notice that g (n) (x) = 0 for n ≥ 3.
n 
dn

2 ax
X n
f (n−r) (x) g (r) (x)

xe =
dxn r
r=0
     
n (n) (0) n (n−1) (1) n
= f (x) g (x) + f (x) g (x) + f (n−2) (x) g (2) (x)
0 1 2
     
n n 0 n
= (n)
f (x) g (x) + f (n−1)
(x) g (x) + f (n−2) (x) g 00 (x)
0 1 2
n (n − 1) (n−2)
= f (n) (x) g (x) + nf (n−1) (x) g 0 (x) + f (x) g 00 (x)
2
dn ax 2 d n−1
ax d 2 n (n − 1) dn−2 ax d2
(e ) 2 x2
  
= n
(e ) x + n n−1
(e ) x + n−2
dx dx dx 2 dx dx
n (n − 1)
= an x2 eax + nan−1 eax (2x) + an−2 eax (2)
2
= an x2 eax + 2nan−1 xeax + n (n − 1) an−2 eax

or
dn
x2 eax = an−2 eax a2 x2 + 2nax + n (n − 1) . (4.51)
 
dx n

(b) For z = x3 sin 2x, let f (x) = sin 2x and g (x) = x3 . Then
(n) n
 nπ 
f (x) = 2 sin 2x +
2
and (
3!
x3−n , n<4
g (n) (x) = (3−n)!
0, n ≥ 4.
In particular,
 
(n−1) n−1 (n − 1) π  nπ π 
f (x) = 2 sin 2x + = 2n−1 sin 2x + −
2 2 2
 nπ 
= −2n−1 cos 2x +
 2 
(n − 2) π  nπ 
f (n−2) (x) = 2n−2
sin 2x + = 2n−2 sin 2x + −π
2 2
 nπ 
= −2n−21 sin 2x +
 2   
(n − 3) π nπ 3π
f (n−3) (x) = 2n−3
sin 2x + =2 n−3
sin 2x + −
2 2 2
 nπ 
= 2n−3 cos 2x +
 2 
(n − 4) π  nπ 
f (n−4) (x) = 2n−4
sin 2x + = 2n−4 sin 2x + − 2π
2 2
 nπ 
= 2n−4 sin 2x +
2
151 MATH 122: Calculus I - Kojoga Aziedu
and
g 0 (x) = 3x2
g 00 (x) = 6x
g 000 (x) = 6
g (4) (x) = 0.

n 
dn

3
X n
f (n−r) (x) g (r) (x)

∴ x sin 2x =
dxn r
r=0
   
n (n) (0) n
= f (x) g (x) + f (n−1) (x) g (1) (x)
0 1
   
n (n−2) (2) n
+ f (x) g (x) + f (n−3) (x) g (3) (x)
2 3
n (n − 1) (n−2)
= f (n) (x) g (x) + nf (n−1) (x) g 0 (x) + f (x) g 00 (x)
2
n (n − 1) (n − 2) (n−3)
+ f (x) g 000 (x)
6
dn 3 dn−1 d
x3
 
= n
(sin 2x) x + n n−1
(sin 2x)
dx dx dx
n (n − 1) dn−2 d2
(sin 2x) 2 x3

+
2 dxn−2 dx
n−3
n (n − 1) (n − 2) d d3 3

+ n−3
(sin 2x) x
 6 nπ  dx dx3 
 nπ 
= 2n sin 2x + x3 + n −2n−1 cos 2x + 3x2

2 2
n (n − 1)   nπ 
+ −2n−2 sin 2x + (6x)
2 2
n (n − 1) (n − 2) n−3  nπ 
+ 2 cos 2x + (6)
6 nπ  2 
nπ 
= 2n x3 sin 2x + − 3n · 2n−1 x2 cos 2x +
2 2
 nπ 
−3n (n − 1) 2n−2 x sin 2x +
 2 nπ 
n−3
+n (n − 1) (n − 2) 2 cos 2x + .
2

4.8.1.8 Evaluating nth Order Derivatives at Given Points.

Example 4.73. Find the value of


n
X f (i) (0)
i=0
i!
if f (x) = (1 − x)n .

Solution.

Example 4.74. Let f (x) = xn . Show that


n
X (−1)i f (i) (1)
= 0.
i=0
i!

152 MATH 122: Calculus I - Kojoga Aziedu


Solution.

2
Example 4.75. (a) Let f (x) = sin−1 x . Show that
1 − x2 f 00 (x) − xf 0 (x) − 2 = 0.


Hence, show that


f (n+2) (0) = n (2n − 1) f (n) (0) .

(b) If f (x) = ex sin x, show that


√  nπ 
f (n) (0) = 2n sin .
4
Solution.
2
(a) Given f (x) = sin−1 x ,
 d
f 0 (x) = 2 sin−1 x sin−1 x

dx
1
= 2 sin−1 x √

1 − x2
or

1 − x2 f 0 (x) = 2 sin−1 x.
Differentiating both sides again with respect to x, we have
d √  d
1 − x2 f 0 (x) sin−1 x

= 2
dx dx
1 − 1 √ 2
(−2x) 1 − x2 2 f 0 (x) + 1 − x2 f 00 (x)

= √
2 1 − x2
−xf 0 (x) √ 2
√ + 1 − x2 f 00 (x) = √
1−x 2 1 − x2
−xf 0 (x) + 1 − x2 f 00 (x)

= 2
or
1 − x2 f 00 (x) − xf 0 (x) − 2 = 0


as required.

Using Leibniz theorem, we have


dn   00 dn
2
[xf 0 (x)]

n
1 − x f (x) = n
dx dx
or n   n  
X n (n−r) (r)
X n
F (x) G (x) = H (n−r) (x) K (r) (x)
r r
r=0 r=0

where F (x) = f (x), G (x) = 1 − x , H (x) = f (x) and K (x) = x. Now


00 2 0

n      
X n (n−r) (r) n (n) (0) n
F (x) G (x) = F (x) G (x) + F (n−1) (x) G(1) (x)
r 0 1
r=0
 
n
+ F (n−2) (x) G(2) (x)
2
n (n − 1) (n)
= f (n+2) (x) · 1 − x2 + nf (n+1) (x) · (−2x) +

f (x) · (−2)
2
= 1 − x2 f (n+2) (x) − 2nxf (n+1) (x) − n (n − 1) f (n) (x) .


153 MATH 122: Calculus I - Kojoga Aziedu


and
n      
X n (n−r) (r) n (n) (0) n
H (x) K (x) = H (x) K (x) + H (n−1) (x) K (1) (x)
r 0 1
r=0

= f (n+1) (x) · x + nf (n) (x) · 1


= xf (n+1) (x) + nf (n) (x) .

dn 00 dn 0
where [f (x)] = f (n+2)
(x), [f (x)] = f (n+12) (x) and f (0) (x) = f (x).
dxn dxn
Therefore,

1 − x2 f (n+2) (x) − 2nxf (n+1) (x) − n (n − 1) f (n) (x) = xf (n+1) (x) + nf (n) (x)


1 − x2 f (n+2) (x) = (2n − 1) xf (n+1) (x) + n (2n − 1) f (n) (x)




or

f (n+2) (0) = n (2n − 1) f (n) (0) .

(b) Given f (x) = ex sin x, then by Example 4.70, we have

(n) n
x
 nπ 
f (x) = 2 e sin x +
2
4 
√ x  nπ
= n
2 e sin x + .
4
π
with a = b = 1 and = tan−1 (1).
4
√  nπ 
f (n) (0) = 2n e0 sin 0 +
√  nπ  4
= 2n sin .
4

Example 4.76. Let f (x) = x9 ex . Obtain a formula for f (n) (x). Hence, evaluate f (10) (0).

Solution. m

4.8.2 Forming Differential Equations.


Definition 4.77. Differential Equations (See Chapter 8 for more).
A differential equation is an equation involving an unknown function and one or more of its derivatives.

To form a differential equation, we simply eliminate arbitrary constants.

Example 4.78. Obtain the differential equation satisfied by the function

y = ax + bex .

154 MATH 122: Calculus I - Kojoga Aziedu


Solution. First of all, we rewrite the function

y = ax + bex

as
ye−x = axe−x + b
so as to isolate the constant b for elimination upon differentiating both sides with respect to x. Thus,
d d
ye−x = axe−x + b
 
dx dx
⇒ y 0 e−x − ye−x = ae−x − axe−x ⇒ (y 0 − y) e−x = a (1 − x) e−x

or
y0 − y
= a.
1−x
Differentiating again, we have

d y0 − y
 
d
= (a)
dx 1 − x dx
(1 − x) (y 00 − y 0 ) + y 0 − y
⇒ = 0
(1 − x)2
⇒ (1 − x) y 00 − y 0 + xy 0 + y 0 − y = 0

or
(1 − x) y 00 + xy 0 − y = 0.
Therefore, the differential equation that satisfies

y = ax + bex

is
(1 − x) y 00 + xy 0 − y = 0.

Example 4.79. Prove that if xy = px2 + q + 21 x2 ln x, where q and q are constants, then

d2 y dy
x2 2
+x = x + y;
dx dx
and if
sin x
y= ,
x2
show that
d2 y dy
x2 + 4x + x2 + 2 y = 0.

dx 2 dx

Solution. Given that xy = px2 + q + 21 x2 ln x, we have

1
y + xy 0 = 2px + x ln x + x
2
or
y y0 1 1
+ = p + ln x + .
2x 2 2 4
Thus
y0 y y 00 1
− 2+ =
2x 2x 2 2x
155 MATH 122: Calculus I - Kojoga Aziedu
when we differentiate again with respect to x. Multiplying through by 2x2 , we get
d2 y dy
x2 + x =x+y
dx2 dx
as required. Also,

sin x
⇒ x2 y = sin x.
y=
x2
Differentiate both sides twice with respect to x:
x2 y 0 + 2xy = cos x
and
x2 y 00 + 2xy 0 + 2xy 0 + 2y = − sin x
⇒ x2 y 00 + 4xy 0 + 2y = −x2 y, ∵ −x2 y = − sin x
or
d2 y dy
x2 2

+ 4x + x + 2 y = 0.
dx2 dx

4.8.3 Differential Equations Involving nth Derivatives.


−1
Example 4.80. If f (x) = etan x , show that
1 + x2 f (n+1) (x) + (2nx − 1) f (n) (x) + n (n − 1) f (n−1) (x) = 0


dn f (x)
where f (n) (x) = .
dxn
−1
Solution. Given f (x) = etan x
, we have
ln f (x) = tan−1 x
and so
f 0 (x) 1
=
f (x) 1 + x2
or
f (x) = 1 + x2 f 0 (x) .


We differentiate the above equation n-times with respect to x, i.e.


dn f (x) dn  2
 0 
= 1 + x f (x) .
dxn dxn
Applying Leibniz’s theorem, we have
n   n−r
(n)
X n d 0 dr
1 + x2

f (x) = (f (x))
r dx n−r dx r
r=0
     
n (n+1) 2 n (n) n
f (n−1) (x) (2)

= f (x) 1 + x + f (x) (2x) +
0 1 2
= 1 + x2 f (n+1) (x) + 2nxf (n) (x) + n (n − 1) f (n−1) (x)


or
1 + x2 f (n+1) (x) + (2nx − 1) f (n) (x) + n (n − 1) f (n−1) (x) = 0


as required.

156 MATH 122: Calculus I - Kojoga Aziedu


4.9 Exercises.
1. Use the definition of the derivative to differentiate each of the following functions.

(a) x2 − 2x + 3 (e) (x + 1) (x − 4)
(b) x3 + x2 + x + 1 √
(f) (x − 1) x + 2
x2 + 5
(c) 4
x2 − 3 (g) √
x − 4x
x2 + x + 3 √
(d)
(x − 1)2 (h) 1 − 5x − x2

2. Recall that, the derivative of a function f (x) at the point x = a is given by


f (x) − f (a)
f 0 (a) = lim .
x→a x−a
Use this to evaluate each of the following.

(a) −2 − (x + 3)2 , a = 2 x−2


(e) ,a=0
x2 − 3x + 2
(b) x − cos x, a = π
3x + 6
9 (f) , a = −2
(c) + 9, a = 1 x3 + 8
x (g) x3 (x + 1)2 , a = 1
1
(d) x2 − + 1, a = 1 (h) (x − 3) (x2 − 5) , a = 4.
x

3. Let f (x) = x8 − 2x + 3. Show that


f 0 (z) − f 0 (2)
lim = 127.
z→2 z−2

4. By interpreting the expressions as appropriate derivatives, evaluate the following limits.


tan−1 (1 + h) − π
x3 − 1
(a) lim 4 (c) lim
h→0 h x→1 x − 1
−1
3 sec z − π −
(b) lim (d) lim
z→2 z−2 →

5. Use the rules of differentiation to find the derivatives the following functions with respect to x.
√ √ n
(a) x4 − 4x3 + x2 (k)

1 − 2 − x − x4 (f) x + 1 + x 2

x3 − 4x x3 + x2 + x
(b) x cot2 (x2 )
2 (g) sin (sec x 2
) (l)
2x + 3x sin x + cos x
3x − 2x−1 (h) −1 3
cos (4x + 2x − 1) 2  
−x2
(c) (m) ln x 2
− xe
2x2 + 4x−1 
1

(i) tan −1
√ √
(n)
3
x3 (x + 1)2 (x − 1) 2x + 1 e 1+5x2
· 4x
(d) "√
x3 + 1 2−1
#
(o) (x3 − 2x)
2 ln x
1 + x
(e)
1

3 (j) tan −1
(p)
2

1−x 1−x 3 x π sin x tan x

6. If the function defined piecewise as


(
x2 + a, x>2
f (x) =
bx2 − 3ax, x ≤ 2

is differentiable at x = 2, find the required value s of a, b.

157 MATH 122: Calculus I - Kojoga Aziedu


7. The function f (x) is defined by
(
2x + b, x < 2
f (x) =
ax + x, x ≥ 2

where a and b are constants. Given that f (x) is continuous and differentiable at x = 2, find the
values of a and b.
8. Check Calculus of a single variable page 711 .....,
Prince’s bk.......for more parametric diff
dy
9. For each of the following functions, find .
dx
(a) xy = ex−y (g) xexy = y + sin2 x at x = 0.
 √  √
x − 1 − x2 (h) y = cot −1
cos 2x at x = π6
(b) y = tan −1

x + 1 + x2 p √
 2
 (i) y = sec x
1−x √
(c) y = ln  
1 + x2 5x + 12 1 − x2
√ (j) y = sin−1
(d) y = sin x + y 13
√ !   a+b+2x
1 + x2 − 1 a+x
(e) y = tan −1
(k) y= at x = 0
x b+x
√ √
(f) (l) x = tt and y = tt .
t
x = a cos t cos 2t, y = a sin t cos 2t

1 − t2
    
dy 2t 1 1 −1 2t
10. Find if x = and y = tan cos−1
+ sin .
dx 1 − t2 2 1 + t2 2 1 − t2
11. Find the value of the constants a and b such that
d 1 + x4 + x5
 
2 4
= ax3 + bx.
dx 1 + x + x

12. Let f be a function defined for all x ∈ R\ {0}. If f is differentiable at x and f (x3 ) = x5 for all
x ∈ R\ {0}, find f 0 (27).
13. Let f (x) = g (x) h (x) k (x) for all real values of x, where g, h and k are all differentiable functions
of x. If f 0 (2) = 18f (2), g 0 (2) = 3g (2), h0 (2) = −4h (2) and k 0 (2) = βk (2), find the value of β.
14. Let f (x) = 3x2 + 4xg 0 (1) + g 00 (2) and g (x) = 2x2 + 3xf 0 (2) + f 00 (3). Show that
f 00 (3) + g 00 (2) = 10.

15. Let f (x) = cos x cos 2x cos 4x cos 8x. Show that
sin 16x
f (x) = .
24 sin x
π  √
Hence, or otherwise, show that f 0
= 2.
4
16. Find f (0) if f (x) = (1 + x) (1 + x ) (1 + x4 ) (1 + x6 ) · · · (1 + x2n ).
0 2

17. Let f (x) = mx2 + nx + p. Show that


f 0 (1) + f 0 (4) − f 0 (5) = n.

18. Let f (x) = (ax + b) sin x + (cx + d) cos x where a, b, c, d are constants. Find the values of a, b, c,
such that f 0 (x) = x cos x.
19. For each of the following functions, find the indicated differential coefficient (derivative).

158 MATH 122: Calculus I - Kojoga Aziedu


d2 x (f)
(a) y = x + ex ;
dy 2 (g)
d2 P
(b) 2 2
P = a cos θ + b sin θ; 2 2
(h)
dθ2
dy2 (i)
(c) ax2 + 2hxy + by 2 = 1; 2
dx (j)
1 1
(d) x2 + y 2 = t + , x4 + y 4 = t2 + 2 (k)
t t
(e) (l)

20. Let 2x3 − 3x2 y 2 + 4x − y + 7 = 0 where y = y (x) is a differentiable of x. Find y 00 (1) if y 0 (1) = 1.
h  x i d2 y
21. Let x = cos x + ln tan and y = sin x. Find x for which 2 = 0.
2 dx
22. Let f be twice differentiable and injective on an open interval I. Show that if g = f −1 , then

f 00 (g (x))
g 00 (x) = .
[f 0 (g (x))]3

23. Let ˆ x
dt
f (x) = √ .
2 1 + t4
−1 0
Show that f has an inverse and find (f ) (0).

24. Prove that h : (−∞, 1] → [1, ∞) defined by



h (x) = 1−x
0
is injective and that h−1 , the inverse of h exists. Hence, find a formula for (h−1 ) (x).

25. Show that the point (1, 8) lies on the curve


3
f (x) = x3 + 1
0
and find (f − ) where f −1 is the inverse of f .
√ √
26. Let a > 0, x = λasin t and y = µ acos−1 t . Show that
−1

dy y

dx x
where β, λ and µ are constants. [Hint: Write ax = ex ln a and use Example 4.62.]

27. m

28. Given that cos y = x cos (a + y), show that


 
−1 x cos a
y = cot .
x cos a − 1

Hence, or otherwise, show that


dy k
= 2
dx 1 + x − 2x cos a
provided k = sin a.
1 − cos x
29. Find f 0 π
if f (x) = .

4
1 − sin x

159 MATH 122: Calculus I - Kojoga Aziedu


30. Find the intervals of increase and/or decrease of each of the following functions.

(a) f (x) = .....


(b) m
(c) m
(d) m
(e) m
(f) m

31. Find the differential equation having its general solution

y = me−2x + 3x − 4

where m is a constant.

32. Given that


y = x sin x,
establish that
x2 D2 − 2xD + x2 + 2 y = 0,


d
where D = .
dx
33. Given that
ax2 + bxy + cy 2 + d = 0
where a, b, c, d are constants, show that
dy 2ax + by
=− .
dx bx + 2cy

d2 y
Hence, find 2 at the point (1, 2) when a = b = c = d = 1.
dx
34. f the equation
2x2 + 6xy + 4y 2 = 3
dy
defines y as a twice-differentiable function of x, express in terms of x and y and show that
dx
d2 y 3
= .
dx 2
(3x + 4y)3

35. m

36. m

37. Let f (x) = xn where n ∈ N.

(a) Find f (n) (x).


(b) Find f (n) (x) for f (x) = xk given that 0 < k < n.
Xn
(c) Find f (x) if f (x) =
(n)
ai x i .
i=0

38. m

160 MATH 122: Calculus I - Kojoga Aziedu


Chapter 5

Applications of Differentiation.

Now that we know how to differentiate the basic functions, let us now proceed to look into what we
can use these derivatives for.

5.1 Critical (Stationary) Points (Numbers).


Definition 5.1. Critical (Stationary) Points (Values).
Suppose a function f is defined on the interval I ⊆ R. Then the point x = c ∈ I is called a critical
point of f if

• f 0 (c) = 0,

or

• f 0 (c) does not exist, i.e. f is not differentiable at x = c ∈ I.

A critical point is also called a stationary point.

How to Find the Critical Points of a Function.

• Given the function f (x), compute f 0 (x).

• Determine those values of x for which f 0 (x) = 0 or for which f 0 (x) does not exist are the critical
values of x.

Example 5.2. Find the stationary values of the following functions.

(a) f (x) = 3x4 − 8x3 + 6x2 (c) h (x) = x + 2 sin x, x ∈ [0, 2π]

(b) g (x) = x − 3x1/3 (d) i (x) = 2x3 − 3x2 − 12x + 15, [0, 3].

Solution.

161
(a)

f (x) = 3x4 − 8x3 + 6x2 ⇒ f 0 (x) = 12x3 − 24x2 + 12x = 12x x2 − 2x + 1 = 12x (x − 1)2 .


Now,
f 0 (x) = 0 ⇒ 12x (x − 1)2 = 0 ⇒ x = 0, 1.
Therefore, the critical values of f are x = 0 or x = 1. [Since no interval is given, we use its natural
natural domain, R.]

1 2
(b) g (x) = x − 3x 3 ⇒ g 0 (x) = 1 − x− 3
2
0 1 x3 − 1
g (x) = 1 − 2 = 2
x3 x3
which is undefined when
2
x 3 = 0 ⇒ x = 0.
Also,
2
g 0 (x) = 0 ⇒ x 3 − 1 = 0
2
 2 3
⇒ x 3 = 1 ⇒ x 3 = (1)3
x2 = 1 ⇒ x = ±1.

Hence, the critical points of g are x = −1, x = 0 and x = 1.

(c) h (x) = x + 2 sin x ⇒ h0 (x) = 1 + 2 cos x, 0 ≤ x ≤ 2π. Then


1
h0 (x) = 0 ⇒ cos x = −
  2
1 2π
x = cos−1 − = + 2nπ, n ∈ Z.
2 3
2π 2π
But for x ∈ [0, 2π], x = . Therefore, the critical point of h is x = .
3 3

(d) Since i is a polynomial, i0 (x) exists for all x ∈ [0, 3]. Now

i (x) = 2x3 − 3x2 − 12x + 15 ⇒ i0 (x) = 6x2 − 6x − 12


⇒ i0 (x) = 6 x2 − x − 2 = 6 (x + 1) (x − 2)


= 0 ⇔ x = −1, x = 2.

But −1 ∈
/ [0, 3]. Thus the only critical number in the given interval is x = 2.

5.2 Increasing and Decreasing (Monotone) Functions.


Definition 5.3. Increasing Functions.
Let f be a function defined on an interval I. Then f is said to be increasing on I if ∀a, b ∈ I with
a < b, we have f (a) ≤ f (b). f is said to be strictly increasing on I if ∀a, b ∈ I with a < b, we have
f (a) < f (b).

162 MATH 122: Calculus I - Kojoga Aziedu


An increasing function A strictly increasing function.

Figure 5.1: Increasing and strictly increasing functions.

Definition 5.4. Decreasing Functions.


A function f defined on an interval I is said to be decreasing on I if ∀a, b ∈ I with a < b, we have
f (a) ≥ f (b). It is said to be strictly decreasing on I if ∀a, b ∈ I with a < b, we have f (a) > f (b).

163 MATH 122: Calculus I - Kojoga Aziedu


A decreasing function A strictly decreasing function.

Figure 5.2: Decreasing and strictly decreasing functions.

An increasing or decreasing function is called a monotone function. Also function f is said to be


nondecreasing when it is increasing and nonincreasing when it is decreasing.
Summary:
Iff is defined on an interval I and a, b ∈ I, then f is said to be

• strictly increasing on I if • increasing on I if

a < b ⇒ f (a) < f (b) (5.1) a < b ⇒ f (a) ≤ f (b) (5.5)

or or
a > b ⇒ f (a) > f (b) . (5.2) a > b ⇒ f (a) ≥ f (b) . (5.6)

• strictly decreasing on I if • decreasing on I if

a < b ⇒ f (a) > f (b) (5.3) a < b ⇒ f (a) ≥ f (b) (5.7)

or or
a > b ⇒ f (a) < f (b) (5.4) a > b ⇒ f (a) ≤ f (b) . (5.8)

Theorem 5.5. Condition for a Differentiable Function to be Increasing or Decreasing on


an Interval.
Suppose f is differentiable on a closed interval [a, b]. Then if

• f 0 (x) ≥ 0 ∀x ∈ (a, b), then f is increasing on [a, b]. If f 0 (x) > 0 ∀x ∈ (a, b), then f increases
strictly on [a, b].

• f 0 (x) ≤ 0 ∀x ∈ (a, b), then f is decreasing on [a, b]. If f 0 (x) < 0 ∀x ∈ (a, b), then f is strictly
decreasing on [a, b].

164 MATH 122: Calculus I - Kojoga Aziedu


Proof. We want to show that if x1 , x2 ∈ (a, b) with x1 < x2 ⇒ f (x1 ) < f (x2 ) if f 0 (c) ≥ 0 for some
c ∈ (x1 , x2 ) ,and x1 < x2 ⇒ f (x1 ) < f (x2 ) if f 0 (c) ≤ 0 for some c ∈ (x1 , x2 ).

Now, a, b ∈ I

Applications.

We can use the theory of increasing and decreasing functions to


• determine whether or not a function is increasing or decreasing (monotonicity) on a given interval.
• determine the interval(s) of increase and/or decrease (if any) of a given function.
• prove inequalities involving differentiable functions.

5.2.1 Proof of Monotonicity of Functions.


Procedure for finding when f (x) is increasing and when it is decreasing.
• Find the critical points of f , i.e. solve for x in f 0 (x) = 0, and tell when f 0 (x) has no value. List
these values in order.
• At every point other than those values, f 0 (x) is either positive or negative. Tell which.
• Every interval in which
 f 0 (x) > 0 is an interval where f is increasing.
 f 0 (x) < 0 is an interval where f is decreasing.

Example 5.6. Show that the function f (x) = x3 − 3x2 + 3x + 1 increases for all values of x.
Solution. Given f (x) = x3 − 3x2 + 3x + 1, we have
f 0 (x) = 3x2 − 6x + 3 = 3 x2 − 2x + 1


= 3 (x − 1)2 ≥ 0 ∀x ∈ R.
Since f 0 (x) ≥ 0 ∀x ∈ R, we have that f (x) increases for all real values of x.

1
Example 5.7. Let f (x) = where x 6= 2. Show that f is a decreases function for all x ∈ R\ {2}.
x2 −2
Solution. The derivative of f is
3 (2x) 6x
f 0 (x) = − 2 =− < 0 ∀x 6= 2.
(x2 − 2) (x2 − 2)2
1
So the function f (x) = is decreasing on the given interval.
x2 −2

165 MATH 122: Calculus I - Kojoga Aziedu


5.2.2 Determining the Intervals of Increase and Decrease of a Function.
We seek here to determine the subintervals of the given interval on which the function increases or
decreases.
Example 5.8. Determine where the function defined by
f (x) = 3 + 8x2 − x4
is strictly increasing and where is is strictly decreasing.

Solution. From f (x) = 3 + 8x2 − x4 , we have


f 0 (x) = 16x − 4x3 = 4x 4 − x2 = 4x (2 − x) (2 + x) .


The graph of f 0 (x) = 4x (2 − x) (2 + x) is

From the sketch, f 0 (x) > 0 for x ∈ (−∞, −2) ∪ (0, 2) and f 0 (x) < 0 for x ∈ (−2, 0) ∪ (2, ∞). Therefore,
f is strictly increasing on
(−∞, −2) ∪ (0, 2)
and strictly decreasing on
(−2, 0) ∪ (2, ∞) .

Example 5.9. Find the interval of increase and decrease of the following functions.
1
(a) f (x) = x2 + 2x + 2 (d) f (x) = (x + 2) 3 + 8
1
(b) f (x) = (e) f (x) = x + sin x
x2 + 1
(c) f (x) = x3 (5 − x)2 (f) f (x) = sin x + cos x, [0, 2π]

Solution.

(a) f (x) = x2 + 2x + 2 =⇒ f 0 (x) = 2x + 2 = 2 (x + 1) .


Therefore,
f 0 (x) > 0 ⇒ 2 (x + 1) > 0
⇒ x > −1,
and
f 0 (x) < 0 ⇒ 2 (x + 1) < 0
⇒ x < −1.

Hence, f increases on (−1, ∞) and decreases on (−∞, −1).

166 MATH 122: Calculus I - Kojoga Aziedu


1 −2x
(b) f (x) = =⇒ f 0 (x) = .
x2
+1 (x + 1)2
2

Now, for f to be increasing, we have


−2x 2x
f 0 (x) > 0 ⇒ 2 >0⇔ <0
(x2 + 1) (x2 + 1)2
2
⇔ 2x < 0 ∵ x2 + 1 > 0 ∀x
⇔ x < 0,

and for f to be decreasing, we have


−2x 2
f 0 (x) < 0 ⇒ 2 < 0 ⇔ 2x > 0 ∵ x2 + 1 > 0 ∀x
(x2 + 1)
⇔ x > 0,

Hence, f increases on (−∞, 0) and decreases on (0, ∞).

(c) f (x) = x3 (5 − x)2


1
(d) f (x) = (x + 2) 3 + 8

(e) f (x) = x + sin x

(f) f (x) = sin x + cos x, [0, 2π]

Example 5.10. Determine the intervals of the x-axis on which the function

g (x) = 3x4 − 4x3 − 12x2 + 5

is increasing and those on which it is decreasing.

Solution. The derivative of g is

g 0 (x) = 12x3 − 12x2 − 24x = 12x x2 − x − 2 = 12x (x + 1) (x − 2)




= 0 ⇔ x = −1, x = 0, x = 2.

Thus the critical numbers x = −1, x = 0 and x = 2 partition the x-axis into open intervals (−∞, −1),
(−1, 0), (0, 2) and (2, ∞). We now determine the sign of g 0 (x) in these intervals.

Interval Sing of (x + 1) Sing of 12x Sing of (x − 2) Sign of f Conclusion


(−∞, −1) − − − − f decreasing
(−1, 0) + − − + f increasing
(0, 2) + + − − f decreasing
(2, ∞) + + + + f increasing

It follows that f is decreasing on (−∞, −1) and (0, 2), and increasing on (−1, 0) and (2, ∞).

167 MATH 122: Calculus I - Kojoga Aziedu


5.2.3 Determining Positivity and Negativity of Functions.
Definition 5.11. A function f (x) defined on an interval I is said to be positive if

f (x) > 0, ∀x ∈ I

and negative if
f (x) < 0, ∀x ∈ I.


Example 5.12. Let f (x) = ex − 1 − x2 . Show that f is strictly positive for all 0 < x < 1.

Solution. Given f (x) = ex − 1 − x2 ,

d h x √
 
0
i 1 − 1
f (x) = e − 1 − x2 = f (x) = ex − (−2x) 1 − x2 2
dx 2
x
= ex + √ > 0 ∀x (0, 1) .
1 − x2
Thus f is strictly increasing for 0 < x < 1. Therefore,
q
x > 0 ⇒ f (x) > f (0) = e(0) − 1 − (0)2 = 1 − 1 = 0,

i.e. for x > 0 ⇒ f (x) > 0. Therefore, f is a positive function for all x ∈ (0, 1).

Example 5.13. Show that the function f defined for by


x−1
f (x) =
x2 − 2x
always lies below the x-axis for 1 < x < 2.

x−1
Solution. Given that f (x) = , we have
x2 − 2x

0 (x2 − 2x) (1) − (x − 1) (2x − 2) x2 − 2x − 2x2 + 2x + 2x − 2


f (x) = =
(x2 − 2x)2 (x2 − 2x)2
−x2 + 2x − 2 (x2 − 2x + 2)
= = − .
(x2 − 2x)2 (x2 − 2x)2
2
And since x2 − 2x + 2 > 0 and (x2 − 2x) > 0 for all x ∈ R, we conclude that f 0 (x) < 0 ∀x (1, 2). Thus
f is strictly decreasing on (1, 2) so by definition,

x > 1 ⇒ f (x) < f (1) .


1−1
But f (1) = 2 = 0. i.e.
(1) − 2 (1)
x > 1 ⇒ f (x) < 0.
Therefore for all x ∈ (1, 2), f lies below the x axis.

Example 5.14. Let f (x) = x3 (5 − x)2 . Show that f (x) ≥ 0 for all real x such that x ≥ 0.

168 MATH 122: Calculus I - Kojoga Aziedu


Solution. For x ≥ 0,

f 0 (x) = 3x2 (5 − x)2 + x3 (−1) (2) (5 − x)


= 3x2 (5 − x)2 − 2x3 (5 − x) = x2 3 (5 − x)2 − 2x (5 − x)
 

= 5x2 x2 − 8 + 15 = 5x2 (x − 4)2 − 1


   

≥ 0 ∀x ≥ 0.

It follows that f is increasing on [0, ∞). i.e.

x ≥ 0 ⇒ f (x) ≥ f (0) = (0)3 (5 − 0)2 = 0

∴ f (x) ≥ 0 ∀x ≥ 0.

1
Example 5.15. Suppose f is a function defined by f (x) = x 3 (1 − x) for x ≥ 1. Show that in this
interval f is either negative or zero.

Solution.
1 1  −2  1
f (x) = x 3 (1 − x) ⇒ f 0 (x) = x 3 (1 − x) + x 3 (−1)
3  
0 1 −2
  1 1 1
⇒ f (x) = x 3 (1 − x) + x 3 (−1) = x 3 (1 − x) − 1
3 3x2
1 1  
1 − x − 3x2
  
1 x3  2  x3 2 1 1
= x3 = − 2 3x + x − 1 = − 2 3 x + x −
3x2 3x 3x 3 3
1
" " 2 ## 1
" 2 #
x3 2 7 x3 2 7
= − 2 3 x+ − =− 2 x+ − < 0 ∀x ≥ 1
3x 3 9 x 3 3
1
2 2
since x + 7
and x3
> 0 for all x ≥ 1. Thus f decreases strictly for x ≥ 1.

3
> 3 x2

1
∴ x ≥ 0 ⇒ f (x) ≤ f (0) = (1) 3 (1 − 1) = 0.

i.e. f (x) ≤ 0 for all x ≥ 1. Hence, f is either zero or negative when x ≥ 1.

Example 5.16. Prove that the function 1


2
sin x tan x − ln sec x is positive and increasing for 0 < x < π2 .

Solution. Let f (x) = 12 sin x tan x − ln sec x. Then


 sec x tan x
f 0 (x) = 1
2
cos x tan x + sin x sec2 x −
  sec x
sin x sin x
= 21 cos x + sin x sec2 x −
cos x cos x
1 2

= 2 sin x sec x + 1 − sin x sec x
= 21 sin x sec2 x − 2 sec x + 1


= 1
2
sin x (sec x − 1)2 .

Now, for 0 < x < π2 , sin x > 0; and since (sec x − 1)2 > 0 ∀x, we conclude that f 0 (x) > 0 ∀x ∈ 0, π2 .


Thus, f increases for 0 < x < π2 .


Hence, by definition, f increases on 0 < x < π
2
means

x > 0 ⇒ f (x) > f (0) .

169 MATH 122: Calculus I - Kojoga Aziedu


But
f (0) = 12 sin (0) tan (0) − ln (sec (0)) = 1
2
× 0 × 0 − ln (1) = 0.
Thus
f (x) = 12 sin x tan x − ln sec x > 0
Hence, 21 sin x tan x − ln sec x is positive and increasing on 0, π2 .


5.2.4 Proof of Inequalities.


Example 5.17. Prove the following inequalities on the stated intervals.

(a) 1 + x < ex , 0 < x < 1 (d) sin x > π2 x, 0 < x < π


2

1 √ 1
(b) ex < , 0<x<1 (e) x + √ > 2, x > 1
1−x x
√ √ √
(c) 3
x − 3 4 < 3 x − 4, x > 4 (f) ln (1 + x) > x − 12 x2 , x > 0

Solution.

(a) 1 + x < ex , 0 < x < 1


1
(b) ex < , 0<x<1
1−x
√ √ √
(c) 3
x − 3 4 < 3 x − 4, x > 4

(d) sin x > π2 x, 0 < x < π


2

√ 1
(e) x + √ > 2, x > 1
x
(f) ln (1 + x) > x − 12 x2 , x > 0

√ x
Example 5.18. Show that 1 + x < 1 + for x > 0. Hence, prove that ∀n ∈ N, and ∀x > 0,
2
(1 + x)n > 1 + nx.

√ x 1 1

Solution. Let f (x) = 1+x− − 1. Then f 0 (x) = (1 + x)− 2 − 1 .
2 2
Now, since √
x>0⇒ 1 + x > 1,
1 1
we have (1 + x)− 2 = √ < 1. In effect, we have that
1+x
1  1 1 
0 − 21
f (x) = (1 + x) − 1 = √ − 1 < 0.
2 2 1+x
and so f strictly decreases on (0, ∞). Hence,

170 MATH 122: Calculus I - Kojoga Aziedu


.
.
.
.
.

a b
Example 5.19. Prove that for any positive numbers a and b the inequality + ≥ 2.
b a
a 1 b 1
Solution. Let x = so that = . Then x and are positive since a, b > 0. Now consider the
b x a x
1
function f (x) = x+ . Then
x
1
f 0 (x) = 1− 2 = 0 ⇔ x2 = 1 ⇒ x = ±1.
x
Bub −1 ≯ 0, so x = 1. Since the derivative changes sign from minus to plus when passing through
x = 1 (from left to right), the point x = 1 is a minimum. The value of the function at this point is
equal to
1
f (1) = 1+ = 0.
1
Consequently,
1 a b
f (x) ≥ 2 ⇒ x+ ≥ 2 ⇒ + ≥ 2.
x b a

5.2.5 Real World Applications of Monotonicity


Example 5.20. The weight W (in pounds) of a newborn infant during its first three months of life
can be modeled by the equation
1 5 19
W = t3 + t2 − t + 8
3 2 6
where t is measured in months. Determine when the infant was gaining weight and when it was losing
weight.

Solution. Given
1 5 19
W = t3 + t2 − t + 8
3 2 6
where t ∈ [0, 3], we compute the rate at which the baby gains or loses weight as
dW 19
= t2 + 5t − .
dt 6
The peak of this weight loss/gain requires that
dW 19
= 0 ⇒ t2 + 5t − = 0 ⇒ 5t2 + 30t − 19 = 0.
dt 6
So we solve this quadratic for t:
q
−30 ± (30)2 + 4 (5) (19)
t= ⇒ t = 0.58, −6.58.
2 (5)
But −6.58 ∈
/ [0, 3].
∴ t = 0.58.

171 MATH 122: Calculus I - Kojoga Aziedu


Now, t < 0.58 ⇒ W 0 (t) < 0 and t > 0.58 ⇒ W 0 (t) > 0. Thus the function W is increasing for
t > 0.58 and decreasing for t < 0.58. We conclude that the infant was losing weight for the first 0.58
months of its life and then began gaining weight afterwards at least up to the third month.

Example 5.21. I

Solution.

Example 5.22. Inflations functions, Stock functions, etc

Solution.

Example 5.23. The female menstrual stuff

Solution.

5.2.6 Extreme Values (The Extrema).


Definition 5.24. Local and Absolute Maximum and Minimum Values.
Let f be a function defined on the interval I. If c ∈ J ⊂ I such that f (c) ≥ f (x) ∀x ∈ J, then f is
said to have a local maximum on J and f (c) is the local maximum value of f on J.
Also f is said to have an absolute maximum on I if ∀x ∈ I, f (c) ≥ f (x). The number f (c) called the
absolute maximum value of f on I. The point x = c is called the maximum point of f on I.

Similarly, f if f (c) ≤ f (x) ∀x ∈ J, then f is said to have a local minimum on J and f (c) is the local
minimum value of f on J.
And if ∀x ∈ I, f (c) ≤ f (x), then f is said to have an absolute minimum on I and the number f (c)
called the absolute minimum value of f on I. The point x = c is called the minimum point of f on I.

Figure 5.3: Maximum and minimum values.

172 MATH 122: Calculus I - Kojoga Aziedu


Theorem 5.25. Occurrence of Extreme Values.
Let f be a differentiable real-valued function defined on [a, b]. Given that x is a local maximum/minimum
point of f on [a, b], then
f 0 (x) = 0. (5.9)

Proof. Suppose that f has a local maximum at the point x, and choose h such that x + h ∈ [a, b]. Then

f (x) ≥ f (x + h) ⇒ f (x + h) − f (x) ≤ 0.

Now, if h > 0, then


f (x + h) − f (x)
≤0
h
and
f (x + h) − f (x)
f+0 (x) = lim ≤ 0.
h→0 h
Similarly, if h < 0, then
f (x + h) − f (x)
≥0
h
and so
f (x + h) − f (x)
f−0 (x) = lim ≥ 0.
h→0 h
But f is differentiable at x, hence,

f 0 (x) = f+0 (x) = f−0 (x) .

Thus,
f+0 (x) ≤ 0 and f−0 (x) ≥ 0 ⇒ f 0 (x) = 0.

Also, if f has a local minimum at the point x, and that x + h ∈ [a, b] for some h ∈ R. Then

f (x) ≤ f (x + h) ⇒ f (x + h) − f (x) ≥ 0.

173 MATH 122: Calculus I - Kojoga Aziedu


Therefore, for h > 0, we have
f (x + h) − f (x)
f+0 (x) = lim ≥0
h→0 h
and
f (x + h) − f (x)
f−0 (x) = lim ≤0
h→0 h
when h < 0. Hence, for f 0 (x) to exist,

0 ≤ f+0 (x) = f−0 (x) ≥ 0 ⇒ f 0 (x) = 0

completing the proof.

How to Find the Extrema of a Function f Over a Given Interval [a, b].
• Find the critical points of f in (a, b).

• Compute the values of f at each of these critical points.

• Compute the values of f at a and b.

• The absolute maximum and minimum values of f on [a, b] are the largest and the smallest,
respectively, of the computed values of f at the critical points in (a, b) and at a and b.

Example 5.26. Compute the absolute maximum and minimum values of the following functions.

(a) f (x) = 3x4 − 4x3 − 8 on [−1, 2]. (c) h (x) = 2 cos x − x on [0, 2π].

(b) g (x) = x − 3x1/3 on [0, 4]. (d) i (x) = 5x2/3 − x5/3 on [−1, 4].

Solution.

(a) For f (x) = 3x4 − 4x3 − 8,

f 0 (x) = 12x3 − 12x2 = 12x2 (x − 1) .

The critical points of f occur at x for which

f 0 (x) = 12x2 (x − 1) = 0 ⇒ x = 0, x = 1 ∈ [−1, 2] .

174 MATH 122: Calculus I - Kojoga Aziedu


Now,

f (−1) = 3 (−1)4 − 4 (−1)3 − 8 = 3 + 4 − 8 = −1


f (0) = 3 (0)4 − 4 (0)3 − 8 = 0 − 0 − 8 = −8
f (1) = 3 (1)4 − 4 (1)3 − 8 = 3 − 4 − 8 = −9
f (2) = 3 (2)4 − 4 (2)3 − 8 = 48 − 32 − 8 = 8.

Hence, on [−1, 2], the absolute minimum value of f is

min {−1, −8, −9, 8} = −9,

which occurs at x = 1. Similarly, the absolute maximum value of f is

max {−1, −8, −9, 8} = 8

occurring at x = 2.

Figure 5.4: Graph of f (x) = 3x4 − 4x3 − 8 showing its maximum and minimum values.

(b) From Example 5.2 (b), the critical points of g (x) = x − 3x1/3 are x = −1, x = 0 and x = 1 which
are all in [0, 4] except. Now,

g (0) = 0 − 3 (0)1/3 = 0
g (1) = 1 − 3 (1)1/3 = 1 − 3 = −2

g (4) = 4 − 3 (4)1/3 = 4 − 3 4 ≈ −0.76.
3

Therefore, the absolute maximum and minimum values of g on [0, 4] are

max {0, −2, −0.76} = 0

175 MATH 122: Calculus I - Kojoga Aziedu


and
min {0, −2, −0.76} = −2
respectively. These values occur at x = 0 and x = −2 respectively.

Figure 5.5: Graph of g (x) = x − 3x1/3 showing its maximum and minimum values.

(c) When h (x) = 2 cos x − x, then


 
0 −1 1 11π
h (x) = −2 sin x − 1 = 0 ⇒ x = sin − = + 2nπ, n ∈ Z.
2 12
11π
The critical point of h on [0, 2π] is x = . Now,
12
h (0) = 2 cos (0) − 0 = 2
   
11π 11π 11π
h = 2 cos − ≈ −4.81
12 12 12
h (2π) = 2 cos (2π) − 2π ≈ −4.28.

Thus, the absolute maximum value of h on [0, 2π] are

max {2, −4.81, −4.28} = 2

while the absolute minimum value of h is

min {2, −4.81, −4.28} = −4.81


11π
respectively. They occur at x = 0 and x = respectively.
12

Figure 5.6: Graph of h (x) = 2 cos x − x showing its maximum and minimum values.

176 MATH 122: Calculus I - Kojoga Aziedu


(d) Differentiating i (x) = 5x2/3 − x5/3 yields

10 − 1 5 2 5 1 5 (2 − x)
i0 (x) = x 3 − x 3 = x− 3 (2 − x) = 1 = 0.
3 3 3 3x 3
Thus i (x) has two critical points; x = 0 where i0 (x) fails to exist and x = 2 where i0 (x) = 0.

i (−1) = 6
i (0) = 0
i (2) ≈ 4.7
i (4) ≈ 2.5.

Thus the maximum value of 6 occurs at the end point x = −1 and the minimum value of 0 occurs
at the point x = 0 where i (x) is non-differentiable.

Figure 5.7: Maximum and minimum values of i (x) = 5x2/3 − x5/3 on [−1, 4]

Example 5.27. The general equation giving the height of an oscillating object attached to a spring is
r r
k k
z = a sin t + b sin t
m m
where k is the spring constant and m is the mass of the object. Show that the maximum displacement
of the object is √
a2 + b 2
and that the object oscillates with a frequency of
r
1 k
f= .
2π m

[Frequency f is number of oscillations per second, f = 1/T where T is the period.]

Solution. m

177 MATH 122: Calculus I - Kojoga Aziedu


5.2.7 Classification of Critical Points.
Classify critical points is to tell which of them is maximum or minimum. There are two main ways of
distinguishing between (among) critical points; they are the first and second derivative tests.

5.2.7.1 First Derivative (Change of Sign) Test.

In the neighbourhood of a critical point x = c, the following may occur. Either:

• f 0 (x) < 0 for x = c− and f 0 (x) > 0 for x = c+ . Then f has a relative minimum at x = c.

• f 0 (x) > 0 for x = c− and f 0 (x) < 0 for x = c+ . Then f has a relative maximum at x = c.

Figure 5.8:

Example 5.28. Determine the nature of the critical points of the following functions.

(a) f (x) = x3 − 3x2 − 9x + 1. (b) g (x) = x − 2 sin x, x ∈ (c) i (x) = x4 − 4x3 + 13.
[0, 2π].

Solution.

(a) Given f (x) = x3 − 3x2 − 9x + 1, we have

f 0 (x) = 3x2 − 6x − 9 = 3 x2 − 2x − 3


= 3 (x + 1) (x − 3) .

But at the critical point, f 0 (x) = 0, i.e.

3 (x + 1) (x − 3) = 0 ⇒ x = −1, x = 3

are the critical points of f .

Method I: Let 0 < ε1. Then

f+0 (−1 + ε) = 3 (−1 + ε + 1) (−1 + ε − 3)


= 3ε (ε − 4) = 3ε2 − 12ε
< 0 ∀ε ∈ (0, 1)

178 MATH 122: Calculus I - Kojoga Aziedu


and

f−0 (−1 − ε) = 3 (−1 − ε + 1) (−1 − ε − 3)


= 3ε (ε + 4) = 3ε2 + 12ε
> 0 ∀ε ∈ (0, 1) .

Therefore, x = −1 is a maximum point of f .


Similarly, let ε > 0 so that

f+0 (3 + ε) = 3 (3 + ε + 1) (3 + ε − 3)
= 3ε (ε + 4) = 3ε2 + 12ε
> 0 ∀ε ∈ (0, 1)

and

f−0 (3 − ε) = 3 (3 − ε + 1) (3 − ε − 3)
= 3ε (ε − 4) = 3ε2 − 12ε
> 0 ∀ε ∈ (0, 1) .

Therefore, x = 3 is a maximum point of f .

Method II: The critical points of f (x) = x3 − 3x2 − 9x + 1 ⇒ f 0 (x) = 3 (x + 1) (x − 3) are


x = −1, x = 3.
We compute value of f 0 by choosing values of x to the left and to the right of x = −1 and x = 3
and determine their signs. We use a table as shown below:

x −2 −1 0 x 2 3 4
0 0
f (x) 15 0 −9 f (x) −9 0 15
Sign of f 0 (x) + - Sign of f 0 (x) + -
Sketch of slope of f 0 (x) / \ Sketch of slope of f 0 (x) \ /
Classification of point Maximum Classification of point Minimum

179 MATH 122: Calculus I - Kojoga Aziedu


(b) g (x) = x − 2 sin x ⇒ g 0 (x) = 1 − 2 cos x, and

g 0 (x) = 0 ⇒ cos x = 1
2
⇒ x = ± π3 + 2πn, n ∈ N

at the critical points. But since 0 ≤ x ≤ 2π, we have x = π3 , x = 5π


3
as the critical points of g.

29π π 31π 2π 5π 8π
x x
90 3 90 3 3 3
g 0 (x) 0.76 0 −0.35 g 0 (x) −9 0 15
Sign of g 0 (x) + - Sign of g 0 (x) + -
Sketch of slope of g 0 (x) / \ Sketch of slope of g 0 (x) \ /
Classification of point Maximum Classification of point Minimum

(c) i (x) = x4 − 4x3 + 13 ⇒ i0 (x) = 4x3 − 12x2 = 4x2 (x − 3)


i.e. the critical points of i are x = 0 and x = 3. Their nature is determined as follows.

x −0.5 0 0.5 x 2.5 3 3.5


0 0
i (x) −3.5 0 −2.5 f (x) −12.5 0 24.5
Sign of f 0 (x) + - Sign of f 0 (x) + -
Sketch of slope of f (x)
0
/ \ Sketch of slope of f (x)
0
\ /
Classification of point Maximum Classification of point Minimum

180 MATH 122: Calculus I - Kojoga Aziedu


5.2.7.2 Second Derivative Test.

Theorem 5.29. Suppose f is a twice differentiable function of x at the point x = c. If x = c is a local


maximum point of f , then
f 00 (c) < 0 (5.10)
and if x = c is a local minimum point of f , then

f 00 (c) > 0. (5.11)

Converse, if x = c is a critical point and f 00 (x) < 0 then x = c is a local maximum point, and similarly,
if x = c is a critical point and f 00 (x) > 0 then x = c is a local minimum point.

However, if f 00 (x) = 0, then the second derivative test cannot be used to determine the nature of the
critical points.

Example 5.30. Determine whether or not the following functions have maximum values or minimum
values on the associated interval.

(a) f (x) = x3 − 3x2 − 9x + 1 in (−2, 5). (b) g (x) = x − 2 sin x in [0, 2π].

Solution.

(a) We have seen in Example .... that the critical points of f (x) = x3 − 3x2 − 9x + 1 are x = −1 and
x = 3 which are both in (−2, 5).
We now seek to classify them using the second derivative. Now

f 0 (x) = 3x2 − 6x − 9 ⇒ f 00 (x) = 6x − 6 = 6 (x − 1) ,

and

f 00 (−1) = 6 (−1 − 1) = −12 < 0;


f 00 (3) = 6 (3 − 1) = 12 > 0.

Therefore, the point x = −1 is a local maximum point and x = 3 is a local minimum point as
before.

(b)
g (x) = x − 2 sin x ⇒ g 0 (x) = 1 − 2 cos x
has the critical points x = π
3
andx =
in [0, 2π]. But g 00 (x) = 1 + 2 sin x, and so

3
π  π  √
g 00 = 1 + 2 sin =1+ 3>0
 3 3 
5π 5π √
g 00 = 1 + 2 sin = 1 − 3 < 0.
3 3

Hence, the local minimum of g occurs at x = π


3
while the local maximum occurs at x = 5π
3
.

181 MATH 122: Calculus I - Kojoga Aziedu


5.2.8 Optimisation Problems.
Definition 5.31. Optimsation.
Optimisation involve the determination of the “optimal” (the best) value of a quantity.

In optimisation problems we seek to find the largest value or the smallest value that a function can
take subject to some kind of condition. The condition may sometimes be described by some equation
that must always be true on the stated interval.

How to Solve Optimisation Problems.

• Draw a well labelled picture of the situation, and identify all quantities involed.

• Create a corresponding mathematical model of the syatem and explicitly identify the quantity
you want to maximize or minimize.

 Establish a mathematical relationship between the quantity to optimise and the other quant-
ities.

• Identify the constraints.

• Use the restraints to reduce the mathematical relationship to a well-defined and sensible function
of just one variable, with its domain well specified.

• Differentiate this function with the quantity to be optimised, preferrably, be the dependent vari-
able and find the critical numbers

• Use any of following methods to determine if our solution is in fact the absolute minimum or
maximum value we are looking for.

• closed interval method.

 first derivative test.


 second derivative test.

• Answer the question, and do not forget the units!

Example 5.32. Find the maximum area of a rectangle whose perimeter is 100 meters.

Solution.

Step 1: Quantity to optimize is the area A of a rectangle, which is the product of its length L and
width W ;
A = LW. (5.12)

Step 2: The constraint is that the perimeter of the rectangle must be 100 meters. i.e.

2L + 2W = 100 ⇒ L + W = 50. (5.13)

182 MATH 122: Calculus I - Kojoga Aziedu


Step 3: W now express the function (5.12) in terms of a single variable, L, say:

W = 50 − L ⇒ A = L (50 − L) = 50L − L2 .

Step 4: We find the derivative of the function A with respect to the variable L. i.e.
dA d 
50L − L2 = 50 − 2L.

=
dL dL

Step 5: Determine the critical values of A by setting the function dA


dL
to zero. i.e.

dA
= 50 − 2L = 0 ⇒ L = 25.
dL

Step 6: Determine whether this solution gives us the absolute minimum or maximum value. We use
the second derivative test:
d2 A d
2
= [50 − 2L] = −2 < 0 ∀L.
dL dL
Thus the function A yields the maximum value when L = 25.
Step 7: With this value of L, calculate the corresponding optimal value of the function. i.e.

A = 50 (25) − (25)2 = 625 m2

Step 8: We answer the question: The maximum area of a rectangle whose perimeter is 100 metres is
625 square metres.

Example 5.33. Suppose want to construct a box whose base length is 3 times the base width. The
material used to build the top and bottom cost GH¢10/ft2 and the material used to build the sides cost
GH¢6/ft2 . If the box must have a volume of GH¢50 ft3 determine the dimensions that will minimize
the cost to build the box.

Solution.

Let C denote the cost of building the box and V the volume of the box to be built. Then

• the diamension of the top and bottom is LW .

• the dimension of the sides is HW .

• the dimension of the front and back is LH.

• the cost of building the box will be

C = 10 (2LW ) + 6 (2HW ) + 6 (2LH)


= 20LW + 12HW + 12LH.

183 MATH 122: Calculus I - Kojoga Aziedu


But we have been constrained that L = 3W and V = LW H = 50. Thus
50
3W 2 H = 50 ⇒ H = .
3W 2
   
50 50
∴ C = 20LW + 12HW + 12LH = 20 (3W ) W + 12 W + 12 (3W )
3W 2 3W 2
200 600 800
= 60W 2 + + = 60W 2 +
W W W

i.e.
800
C (W ) = 60W 2 + .
W
The derivative of C(W ) is

800 120W 3 − 800


C 0 (W ) = 120W − = .
W2 W2
  1   13
800 3 20
Since W > 0, the only critical value of C is W = = .
120 3
• We now determine whether this walue of W gives us the absolute minimum cost we desire.

 Method I: First derivative test.


  13   31
20 20
Notice that for 0 < W < , C (W ) < 0 and W >
0
, C 0 (W ) > 0.
3 3

 Method II: Second derivative test.


  31 !
00 1600 20 1600
C (W ) = 120 + 3
⇒ C 00 = 120 + "  1 #3
W 3 20 3
3
3 × 1600
= 120 + = 120 + 240 = 360 > 0.
20
  31
20
• Therefore the minimum value of the cost must occurs when W = . The dimensions are
3
  13
20
W = ≈ 1.88 ft
3
1 
20 3
L = 3W = 3 ≈ 5.65 ft
3
50 50
H = 2
=   2 ≈ 4.71 ft.
3W 20 3
3
3

• Hence the minimum cost bulding the box is


"  1 #   32
20 3 20 800
C = 60 +   1 = GH¢637.60.
3 3 20 3
3

184 MATH 122: Calculus I - Kojoga Aziedu


Example 5.34. Find the dimensions of a cylinder so that for a hive volume V , its surface area S is
minimum.

Solutions. Let r be the radius of the base of the cylinder and h its height.

Figure 5.9: A cylinder of radius r and height h..

Then the volume and surface area of the closed cylinder are
V = πr2 h
and
S = 2πr2 + 2πrh
where V is constant. Thus
2πrV 2V
S = 2πr2 + 2
= 2πr2 + .
πr r
dS 2V
∴ = 4πr − 2 .
dr r
dS
For maximum or minimum surface area S, = 0, i.e.
dr
 1/3
2V V
4πr − 2 = 0 ⇒ r = .
r 2π
But
d2 S 4V
2
= 4π + 3
dr r
d2 S

4V
⇒ = 4π +   3
dr2 r=( V )1/3 V 1/3


= 4π + 8π = 12π > 0.
1/3

V
Therefore, S has a minimum value when r = . Now,

V 1/3 V 1/3
 
V V 2π V 2π
h = =   3 = V

πr2 V 1/3 π
π 2π

 1/3
V
= 2 .

Hence the dimensions of a cylinder of volume V so that its surface area S is minimum is radius
 1/3  1/3
V V
r= and height h = 2 = 2r.
2π 2π

185 MATH 122: Calculus I - Kojoga Aziedu


Example 5.35. A cuboid is has its base length as twice as its width. If the volume of the cuboid is
  31
3V
V , show that the internal surface area is minimum when the width is .
4
Solution.

Example 5.36. A horizontal beam of length 2L is supported at its ends. The small downward sag y
due to its weight at any point x from one end of the beam satisfies
1
4Lx3 − 8L3 x − x4 .

y=
24
(a) Show that the maximum sag occurs in the middle of the beam.
(b) What is the maximum sag?

Solution.

Example 5.37. A cylindrical can with a top has a volume V . If it is made from the least amount of
  13
V
material, show that its diameter is equal to its height, and equal to 2 .

Solution.

5.2.8.1 Concavity.

Definition 5.38. Concave Upwards and Concave Downwards.


Suppose f is a twice differentiable function on the open interval (a, b). Then the graph of f is said to
be

• concave upwards on I if f 00 (x) > 0 ∀x ∈ I, and


• concave downwards on I if f 00 (x) < 0 ∀x ∈ I.

If the graph of the function f is concave upwards on the interval I, then all tangents to the curve
y = f (x) lie below the graph for all x ∈ I and if the graph of the function f is concave downwards on
the interval I, then all tangents to the curve y = f (x) lie above the graph for all x ∈ I.
However, if f 00 (x) = 0, then the graph of f is neither concave upwards nor concave downwards on I.

Figure 5.10: Concave upwards and concave downwards

186 MATH 122: Calculus I - Kojoga Aziedu


Example 5.39. If f (x) = 4 − x2 , then f 0 (x) = −2x which has the critical point x = 0. But
f 00 (x) = −2 < 0. Thus the graph of f (x) = 4 − x2 is concave downward on R.

Figure 5.11: Graph of f (x) = 4 − x2 which is concave downward.

Example 5.40. Find the intervals on which the graph of the following functions will be concave
upwards and concave downwards.

(a) f (x) = x3 − 6x2 + 9x + 2. (b) g (x) = 3x5 − 5x3 + 3.

Solution.

(a) From f (x) = x3 − 6x2 + 9x + 2,

f 0 (x) = 3x2 − 12x + 9; f 00 (x) = 6x − 12.

For f to be concave upward,

f 00 (x) > 0 ⇒ 6 (x − 2) > 0 ⇒ x > 2

and concave down when


f 00 (x) < 0 ⇒ 6 (x − 2) < 0 ⇒ x < 2.
Therefore, f is concave upwards on (2, ∞) and concave downwards on (−∞, 2).

187 MATH 122: Calculus I - Kojoga Aziedu


Figure 5.12: Graph of f (x) = x3 − 6x2 + 9x + 2.

(b)
g (x) = 3x5 − 5x3 + 3 ⇒ g 0 (x) = 15x4 − 15x2
and so √  √ 
g 00 (x) = 60x3 − 30x = 30x 2x2 − 1 = 30x

2x + 1 2x − 1 .
Thus, √ √
√  √  2 2
00
g (x) > 0 ⇒ 30x 2x + 1 2x − 1 > 0 ⇒ − < x < 0, x >
2 2
and √ √
√  √  2 2
00
g (x) < 0 ⇒ 30x 2x + 1 2x − 1 > 0 ⇒ x < − , 0<x< .
2 2

√ ! √ ! √ !
2 2 2
Hence, g is concave upward on − ,0 ∪ , ∞ and concave down on −∞, − ∪
2 2 2
√ !
2
0, .
2

188 MATH 122: Calculus I - Kojoga Aziedu


Figure 5.13: Graph of g (x) = 3x5 − 5x3 + 3.

5.2.8.2 Points of Inflexion.

Definition 5.41. Point of Inflexion.


The point of inflexion is that point that separates the concave upward part of the graph of a continuous
function from the concave downward part or vice versa.
At the point of inflexion, x = c, of the function f (x), the following occur:

• f 00 (x) = 0; and

• f 0 (x) changes sign as x moves from one interval into the other, i.e. f 0 (c± ) > 0 or f 0 (c± ) < 0.

• Note that in the neighbourhood of x = c, the following may occur with the associated implications:

 f 00 (c+ ) > 0 and f 00 (c− ) < 0 ⇒ x = c is a point of inflexion.


 f 00 (c+ ) < 0 and f 00 (c− ) > 0 ⇒ x = c is a point of inflexion.
 f 00 (c+ ) > 0 and f 00 (c− ) > 0 ⇒ x = c is not a point of inflexion.
 f 00 (c+ ) < 0 and f 00 (c− ) < 0 ⇒ x = c is not a point of inflexion.

189 MATH 122: Calculus I - Kojoga Aziedu


Figure 5.14: A point of inflexion.

Theorem 5.42. Condition for a Point x = c to be a Point of Inflection.


Suppose x = c is a point of inflexion of the continuous function f (x) on the interval [a, b]. Then

f 00 (c) = 0

or f 00 (c) does not exist, and


f 000 (c) 6= 0.

Proof. mm

Note: The converse of the above theorem is not necessarily true. That is the fact that we can find
values of x for which f 00 (x) = 0 or f 00 (x) does not exist does not exist does not mean that x is a point
of inflexion.

How to find the Points of Inflexion of a Function on a Interval I.

• Find the second derivative of the function and equate it to zero; i.e. set f 00 (x) = 0 and solve for
x ∈ I.

• Also, find those values of x ∈ I for which f 00 (x) is undefined.

• Those values of x as computed above that satisfy f 000 (x) 6= 0 are inflexion points in I.

Example 5.43. Find the points of inflexion of the graphs of each of the following functions.

190 MATH 122: Calculus I - Kojoga Aziedu


(a) f (x) = 6x6 − 25x4 + 9. (b) g (x) = x2/3 − 1. (c) h (x) = 2x3 + x2 − 20x + 4.

Solution.

(a) From f (x) = 6x6 − 25x4 + 9, we have

f 0 (x) = 36x5 − 100x3 ⇒ f 00 (x) = 180x4 − 300x2 = 60x2 3x2 − 5 .




Hence, r
00 2 2
 5
f (x) = 60x 3x − 5 = 0 ⇒ x = 0, x = ± .
3
Now,
f 000 (x) = 720x3 − 600x = 120x 6x2 − 5


and

f 000 (0) = 720 (0)3 − 600 (0) = 0,


 q   q 3  q  q
000 5
f − 3 = 720 − 3 − 600 − 3 = −600 53 6= 0,
5 5

q  q 3 q  q
000 5 5 5 5
f 3
= 720 3
− 600 3
= 600 3
6= 0

Thus the point x = 0 is not a point of inflexion. The points of inflexion are
 q  q   q  q q  q 
5
− 3, f − 3 5
≡ − 3 , − 3 and
5 98 5
3
,f 5
3
≡ 5
3
98
,− 3

Figure 5.15: Graph of f (x) = 6x6 − 25x4 + 9

191 MATH 122: Calculus I - Kojoga Aziedu


(b) Given that g (x) = x2/3 − 1, we have
 
2 2 1 −4/3 2
g (x) = 1/3 ⇒ g 00 (x) =
0
− x = − 4/3 .
3x 3 3 9x
Therefore,
2
g 00 (x) = 0 ⇒ − =0
9x4/3
2
which has no solution in R. But g 00 (x) = − is undefined for x = 0.
9x4/3
Now in the neighbourhood of x = 0, we have

2 2
f 00 (−1) = − 4/3
=− <0
9 (−1) 9
and
2 2
f 00 (1) = − = − < 0.
9 (1)4/3 9
Thus the point (0, f (0)) ≡ (0, −1) is not a point of inflexion. Hence, the function g (x) = x2/3 − 1
has no point of inflexion on R. The graph of g is shown below.

Figure 5.16: Graph of g (x) = x2/3 − 1

(c)
h (x) = 2x3 + x2 − 20x + 4 ⇒ h0 (x) = 6x2 + 2x − 20
and
1
h00 (x) = 12x + 2 = 0 ⇒ x = − .
6
Now since h is defined for all x and
00

h000 (x) = 12 6= 0 ∀x ∈ R,

192 MATH 122: Calculus I - Kojoga Aziedu


we have that − 16 , h − 16 = − 61 , 397 is a point of inflexion of h.
 
54

Figure 5.17: Graph of h (x) = 2x3 + x2 − 20x + 4

Example 5.44. Find all the critical points of the function f (x) = x4 − 4x3 + 10 and classify them.

Solution. From f (x) = x4 − 4x3 + 10, we have

f 0 (x) = 4x3 − 12x2 = 4x2 (x − 3) .

Therefore, the critical points are x = 0 and x = 3.


But
f 00 (x) = 12x2 − 24x = 12x (x − 2) .
Now,
f 00 (0) = 12 (0) (0 − 2) = 0
and
f 00 (3) = 12 (3) (3 − 2) = 36 > 0.
Thus the point (3, f (3)) ≡ (3, −17) is a minimum point while the point (0f (0)) ≡ (0, 10) is neither
maximum nor minimum. However,

f 000 (x) = 24x − 24 ⇒ f 000 (0) = −24 6= 0.

Hence, the point (0f (0)) ≡ (0, 10) is a point of inflexion.

193 MATH 122: Calculus I - Kojoga Aziedu


Figure 5.18: Graph of f (x) = x4 − 4x3 + 10

5.3 Rolle’s Theorem.


Theorem 5.45. Rolle’s Theorem
Let f be a function such that

• f is continuous on the closed interval [a, b];


• f is differentiable on the open interval (a, b); and
• and f (a) = f (b).

Then c ∈ (a, b) such that f 0 (c) = 0.

Figure 5.19: Rolle’s theorem

194 MATH 122: Calculus I - Kojoga Aziedu


Proof. We shall proof this theorem in two parts: when f is constant and when it is not.
Case I: f is a constant function:
Suppose f is a constant function, i.e. f (x) = f (a) ∀x ∈ [a, b], say. Then

f 0 (x) = 0 ∀x ∈ (a, b) ⇒ f 0 (c) = 0 ∵ c ∈ (a, b) .

Case II: f is not a constant function:


Now, suppose f is not constant on [a, b], i.e. ∃x ∈ [a, b] such that f (x) 6= f (a) .
Also, suppose that f (x) > f (a). Then since f is continuous on [a, b], then by the maximum-minimum
theorem (Theorem 3.66), f must have a maximum value at some point c ∈ [a, b]. Thus we can write

f (c) ≥ f (x) > f (a) = f (b) .

Similarly, if f (x) < f (a), then f must have a minimum value at some point c ∈ [a, b] so that

f (c) ≤ f (x) < f (a) = f (b) .

Thus, c can neither be a nor b, i.e. c ∈ (a, b) and so differentiable at c. By Theorem 5.25, c must be a
critical point of f . That is,
f 0 (c) = 0
completing the proof.


Example 5.46. Verify that the function f (x) = x x + 6 on [−6, 0] satisfies the hypothesis of Rolle’s
theorem. Hence, find all numbers c that satisfy the conclusion of the theorem.

Solution. From Theorem 3.3.1.2, f is continuous on [−6, 0] since it is a product of two continuous
functions (polynomial and power functions.)
Also,
√ 1 1
f 0 (x) = x + 6 + x (x + 6)− 2
2
√ x
= x+6+ √ ,
2 x+6

which is defined for all x ∈ (−6, 0) and so exists.


Finally,

f (−6) = −6 −6 + 6 = 0

f (0) = (0) 0 + 6 = 0;

i.e.
f (−6) = f (0) = 0.
Hence, the hypothesis of Rolle’s theorem is satisfied. By conclusion of the theorem, there must exist
some numbers c ∈ (−6, 0) such that f 0 (c) = 0. That is
√ c 2 (c + 6) + c
f 0 (c) = c+6+ √ = √ =0
2 c+6 2 c+6
or
3c + 12 = 0 ⇒ c = −4 ∈ (−6, 0) .

195 MATH 122: Calculus I - Kojoga Aziedu



Figure 5.20: Verification of Rolle’s Theorem for the function f (x) = x x + 6 on [−6, 0].

Example 5.47. m

Solution. m

5.3.1 Applications of the Rolle’s Theorem.


5.3.1.1 Proof of Uniqueness of Roots of Equations in Given Intervals.

.
.
.
Example 5.48. Prove that the equation x3 + x − 1 = 0 has exactly one root in [0, 1].

Solution. Let f (x) = x3 + x − 1, then


f (0) = −1 < 0
and
f (1) = 1 > 0.
But f is a continuous function on [0, 1] since it is a polynomial. Hence, by the IVT
∃c ∈ (0, 1) : f (c) = 0.
That is the given equation has a root in [0, 1]. To show that there is no other real root on [0, 1], we use
Rolle’s theorem and argue by contradiction.
Suppose f has two real roots a and b in [0, 1]. Then f (a) = 0 = f (b). But f being a polynomial means
it is differentiable (a, b) and continuous on [a, b]. Thus by Rolle’s theorem,
∃c ∈ (a, b) : f 0 (c) = 0.
But
f 0 (x) = 3x2 + 1 ≥ 1 ∀x ∈ R ⊇ (a, b) .
That is f 0 (x) 6= 0 ∀ ∈ (a, b) , which contradicts the assertion that a, b are roots ⇒ ∃c ∈ (a, b) : f 0 (c) = 0.
Hence, f (x) cannot have two real roots.

196 MATH 122: Calculus I - Kojoga Aziedu


Example 5.49. Show that the equation x5 + 4x = 1 has exactly one solution.

Solution.

5.3.1.2 Proof of the Mean-value Theorem.

The Rolle’s theorem is used to prove the Mean-value theorem (MVT), which states that

Theorem 5.50. The Mean Value Theorem.

Let f be a continuous function on [a, b] and differentiable on (a, b). Then there is a number c in (a, b)
such that
f (b) − f (a)
f 0 (c) = ,
b−a
or
f (b) − f (a) = (b − a) f 0 (c) .

We can use the MVT, to among other things,

• to determine the monotonicity;

• to determine the intervals of increase and/or decrease;

• to proof the positivity or negativity;

• establish the validity of inequalities (involving)

of continuously differentiable functions. We shall look at the MVT in detail in MATH 223: Calculus
II.

5.4 Taylor Polynomials.


Definition 5.51. Taylor’s Polynomials.

Let f be a function defined on [a, b] such that f (n) (x) is continuous on [a, b] and f (n) (x) is differentiable
on (a, b). Then ∀x ∈ [a, b], the Taylor’s polynomial of the function f (x) is given by
n
X f (k) (a)
f (x) = (x − a)k + Rn (x) (5.14)
k=0
k!

or
f 00 (a) (x − a)2 f (n) (a) (x − a)n
f (x) = f (a) + f 0 (a) (x − a) + + ··· + + Rn (x) (5.15)
2! n!
where
f (n+1) (c)
Rn (x) = (x − a)n+1 (5.16)
(n + 1)!

for some c ∈ (x, a). Therefore, for every function f (x), we have

f (x) = Tn (x) + Rn (x) (5.17)

197 MATH 122: Calculus I - Kojoga Aziedu


where
n
X f (k) (a)
Tn (x) = (x − a)k (5.18)
k=0
k!
f 00 (a) (x − a)2 f (n) (a) (x − a)n
0
= f (a) + f (a) (x − a) + + ··· + (5.19)
2! n!
is called the Taylor polynomial of degree n expanded about the point x = a.
The term Rn (x) is called the Remainder term for some c ∈ (a, b). The remainder term is used to
estimate the error that may occur in using Taylor polynomial to appropriate a given function.

Deductions.

• If x = b, then
n
X f (k) (a)
Tn (b) = (b − a)k (5.20)
k=0
k!
f 00 (a) (b − a)2 f (n) (a) (b − a)n
= f (a) + f 0 (a) (b − a) + + ··· + . (5.21)
2! n!

• If x = a, then
n
X f (k) (a)
Tn (a) = (a − a)k = f (a) . (5.22)
k=0
k!

Thus

 Tn0 (a) = f 0 (a)


 Tn00 (a) = f 00 (a)
 Tn000 (a) = f 000 (a)
..
.
(n−1)
 Tn (a) = f (n−1) (a)
(n)
 Tn (a) = f (n) (a).

• Let n = 0. Then
0
X f (k) (a) f 0 (c)
f (x) = T0 + R1 (x) = (x − a)k + (x − a)
k=0
k! 1!
= f (a) + f 0 (c) (x − a)

or
f (x) − f (a)
f 0 (c) =
x−a
for some c ∈ (a, x). This is the Mean-value Theorem (see Theorem 5.50 above); a special case of
Taylor’s Theorem.

• Let ε ∈ (0, 1) we have

0 < ε (b − a) < b − a ⇒ a < a + ε (b − a) < b ≡ a < a + εh < b

198 MATH 122: Calculus I - Kojoga Aziedu


where h = b − a ⇒ b = a + h. Let c = a + ε (b − a) = a + εh. Therefore, from (5.20), we have
n
X f (k) (a)
Tn (a + h) = hk (5.23)
k=0
k!
f 00 (a) 2 f (n) (a) n
= f (a) + hf 0 (a) + h + ··· + h (5.24)
2! n!
and
f (n+1) (a + εh)
Rn (x) = (x − a)n+1 . (5.25)
(n + 1)!

Fact. One useful consequence of Taylor’s Theorem is that

|x − c|n+1
(5.26)

|Rn (x)| = max f (n+1) (c)
(n + 1)!

where max f (n+1) (c) is the maximum value of f (n+1) (c) between x and a.

• If a = 0, then c = εh and
n
X f (k) (0)
Tn (x) = xk (5.27)
k=0
k!
f 00 (0) 2 f (n) (0) n
= f (0) + xf 0 (0) + x + ··· + x (5.28)
2! n!
and
f (n+1) (εh) n+1
Rn (x) = x . (5.29)
(n + 1)!
This special case (a = 0) of Taylor’s polynomial is called the Maclaurin’s polynomial expansion of
f (x).

• If f (x) is a polynomial of degree r < n, then f (n) (x) = 0. Hence

f 00 (0) 2 f (r) (0) n


f (x) = = f (0) + xf 0 (0) + x + ··· + x .
2! n!

• Since Taylor’s theorem is to approximate functions by polynomials, we have the following:

 The linear approximation of the function f (x) is

T1 (x) = f (a) + f 0 (a) x. (5.30)

 The quadratic approximation of the function f (x) is


1 00
T2 (x) = f (a) + f 0 (a) x + f (a) x2 . (5.31)
2!
 The cubic polynomial approximation of the function f (x) is
1 00 1
T3 (x) = f (a) + f 0 (a) x + f (a) x2 + f 000 (a) x3 , (5.32)
2! 3!
etc.

Example 5.52. Find the Taylor polynomial of degree n for the following functions.

199 MATH 122: Calculus I - Kojoga Aziedu


(a) f (x) = x3 − 2x + 1 about x = 12 . (b) g (x) = ln x about x = 1.

Solution.

(a) Given f (x) = x3 − 2x + 1, we have

f 0 (x) = 3x2 − 2
f 00 (x) = 6x
f 000 (x) = 6.

Therefore, the Taylor polynomial expansion of the function f (x) = x3 − 2x + 1 about the point
1
x = is
2
3 k
f (k) 12

X 1
f (x) = x−
k=0
k! 2
00 1 1 2 000 1 1 3
   
− −
    
1 1 1 f x f x
= f + f0 x− + 2 2
+ 2 2
2 2 2 2! 3!
 3 !  2 ! 2 3
6 12 x − 12 6 x − 12
   
1 1 1 1
= −2 +1 + 3 −2 x− + +
2 2 2 2 2 6
   2  3
1 5 1 3 1 1
= − x− + x− + x− .
8 4 2 2 2 2

Therefore,    2  3
1 5 1 3 1 1
3
x − 2x + 1 ≡ − x− + x− + x− . (5.33)
8 4 2 2 2 2
Hence, the Taylor polynomial for f (x) = x3 − 2x + 1 about the point x = 1
2
orders 1 and 2 are
 
1 5 1
T1 (x) = − x−
8 4 2

and    2
1 5 1 3 1
T2 (x) = − x− + x−
8 4 2 2 2
respectively. The graphs of f , T1 and T2 are sketched below.

200 MATH 122: Calculus I - Kojoga Aziedu


Figure 5.21: The Taylor polynomials of orders 1 and 2 for the function f (x) = x3 − 2x + 1 about the
point x = 21 .

Exercise. Verify the identity (5.33).

(b) Recall from equation (4.47) that if g (x) = ln x, then

(−1)n−1 (n − 1)!
g (n) (x) = , n = 1, 2, 3, · · · .
xn
Thus,

(−1)1−1 (1 − 1)!
g 0 (1) = =1
1
2−1
(−1) (2 − 1)!
g 00 (1) = = −1
12
3−1
(−1) (3 − 1)!
g 000 (1) = =2
13
4−1
(−1) (4 − 1)!
g (4) (1) = = −6
14
5−1
(−1) (5 − 1)!
g (5) (1) = = 24
15
6−1
(−1) (6 − 1)!
g (6) (1) = = 120
16
..
.
(−1)(n−1)−1 ((n − 1) − 1)!
g (n−1) (1) = n−1 = (−1)n−2 (n − 2)!
(1)
n−1
(−1) (n − 1)!
g (n) (1) = = (−1)n−1 (n − 1)!.
(1)n

And since g (1) = ln (1) = 0, we have that the Taylor approximation of order n to g (x) = ln x

201 MATH 122: Calculus I - Kojoga Aziedu


about x = 1 is given by
3
X g (k) (1)
g (x) = (x − 1)k
k=0
k!
(x − 1)2 g (1) (x − 1)3
000
g 00 1
g (n) (1) (x − 1)3

0 2
= g (1) + g (1) (x − 1) + + + ··· +
2! 3! n!
1 2 6 24
= 0 + 1 (x − 1) − (x − 1)2 + (x − 1)3 − (x − 1)4 + (x − 1)5 +
2 6 24 120
(−1)n−2 (n − 2)! (−1)n−1 (n − 1)!
··· + (x − 1)n−1 + (x − 1)n
(n − 1)! n!
1 1 1 1
= (x − 1) − (x − 1)2 + (x − 1)3 − (x − 1)4 + (x − 1)5 +
2 3 4 6
1 n−2 n−1 1 n−1
··· + (−1) (x − 1) + (−1) (n − 1)! (x − 1)n
n−1 n
n r−1
X (−1) (r − 1)!
= (x − 1)n .
r=0
r

Hence, the Taylor polynomial approximations of orders 1, 2, 3 and 4 of g (x) = ln x at x = 1 are

T1 (x) = (x − 1) ,
1
T2 (x) = (x − 1) − (x − 1)2
2
1 1
T3 (x) = (x − 1) − (x − 1)2 + (x − 1)3
2 3
and
1 1 1
T4 (x) = (x − 1) − (x − 1)2 + (x − 1)3 − (x − 1)4
2 3 4
respectively. The graphs of g, and T1 , T2 , T3 and T4 are sketched in the diagram below.

Figure 5.22: The Taylor polynomials of orders 1, 2, 3 and 4 for the function g (x) = ln x about x = 1.

Example 5.53. Find the Maclaurin’s polynomial of degree n for the following functions.

202 MATH 122: Calculus I - Kojoga Aziedu


(a) f (x) = ex . 1
(b) g (x) = .
1−x

Solution.

(a) Notice that, given f (x) = ex , then for all n ∈ N,

f (n) (ex ) = ex .

So
f (n) (0) = e0 = 1 ∀n ∈ N.
Hence, the Maclaurin’s polynomial expansion of f (x) = ex is
n
X f (k) (0)
f (x) = xk
k=0
k!
000 000 000
0 f 00 (0) x2 f (0) x3 f (0) x4 f (0) xn
= f (0) + f (0) x + + + + ··· +
2! 3! 4! n!
x2 x3 x4 xn
= 1+x+ + + + ··· + .
2 6 24 n!
Therefore,
n
X xk
x
e = + Rn (x) (5.34)
k=0
k!
where
f (n+1) (c) n+1
Rn (x) = x
(n + 1)!

where c ∈ (0, x).

(b) Recall from equation (4.39) that

dn
 
1 n!
= , n∈N
dxn 1−x (1 − x)n+1

and so
n!
g (n) (0) = = n!
(1 − 0)n+1
Therefore, the Maclaurin’s polynomial expansion of f (x) = ex is
n n n
X g (k) (0) X k! X
g (x) = xk = xk = xk
k=0
k! k=0
k! k=0
2 3 4 n
= 1 + x + x + x + x + ··· + x .

Hence,
n
1 X
= xk + Rn (x) (5.35)
1 − x k=0
where
f (n+1) (c) n+1
Rn (x) = x
(n + 1)!

where c ∈ (0, x).

203 MATH 122: Calculus I - Kojoga Aziedu


5.4.1 Maclaurin’s Expansions for Some Standard Functions.
• ex =

• sin x =

• cos x =

• tan x =

• ln x =
1
• =
1−x
1
• =
1+x
• sin x =

• sin x =

• sin x =

• sin x =

5.5 Linear Approximations and Differentials.

5.5.1 Linear Approximations.


Given the function, f (x), its tangent line at the point x = a has the equation

L (x) = f (a) + f 0 (a) (x − a) .

Figure 5.23: Linear approximation of f (x)

204 MATH 122: Calculus I - Kojoga Aziedu


From the graph above, the tangent line y = L (x) in the neighbourhood of x = a nearly approximates
the graph of the function y = f (x) .Thus we can use the tangent line, L (x), as an approximation to
the function, f (x), near x = a.

Definition 5.54. Linear Approximation of a Function at a Point.


Given the function y = f (x), the tangent line

L (x) = f (a) + f 0 (a) (x − a)

is called the linear approximation to the function at x = a.

Example 5.55. Determine the linear approximation √ for the √


function f (x) = x1/3 at x = 8. Use the
linear approximation to approximate the value of 3 8.05 and 3 25.

2 −(2/3) 1
Solution. First of all, f (x) = x1/3 ⇒ f 0 (x) = x = √
3
. Hence, the linear approximation of
3 3 x2
f (x) at the point x = 8 is
1 √
L (x) = f (8) + f 0 (8) (x − 8) =
3
8+ √
3
(x − 8)
3 82
1 1 4
= 2+ (x − 8) = x + .
12 12 3
√ √
Hence, the approximate values of 3 8.05 and 3 25 are
1 4 481
L (8.05) = (8.05) + = = 2.004166667
12 3 240
and
1 4 41
L (25) = (25) + = ≈ 3.416666667.
12 3 12
respectively.


Figure 5.24: Linear approximation of f (x) = 3
x at x = 8

Note: √
3
8.05 = 2.004158016; L (8.05) = 2.004166667

205 MATH 122: Calculus I - Kojoga Aziedu


and √
3
25 = 2.924017738; L (25) = 3.416666667.
√ √
Thus L (8.05) gives a better approximation of 8.05 than 25 since x = 8.05 is nearer to x = 8 than
3 3

x = 25 is.

Example 5.56. Determine the linear approximation for sin θ at θ = 0.

Solution. Let f (θ) = sin θ ⇒ f 0 (θ) = cos θ. Therefore, the linear approximation for f (θ) = sin θ at
θ = 0 is

L (θ) = f (0) + f 0 (0) (θ − 0)


= sin (0) + cos (0) (θ) = θ.

So, for small θ we have that


sin θ ≈ θ.

Theorem 5.57. Error in Linearisation.


Adams pg 269

Proof. mm

Example 5.58. m

Solution.

5.5.2 Differentials.
Let ∆f denote the change in the function f as a result of a small change, ∆x, in x. That is

∆f = f (x + ∆x) − f (x) .

Suppose that f is differentiable at x. Then

f (x + ∆x) − f (x) ∆f
f 0 (x) = lim = lim
∆x→0 ∆x ∆x→0 ∆x

so that
∆f
= f 0 (x) + ε∆x
∆x
where ε → 0 as ∆x → 0. Hence,

∆f = f 0 (x) ∆x + ε∆x · ∆x = f 0 (x) ∆x + ε (∆x)2 .

Now, if ∆x, then ε (∆x)2 is even smaller. Therefore,

∆f ≈ f 0 (x) ∆x.

This is called the incremental approximation of f (x).

206 MATH 122: Calculus I - Kojoga Aziedu


Definition 5.59. Differentials.
Let ∆f ≈ f 0 (x) ∆x be incremental approximation of f (x). Then if ∆x 6= 0, we have

∆f dy
f 0 (x) = lim :=
∆x→0 ∆x dx
or
df = f 0 (x) dx. (5.36)
The quantity df is called the differential of f at x.

Figure 5.25: The differential of the function f

Example 5.60. Compute the differential for each of the following functions.

(a) y = x3 − 4x2 + 7x (b) z = x2 sin (2x) (c) w = e3−z .


4

Solution. df = f 0 (x) dx

(a)

y = x3 − 4x2 + 7x
⇒ dy = 3x2 − 8x + 7 dx.


(b)

z = x2 sin (2x)
⇒ dz = 2x sin (2x) − 2x2 cos (2x) dx.


(c)
4
w = e3−z
4
⇒ dw = −4z 3 e3−z dx.

Example 5.61. Find the approximate value of sin (45◦ 20 ).

207 MATH 122: Calculus I - Kojoga Aziedu


Solution.   ◦ 
◦ 0 ◦ 2 π π 
sin (45 2 ) = sin 45 + = sin + .
60 4 5400
Now, let
π
f (x) = sin x ⇒ f 0 (x) = cos x, ∆x =
5400
so that
∆f = f (x + ∆x) − f (x) = f 0 (x) ∆x
⇒ f (x + ∆x) = f (x) + f 0 (x) ∆x.
That is
π  π  π
◦ 0
sin (45 2 ) = sin + cos · .
4 4 5400
1 1 π
= √ +√ ·
2 2 5400

 
π 1
= 2 + ≈ 0.7075.
10800 2

Example 5.62. Use calculus to find an approximate value of (4.01)3/2 .

Solution. Notice that (4.01)3/2 = (4 + 0.01)3/2 . Let


f (x + ∆x) = (4 + 0.01)3/2
so that √
f (x) = x3/2 ⇒ f 0 (x) = 3
2
x, ∆x = 0.01.
But
f (x + ∆x) − f (x) = f 0 (x) ∆x
⇒ f (x + ∆x) = f (x) + f 0 (x) ∆x.
Hence,
(4.01)3/2 = f (4 + 0.01)
 √ 
0 3/2
= f (4) + f (4) ∆x = (4) + 32 4 (0.01)
3 1
= 8+ ·2· = 8.03.
2 100

Example 5.63. Compute ∆y and dy if y = cos (x2 + 1) − x as x changes from 2 to 2.03.

Solution.
∆y = y (2.03) − y (2)
= cos (2.03)2 + 1 − 2.03 − cos (2)2 + 1 − 2 = cos (5.1209) − cos (5)
  

= 0.083581127.
Now,
dy = f 0 (x) dx = −2x sin x2 + 1 − 1 dx.
 

Next, the change in x from x = 2 to x = 2.03 is ∆x = 0.03. Assume that dx ≈ ∆x = 0.03. Then the
approximate change in y, ∆y, is
dy = f 0 (x) dx = −2 (2) sin 22 + 1 − 1 (0.03) = 0.085070913.
 

208 MATH 122: Calculus I - Kojoga Aziedu


Example 5.64. A sphere was measured and its radius was found to be 45 inches with a possible error
of no more that 0.01 inches. What is the maximum possible error in the volume if we use this value of
the radius?

Solution. The equation for the volume of a sphere of radius r is


4
V = πr3 .
3
Now, with r = 45, the actual volume of the sphere is
4
V = π (45)3 = 121500π in3 .
3
Also, with dr ≈ ∆x = 0.01, dV ≈ ∆V should give us maximum error. But

dV = 4πr2 dr,

thus
∆V ≈ dV = 4π (45)2 (0.01) = 81π in3 .
Hence, the percentage error in the volume as the result of the 0.01 in error in the radius is
81π 1
× 100% = % ≈ 0.067%.
121500π 15

5.6 Rates of Change.


Definition 5.65. Rate of Change.

• A rate of change is that quantity describes how one quantity changes in relation to another
quantity.
• Mathematically, rate of change R in the quantity Y in relation to the quantity X is given by
change in Y
R=
change in X

• Rates of change can be positive (when both quantities increase/decrease together) or negative
(when one quantity increases the other decrease at the same time).
• When a quantity does not change over time, it is said to be constant, and the rate of change is
zero.
• Average rate of change the change in the value of a quantity divided by the elapsed time.

 For a function y = f (x), average rate of change is the slope of the secant line S that passes
through two distint points A (a) and B (b),
f (b) − f (a)
R= .
b−a
• The rate of change at a particular instant is called the instantaneous rate of change at that point
P (p).

 For a function y = f (x), the instantaneous rate of change, which is usually referred to as
the rate of change, is given by

0 f (x) − f (p) f (p + h) − f (p) dy
R = f (p) = lim = lim = .
x→p x−p h→0 h dx x=p

209 MATH 122: Calculus I - Kojoga Aziedu


 This rate is the slope of the tangent line L to the curve y = f (x).

Figure 5.26: Average and instantaneous rates of change

• The (average or instantaneous) rate of change of height or distance, s, with respect to time is
called velocity, v.
Change in distance s2 − s1
 Average velocity = v̄ = = .
Change in time t2 − t1
 Instantaneous velocity at time t = p

ds s (t) − s (p) s (p + h) − s (p)
v (p) = = lim = lim .
dt t=p t→p
t−p h→0 h

 Speed is the magnitude of velocity.

• Another rate of change we usually encounter is relative rate of change.

 Relative rate of change compares the absolute change in a quantity to a reference value,
usually called the initial population.
 Relative rate of change describes the actual increase or decrease from a reference (or old/earlier)
value to a new (or later) value.
 Thus
Absolute change
Relative change =
Reference value
or
New value − Reference value
Relative change =
Reference value
where
Absolute change = New value − Reference value.
 When a quantity P changes from P0 to P1 , the relative change in P is defined as
Change in P P1 − P0
Relative change in P = = .
P0 P0
 Relative change is a number, without unit; it is often expressed as a percentage or fraction.

Example 5.66. An object is dropped from a tall building, then the d distance it has fallen after t
seconds is given by the function d(t) = 16t2 . Find its average speed (average rate of change) over the
following intervals:

210 MATH 122: Calculus I - Kojoga Aziedu


(a) between 1 s and 5 s (b) between t = a and t = a + h.

Solution. The distance fallen by the object as a function of time is given by d(t) = 16t2 . Thus

(a) Average rate of change between 1 s and 5 s is

d (5) − d (1) 16 (5)2 − 16 (1)2


Average rate of change = =
5−1 5−1
400 − 16
= = 96 ft/s.
4

(b) Average rate of change between t = a and t = a + h is

d (a + h) − d (a) 16 (a + h)2 − 16 (a)2


Average rate of change = =
a+h−a a+h−a
16 (a2 + 2ah + h2 ) − 16a2 16a2 + 32ah + 16h2 − 16a2
= =
h h
32ah + 16h2
= = (32a + 16h) = 16 (2a + h) ft/s.
h

Example 5.67. Let f (x) = mx + b. Find the average rate of change of f between the points x = a
and x = a + h.

Solution. Given f (x) = mx + b, we have the average rate of change between x = a and x = a + h is

f (a + h) − f (a) m (a + h) + b − (ma + b)
Average rate of change = =
a+h−a a+h−a
ma + mh + b − ma − b mh
= = = m.
h h
Example 5.68. A car starting from a dead stop is s (t) = t2 feet from the starting point t seconds
after it begins to move. What is the velocity of the car 20 seconds after it begins its journey? How
long does it take for the car to reach a speed of 60 ft/hr? Of 80 ft/hr?

Solution. The velocity of the car at time 20 s is

s (20 + h) − s (20) (20 + h)2 − (20)2


v (20) = lim = lim
h→0 h h→0 h
2 2 2
(20) + 40h + h − (20) 40h + h2
= lim = lim
h→0 h h→0 h
= lim (40 + h) = 40 + 0 = 20 ft/hr.
h→0

Also, the speed of the car at any time t is given by


ds d 2
v (t) = = t = 2t.
dt dt
∴ v (t) = 60 ⇒ 2t = 60 ⇒ 30 s
and
v (t) = 80 ⇒ 2t = 80 ⇒ 40 s.

211 MATH 122: Calculus I - Kojoga Aziedu


Example 5.69. Find all points on the graph of f (x) = 3x3 + x2 + 1 where the instantaneous rate of
change is 1.

Solution. The instantaneous rate of change of f at the point x is


d
f 0 (x) = 3x3 + x2 + 1 = 9x2 + 2x.

dx
Now, given that f 0 (x) = 1, we have

9x2 + 2x = 1 ⇒ 9x2 + 2x − 1 = 0

or
    " 2 #
2 1
2 2 2 1 1 1 1 10
0 = 9 x + x− =9 x + x+ − − =9 x+ −
9 9 9 81 81 9 9 81

i.e.  2 r √ √
1 10 1 10 1 10 −1 ± 10
x+ = ⇒x+ =± ⇒x=− ± = .
9 81 9 81 9 9 9
 √  √ 
Therefore, the points on f for which the instantaneous rate of change are −1− 10
9
,f −1− 10
9
and
 √  √ 
−1+ 10
9
, f −1+9 10 .

Example 5.70. Suppose the population of a town was 10,000 in 1970 and 15,000 in 2000 . Find the
absolute change and the relative change.

Solution. Absolute change in population = 15, 000 − 10, 000 = 5, 000.


5, 000
Relative change in population = × 100% = 50%.
10, 000
Therefore there is an increase in the 1970 population by 50% in the year 2000.

Example 5.71. The annual average resident unemployment rate was 4.3% in 2009 and 3.1% in 2010.
Determine the relative change.

Solution. Absolute change in the annual unemployment rate = (3.1 − 4.3) % = −1.2%.
−1.2%
Relative change in the annual unemployment rate = × 100% = −27.9%.
4.3%
Thus there is a drop by 27.9% in the unemployment rate from the 2009 figure.

Example 5.72. The volume V of water in a reservoir at time t is given by


π 
V = 4 + 3 sin t .
6

(a) Find the volume of water in the reservoir at time t = 3.

(b) Find the rate of change of the volume of the water at time t = 3.

Solution.

212 MATH 122: Calculus I - Kojoga Aziedu


(a) The volume of water in the reservoir at time t = 3 is
π 
V (3) = 4 + 3 sin (3) = 4 + 3 sin (2π) = 4 + 0 = 3
6

(b) The rate of change of the volume of the water at any time t is

dV π π 
= cos t .
dt 2 6
At time t = 3, the rate of change of water volume in the reservoir is

dV π π  π π
= cos (3) = cos (2π) = .
dt t=3
2 6 2 2

Example 5.73. The total quantity Q of elements in a container at any time t is given by
π 
Q = 5πt + 4 sin t .
2
Find the rate at which Q is changing at time t = 4.

Solution. The rate of change of Q with respect to t is


dQ d h  π i π 
= 5πt + 4 sin t = 5π + 2π cos t
dt dt 2 2
dQ π 
∴ = 5π + 2π cos (4) = 5π + 2π cos (2π)
dt t=4 2
= 5π + 2π = 7π.

Example 5.74. An excavator removes V m3 of soil at time t minutes, where

t2
V = 25t − .
50
Determing the rate of removing the soil after 5 minutes.

t2
Solution. Given the volume of sand as V = 25t − 50
at any time t, we have

dV t
= 25 − .
dt 25
After 5 minutes, the rate of removing the soil stands at
5
25 − = 24.8 m3 /min.
25

5.7 Related Rates of Change: Use of the Chain Rule.


Suppose two or more quantities are related by an equation and one of those quantities changes at a
certain rate. Then we can find the rate at which the related quantities also change. To solve related
rate of change problems, do the following

213 MATH 122: Calculus I - Kojoga Aziedu


• Sketch the object in question and label all the variables and constants therein. If the quantities
are changing with respect to time t, then assume that all the variables are differentiable functions
of t.

• Write down the mathematical relation(s) connecting the variables, noting down what you are
asked to find, usually expressed as a derivative. If there are more that one relations, merge them
into a single one that relates the variable whose rate you want to find to the one whose rate you
already know.

• Differentiate the ensuing relation, using the chain rule where appropriate, and substitute all valid
data at the instant for which the answer to the problem is required.

• Solve for the desired derivative (rate).

Example 5.75. A spherical balloon is inflated in such a way that its radius increases at a rate of
1 cms−1 . How fast is the volume changing when its radius is 12 cm?

Solution. Let r and v denote the radius and volume of the sphere at time t. Then
4
V = πr3 .
3
Differentiating with respect to t, we have
dV 4 d 3  4 d 3  dr
= π r = π r ·
dt 3 dt 3 dr dt
dr
= 4πr2 .
dt
dr
But = 1 cms−1 ∀r. Therefore, when r = 12 cm, we have
dt

dV
= 4π (12)2 (1) = 576π cm3 s−1 .
dt r=12

Example 5.76. A hollow circular cone of height 20 ft with a circular base of radius 5 ft is inverted
and filled with water. Water runs out of the bottom at a constant rate of 2 ft3 min−1 . How fast is the
water level falling when the water is 8 ft deep?

Solution. Let h denote the height of the water level at time t and r the radius of the water at that
time.

Figure 5.27: Circular cone filled with water and water leaking via the inverted apex.

214 MATH 122: Calculus I - Kojoga Aziedu


Then the volume of water in the cone at time t is
1
V = πr2 h.
3
But
r 5 h
= ⇒r= .
h 20 4
Therefore,  2
1 h 1
V = π h = πh3 .
3 4 48
   
dV d 1 3 d 1 3 dh
∴ = πh = πh ·
dt dt 48 dh 48 dt
1 2 dh
= πh .
16 dt
dV
Now, when h = 8, = −2.
dt
1 dh dh 1
−2 = π (8)2 ⇒ =− ft min−1 .
16 dt dt 2π

Example 5.77. Grain is being poured into a heap in the shape of a right circular cone such that its
semi-vertex angle is 68◦ 120 . If the base radius of the cone is increasing at centimetres per second. Find
the rate at which its volume is increasing when its height is 2 centimetres.

Solution.

Example 5.78.(a) Show that the formula for the volume V of a right circular cone of base radius
r and height h is given by
1
V = πh3 tan φ
3
where φ is the semi-vertex angle.

(b) A falling sand forms a heap in the shape of a right circular cone whose semi-vertex angle is 71◦ 330 .
If its height is increasing at 2 metres per second when the heap is 5 metres high, find the rate at
which its volume and surface area is increasing.

Solution.

5.8 Equations of the Tangent and Normal to a Curve.


Definition 5.79. Gradient Function to a Curve.
dy
Given the curve C with equation y = f (x), the derived function f 0 (x) = is called the gradient
dx
dy
function to the curve. The number f 0 (a) = is called the gradient or slope of C at the point
dx x=a
(a, f (a)).
The tangent to the curve at a point is the straight line that touches the curve at that point. The normal
to the curve at that same point is that line that is perpendicular to the tangent at that point.

215 MATH 122: Calculus I - Kojoga Aziedu


Figure 5.28: Tangent and normal lines to a curve at a point.

Fact 5.80. Gradient of Two Perpendicular Lines.


If two straight lines with gradients m and M are perpendicular, then
mM = −1. (5.37)
Therefore, since the gradient and the normal to the perpendicular, if the gradient of the tangent to the
curve y = f (x) at the point (a, f (a)) is f 0 (a) 6= 0, then that of the normal to the curve at that same
1
point is − 0 .
f (a)
Definition 5.81. The Equations of Tangent and Normal to a Curve at a Point.
The equation of the tangent and the normal to the curve y = f (x) at the point (a, f (a)) are
y = f (a) + f 0 (a) (x − a) (5.38)
and
1
y = f (a) − (x − a) (5.39)
f 0 (a)
respectively, where (x, y) are any point on the tangent or the normal.

Example 5.82. A curve given by the equation y = ax4 + bx has a gradient of −2 at the point (1, 1).
Find the values of a and b.

Solution. Given that the point (1, 1) lies on the curve y = ax4 + bx, then
1 = a (1)4 + b (1) ⇒ b = 1 − a. (5.40)
Also, the gradient of y = ax4 + bx is −2 at (1, 1) means

dy
= 4ax3 + b x=1 = −2
dx x=1

⇒ b = −2 − 4a. (5.41)
Solving (5.40) and (5.41) simultaneously, we have
1 − a = −2 − 4a ⇒ 3a = −3 ⇒ a = −1
and so
b = 1 − (−1) = 2.

216 MATH 122: Calculus I - Kojoga Aziedu


Example 5.83. Find the equations of the tangent and normal of the following curves at the given
points.

217 MATH 122: Calculus I - Kojoga Aziedu


√ 
(a) y = x2 at (1, 1). (c) x2 + 2xy + 3y 2 = 9 at 2, 5 .
5
(b) f (x) = (5x3 − 7x + 1) at x = 0. (d) x = 1 + 3t, y = 2 − t2 at t = 1.

Solution.

(a) Given y = x2 , its gradient function is


dy
= 2x.
dx
Therefore, the gradient of the tangent to the curve at (1, 1) is

dy
= 2 (1) = 2
dx x=1

1
and that of the normal is − . Hence, the equation of the tangent and normal to the curve at
2
(1, 1) are
y − 1 = 2 (x − 1) ⇒ y = 2x − 1
and
1 1 3
y − 1 = − (x − 1) ⇒ y = − x +
2 2 2
respectively.

Figure 5.29: Tangent and normal lines to the curve y = x2 at the point (1, 1)

5
(b) The gradient of the tangent to the curve f (x) = (5x3 − 7x + 1) at x = 0 is given by
4
f 0 (x)|x=0 = 5 15x2 − 7 5x3 − 7x + 1

x=0
= 5 (0 − 7) (0 − 0 + 1)4 = −35.
1 5
The corresponding gradient of the normal is . Now, f (0) = 5 (0)3 − 7 (0) + 1 = 1. Hence
35
the equations of the tangent and normal to the curve y = f (x) are

y − 1 = −35 (x − 0) ⇒ y = −35x + 1

218 MATH 122: Calculus I - Kojoga Aziedu


and
1 1
y−1= (x − 0) ⇒ y = x + 1
35 35
respectively.

(c) The gradient function of the curve x2 + 2xy + 3y 2 = 9 is given as


x+y
2x + 2y + 2xy 0 + 6yy 0 = 0 ⇒ y 0 = − .
x + 3y
√ 
That is the gradient of the tangent to the curve at 2, 5 is
√ √
2+ 5 11 + 4 5
− √ =−
2+3 5 41
1 √
and that of the normal is √ = −11 + 4 5. Hence, their equations are
11+4 5
41

√ 11 + 4 5
y− 5=− (x − 2)
41
or  √  √
11 + 4 5 x + 41y = 22 + 49 5;
and √ √
y− 5 = −11 + 4 5 (x − 2)
or  √  √
11 − 4 5 x + y + 11 − 7 5 = 0
respectively.

(d) Given the parametrically defined function, x = 1 + 3t, y = 2 − t2 , we have


dx
dx dy dy dt −2t
= 3; = −2t ⇒ = dx
= .
dt dt dx dt
3

When t = 1, x = 1 + 3 (1) = 4 and y = 2 − (1)2 = 1. So the gradient of the tangent to the curve
is
dy −2 (1) 2
= =−
dx x=4
3 3
and that of the corresponding normal is 32 . Thus the equations of the tangent and the normal at
the point (4, 1) are
2 2 5
y − 1 = − (x − 4) ⇒ y = − x +
3 3 3
and
3 3
y − 1 = (x − 4) ⇒ y = x − 5.
2 2

219 MATH 122: Calculus I - Kojoga Aziedu


Figure 5.30:

Example 5.84. Find the equation of the tangent to the curve y = x (x − 2) (x − 4) at the origin, and
show that this tangent is parallel to the tangent at the point where x = 4.

Solution. Given the curve y = x (x − 2) (x − 4), its gradient function is

dy
= (x − 2) (x − 4) + x (x − 4) + x (x − 2)
dx
and so the gradient of the tangent to the curve at the origin, i.e. (0, 0), is

dy
= (0 − 2) (0 − 4) + (0) (0 − 4) + (0) (0 − 2) = −2 (−4) = 8.
dx 0

Hence, the equation of the tangent at the origin is

y − 0 = 8 (x − 0) ⇒ y = 8x.

Now, at the point where x = 4, the gradient of the tangent to the curve there is

dy
= (4 − 2) (4 − 4) + (4) (4 − 4) + (4) (4 − 2) = 4 (2) = 8.
dx 4

dy
= .
dx 0

Thus gradient of the tangents to the curve at the points where x = 0 and x = 4 are equal and so are parallel.
The equation of the tangent at the point (4, 0) is

y − 0 − 8 (x − 4) ⇒ y = 8x − 32.

220 MATH 122: Calculus I - Kojoga Aziedu


Figure 5.31: Tangent lines to the curve y = x (x − 2) (x − 4) at the points (0, 0) and (4, 0).

dy
Example 5.85. A curve C is defined parametrically by x = 3t − t3 , y = 1 − t2 for all t ∈ R. Find
dx
in terms of t and obtain the equation of the normal to the curve at the point with parameter t. Hence,
find the points on the curve at with the normal passes through the origin.

Solution. From x = 3t − t3 , y = 1 − t2 , we have


dx dy
= ẋ = 3 − 3t2 , = ẏ = −2t
dt dt
and so
dy ẏ 2t
= =− , t 6= ±1
dx ẋ 3 (1 − t2 )
is the gradient of the curve at the point P (t). Thus the gradient of the normal to the curve at this
point is
ẋ 3 (1 − t2 )
m=− = , t 6= 0.
ẏ 2t
Hence, the equation of this normal at P (t) is
 3 (1 − t2 )
y − 1 − t2 = x − 3t − t3 .

2t
But the normal passes through the origin when x = 0, y = 0, i.e. when
 3 (1 − t2 )
1 − t2 = 3t − t3

2t
or
2t 1 − t2 = 3 1 − t2 3t − t3 ⇒ t 1 − t2 3 3 − t2 − 2 = t 1 − t2 3t2 + 7 = 0.
       

And since (3t2 + 7) 6= 0 ∀t ∈ R, we must have that t = 0 or 1 − t2 = 0 ⇒ t = ±1. Hence, when t = 0,

x = 3 (0) − (0)3 = 0, y = 1 − (0)2 = 1,

221 MATH 122: Calculus I - Kojoga Aziedu


when t = 1,
x = 3 (1) − (1)3 = 2, y = 1 − (1)2 = 0
and when t = −1,
x = 3 (−1) − (−1)3 = −2, y = 1 − (−1)2 = 0.
Hence, find the points on the curve at with the normal passes through the origin are (0, 1), (−2, 0) and
(2, 0).

Example 5.86. If the tangent at a variable point P on the curve x = t2 , y = 2t3 meets the curve again
at Q, show that the midpoint of P Q describes the curve 8x = 5t2 , 8y = 7t3 .
Solution. Given curve x = t2 , y = 2t3 , its gradient at point P (t) is
dy dy dx 6t2
= ÷ = = 3t
dx dt dt 2t
for t 6= 0. Therefore, the equation of the tangent to the curve at P (t) is
y − 2t3 = 3t x − t2


or
3tx − y − t3 = 0.
Let T be the parameter corresponding to the point Q on the curve, so that x = T 2 , y = 2T 3 . Then
since the tangent meets at the curve at Q (T ), we have
t
3tT 2 − 2T 3 − t3 = (T − t) (2T + t) = 0 ⇒ T = t, T = − .
2
Hence, the coordinates of the point Q are
 2  3
t t
x= − , y=2 −
2 2
or
t2 t3
x= , y=− .
4 4
The coordinate of the midpoint of P Q are therefore
1 2 t2 5t 7t 3
  2
t3
 
3
(x, y) = t + , 2t − = , .
2 4 4 8 8
That is,
5t2 7t 3
x= , y=
8 8
or
8x = 5t2 , 8y = 7t3
as required.

Example 5.87. Find the coordinates of the points on the curve y = x3 − 3x2 − 3x + 1 at which the
normals are perpendicular to the line 6y + x = 5.
Solution. The gradient function of the tangent to the curve y = x3 − 3x2 − 3x + 1 is given by
y 0 = 3x2 − 6x − 3 = 3 x2 − 2x − 1 .


1
Hence, the normals to the curve at any point (x, y) have gradients − .
3 (x2
− 2x − 1)
Now, the gradient of the line 6y + x = 5 ⇒ y = − 61 x + 56 is − 16 . Therefore, if the normals to the curve
at (x, y) are perpendicular to the line 6y + x = 5, then
1
− 2
= 6 ⇒ 18x2 − 36x − 18 = −1 ⇒ 18x2 − 36x − 17 = 0
3 (x − 2x − 1)

which has solution x = 1± 61 70. Hence the normals to the curve y = x3 −3x2 −3x+1 are perpendicular

to the line 6y + x = 5 at the points where x = 1 ± 16 70.

222 MATH 122: Calculus I - Kojoga Aziedu


5.9 Curves Sketching

5.9.1 Curve Sketching of Rational Functions.


To sketch the graph of a the function y = f (x), obtain all or some of the following information.

• Determine the domain of f .

• Find the intercepts on both axes.

 x-intercept: solve for x such that y = f (x) = 0.


 y-intercept: solve for y such that y = f (0).

• Find the asymptotes, if any.

 Vertical asymptote: The line x = a for which f (a) is undefined.

Example 5.88. Consider the function f defined by


3x − 5
f (x) = .
(x − 1) (x + 2)

Then f is defined for all values of x except when (x − 1) (x + 2) = 0 ⇒ x = −2, 1.


Therefore, the lines x = −2 and x = 1 are the horizontal asymptotes of f .

 Horizontal asymptote.
The line y = b for which
lim f (x) = b.
x→±∞

If f is a rational function of the form

N (x)
f (x) =
D (x)

with N (x) ≤ D (x), then f will have a horizontal asymptote. Specifically,


(
0, if N (x) < D (x)
lim f (x) =
x→±∞ b 6= 0, if N (x) = D (x) .

Example 5.89. Again, for the function


3x − 5 3x − 5
f (x) = = 2 ,
(x − 1) (x + 2) x +x+2

we have
3
− x52
x2 0
lim f (x) = lim 1 2 = = 0.
x→±∞ x→±∞ 1 +
x
+ x2 1
Therefore, the line y = 0 is a horizontal asymptote of f .

223 MATH 122: Calculus I - Kojoga Aziedu


 Oblique asymptote: If an improper rational function of the form

N (x)
f (x) =
D (x)

with N (x) > D (x) and N (x) and D (x) have no common common factors can be expressed
as
R (x)
f (x) = mx + c + ,
D (x)
for some function R (x) < D (x), then the line y = mx + c is called an oblique asymptote
when
R (x)
lim (f (x) − mx − c) = lim = 0.
x→±∞ x→±∞ D (x)

Example 5.90. Consider the function f defined by

x3
f (x) =
x2 + 1
which we can write as
x
f (x) = x −
x2 +1
using partial fractions. Now,
1
x x
lim (f (x) − x) = − lim = − lim = 0.
x→±∞ x→±∞ x2 + 1 x→±∞ 1 + 12
x

Therefore, the lines y = x is an oblique asymptote of f .

• Find the the critical points on the domain.


Recall that the critical numbers occur at the points when f 0 (x) = 0.

 Use the critical to find the local extrema on the domain.

Example 5.91. C

• Determine the intervals of increase and decrease of f .


Recall that f

 increases for f 0 (x) > 0; and


 decreases for f 0 (x) < 0.

• Determine the points of inflexion and concavity of f .


Thus compute f 00 (x) and perform concavity;

 x is a point of inflexion if f 00 (x) = 0 and f 000 (x) 6= 0.


 f is concave upward on I if ∀x ∈ I, f 00 (x) > 0.
 f is concave downward on I if ∀x ∈ I, f 00 (x) < 0.

• Determine the behaviour of f in the neighbourhood of the asymptotes, and as x →


±∞.
Talk about choosing pts in the neighbourhood of ....... and check their signs.

224 MATH 122: Calculus I - Kojoga Aziedu


• Plot the:

 intercepts
 local extrema and/or inflexion points
 asymptotes and any points where f intercepts the horizontal or oblique asymptotes.

• Draw a smooth curve the plotted points.

Example 5.92. Sketch a possible graph of a function f (x) that has the following qualities listed below.
[There may not be a unique graph.]

(a) f (3) = −2 (f) f (1) = 3

(b) f 0 (3) = 0 (g) lim f (x) = 1


x→∞

(c) f 00 (3) > 0 (h) lim f (x) = −1


x→−∞

(d) lim f (x) = ∞ (i) lim f (x) = 1


x→2+
x→0

(e) lim f (x) = ∞ (j) f (0) = −3.


x→2−

Solution. Explain the meaning of the qualities and indicate the coordinate points to be plotted

Example 5.93. Sketch the graph of f (x) = x3 − 6x2 + 9x + 2

Solution.

• Domain: Since f is a polynomial, Df = R.

• Intercepts:

 y-intercept: When x = 0, we have f (0) = (0)3 − 6 (0)2 + 9 (0) + 2 = 2 ⇒ (0, 2)


x-intercept: When f (x) = 0, we have x3 − 6x2 + 9x + 2 ⇒ x ≈ −0.196 = − 250
49 49

 ⇒ − 250 ,0 .

• Critical points and their classification:

 f 0 (x) = 3x2 − 12x + 9 = 3 (x2 − 4x + 3) = 3 (x − 1) (x − 3) = 0 ⇒ x = 1, x = 3. Therefore,


x = 1 and x = 3 are the critical points of f . Now, f 00 (x) = 6x − 12. Hence
 f 00 (1) = 6 (1) − 12 = −6 < 0 ⇒ (1, f (1)) = (1, 6) is a maximum point.
 f 00 (3) = 6 (3) − 12 = 6 > 0 ⇒ (3, f (3)) = (3, 2) is a minimum point.
 Also, f 00 (x) = 6x − 12 = 0 ⇒ x = 2. And since f 000 (x) = 6 6= 0. Therefore, (2, f (2)) = (2, 4)
is a point of inflexion of f .

• Region of increase and decrease:.................

• Region of concavity:.............................

• Behaviour as x → ±∞:

 As x → +∞, f (x) → +∞,


 As x → −∞, f (x) → −∞.

225 MATH 122: Calculus I - Kojoga Aziedu


Figure 5.32: Graph of f (x) = x3 − 6x2 + 9x + 2

x−1
Example 5.94. Sketch the curve f (x) = .
x+2
Solution. E’s Do it!

Example 5.95. Sketch the graph of the function


3x − 5
f (x) = .
x2 − 1
Solution.
Domain: Df = {x ∈ R : x2 − 1 6= 0} = {x ∈ R : x 6= ±1}.

• Intercepts:

 y-intercept: When x = 0, we have f (0) = (0)3 − 6 (0)2 + 9 (0) + 2 = 2 ⇒ (0, 2).


Therefore, the curve crosses the y-axis at (0, 5).
 x-intercept: When f (x) = 0 ⇒ 3x − 5 = 0 ⇒ x = 35 .
Therefore, the curve crosses the y-axis at 53 , 0 .


• Critical points and their classification:

 Notice that, via partial fractions, we have


3x − 5 4 1 1 4
f (x) = 2 = − ⇒ f 0 (x) = 2 − =0
x −1 x+1 x−1 (x − 1) (x + 1)2
at the maximum/minimum point. i.e.
(x + 1)2 − 4 (x − 1)2 1
0
f (x) = 2 2 = 0 ⇒ (x + 1)2 − 4 (x − 1)2 = 0 ⇒ x = , 3.
(x − 1) (x + 1) 3
f 0 (x) = 3x2 − 12x + 9 = 3 (x2 − 4x + 3) = 3 (x − 1) (x − 3) = 0 ⇒ x = 1, x = 3. Hence, the
critical points of f are x = 1/3 and x = 3 .

226 MATH 122: Calculus I - Kojoga Aziedu


 Also,
8 2
f 00 (x) = 3 − .
(x + 1) (x − 1)3
Therefore,
      
1 8 2 81 1 1 1 9
• f 00
= 2 − 1 3 = >0⇒ ,f = , is a minimum point.
3 1
+ 1 − 1 8 3 3 3 2
3 3
 
8 2 1 1
00
• f (3) = − = − < 0 ⇒ (3, f (3)) = 3, is a maximum point.
(3 + 1)3 (3 − 1)3 8 2
 3
8 2 x−1 1
00
• f (x) = 3 − 3 = 0 ⇒ = ⇒ .......... And since f 000 (x) = ........
(x + 1) (x − 1) x+1 4
Therefore, ............. is a point of inflexion of f .

• Asymptotes: Clearly, x = ±1 are vertical asymptotes.

• Behaviours as x → ±1, x → ±∞:


 
4 1
 lim f (x) = lim − = +∞.
x→1− x→1− x+1 x−1
 
4 1
 lim f (x) = lim − = −∞.
x→1+ x→1+ x+1 x−1
 
4 1
 lim f (x) = lim − = −∞.
x→−1− x→−1− x+1 x−1
 
4 1
 lim f (x) = lim − = +∞.
x→−1+ x→−1+ x+1 x−1
3
3x − 5 − x52 0−0 0
 lim 2
= lim x
1 = = = 0+ .
x→+∞ x − 1 x→−1+ 1 − 2 1 − 0 1
x
3
3x − 5 − x52 −0 − 0 −0
 lim = lim x
1 = = = 0− .
x→−∞ x2 − 1 x→−1+ 1 − 2 1 − 0 1
x

227 MATH 122: Calculus I - Kojoga Aziedu


Figure 5.33: Graph of f (x) = 3x−5
x2 −1

Example 5.96. Sketch the graph of the function


3x2
f (x) = .
x2 − 1
Solution. Let y = f (x). Then
• Domain: Since x2 − 1 = 0 ⇒ x = ±1, Domf = {x : x ∈ R, x 6= ±1}.
• Intercepts: When x = 0, y = 0 and when y = 0, x = 0. Therefore, the intercept on both axes is
(0, 0).
• Asymptotes:
 Vertical asymptotes: The lines x = ±1 are the vertical asymptotes.
 Horizontal asymptotes:
3x2 3
lim f (x) = lim 2 = lim = 3.
x→±∞ x→±∞ x − 1 x→±∞ 1 − 12
x
Thus the line y = 3 is a horizontal asymptote.
• Behaviour of curve near the asymptotes.
3x2
lim+ f (x) = lim+ = +∞
x→1 x→1 x2 − 1
3x2
lim− f (x) = lim− 2 = −∞
x→1 x→1 x − 1
3x2
lim f (x) = lim + 2 = −∞
x→−1+ x→−1 x − 1
3x2
lim − f (x) = lim − 2 = +∞.
x→−1 x→−1 x − 1

228 MATH 122: Calculus I - Kojoga Aziedu


• Intervals of increase and decrease.
3x2 0 6x (x2 − 1) − 3x2 (2x) −6x
f (x) = ⇒ f (x) = 2 = .
2
x −1 (x2 − 1) (x2 − 1)2
Now,

Interval Sign of f 0 (x) f (x) is:


(−∞, −) + increasing
(−1, 0) + increasing
(0, 1) − decreasing
(1, ∞, ) − decreasing.

Thus f (x) increases on the intervals (−∞, −1) and (−1, 0) and decreases on (0, 1) and (1, ∞).

• Local maximum and minimum values: The critical numbers x = ±1 ∈ / Domf so we test on only
− 12 0 1
2
Sign of f 0 (x) + −
for the point x = 0 using the first derivative test as follows:
Shape of slope / \

• n

Example 5.97. For each of the following functions, find the following:

(i) domain. (v) interval of increase/decrease.

(ii) intercepts. (vi) local maximum/minimum values.

(iii) asymptotes. (vii) inflexion points and concavity.

(iv) behaviour of the function. (vii) sketch the curve.

(a) g (x) = ln (4 − x2 ). x3
(b) z = .
x2 + 1

Solution.

(a)

(b)

5.9.2 Sketching Polar Curves.


Page 731 of calculus of single variables; TK Calc II

229 MATH 122: Calculus I - Kojoga Aziedu


5.9.2.1 Polar Coordinates.

5.9.2.2 Relationship Between Polar and Cartesian Coordinates.

Negative Polar Coordinates.

5.9.2.3 Sketching Polar Curves.

5.9.2.4 Tangent at Pole.

5.9.2.5 Maximum and Minimum Values of r.

5.9.2.6 Symmetries.

Example 5.98. Two general polar curves before families.

Solution.

Example 5.99. m

Solution.

5.9.3 Families of Polar Curves.


5.9.3.1 The Rose Petal Family.

5.9.3.2 The Cardioid Family.

5.9.4 m
Example 5.100. mmmm

Solution. mmm
Example 5.101. mmmm

Solution. mmm

5.9.5 Finding Velocity of a Particle in Motion


Example 5.102. mmmm

Solution. mmm
Example 5.103. mmmm

Solution. mmm

5.9.6 Finding the Marginal Cost of a Commodity


Example 5.104. mmmm

Solution. mmm

230 MATH 122: Calculus I - Kojoga Aziedu


5.10 Exercises.
1. Determine the nature of the critical points of the following functions.

(a) f (x) = 4x3 − 3x2 − 6x + 2. (d)


(b) (e)
(c) (f)

2. Find the values of x for which the functions are increasing or decreasing?

(a) f (x) = (x + 5)4 − 3 (f) k(x) = 21 x(x2 − 3x − 9)


(b) g(x) = −x2 − 2x + 15 (g) m(x) = 4x2 + 4x − 3
(c) h(x) = 31 x3 + x2 − 8x + 1 (h) n(x) = 15x5 −4x3
(d) i(x) = −32x6 + 12x4 + x2
(i) p (x) = 2x4 −3x2 + 2x
1
(e) j (x) = 2 (j) q(x) =
x − 3x

3. Prove the following inequalities on the stated intervals.



(a) 1 + x < 1 + 12 x, = −1 < x < 0 (e) sin x < x − 16 x3 + 1
120
x5 , x>0
x−1 (f) x2 ≥ 1 + 2 ln x, x > 0
(b) < ln x < x − 1, x > 1
x (g) sin x + cos x > 2x, 0 < x < π2
(c) cos x > 1 − 12 x, x > 0  
2x 1+x 2x
(d) ex > 1 + ln (1 + x) , x ∈ R (h) < ln < , x>0
1+x 1−x 1−x

4. The height of an object attached to a spring is given by the harmonic equation


1 1
z= cos 12t − sin 12t
3 4
where z is measured in inches and t is measured in seconds.

(a) Calculate the height and velocity of the object when t = π/8 second.
(b) Show that the maximum displacement of the object is 5/12 inch.
(c) Find the period T of z. Also, find the frequency f (number of oscillations per second) if
f = 1/T .

5. Determine whether or not the following functions satisfy the three conditions in the hypothesis
of Rolle’s theorem. Hence, find all numbers c that satisfy the conclusion of the theorem.

(a) f (x) = x2 − 4x + 1 on [0, 4] (d) b



(b) f (x) = x x + 6 on [−6, 6] (e) b
(c) f (x) = sin 2πx on [−1, 1] (f) b

6. Let f (x) = 3x−5. Find the average rate of change of f between the following points.

(a) x = 0 and x = 1 (b) x = 3 and x = 7 (c) x = a and x = a + h.

What conclusion can you draw from your answers?

7. Let f (x) = ax2 + mx + b. Find the average rate of change of f between the points

231 MATH 122: Calculus I - Kojoga Aziedu


(a) x = a and x = a + h (b) x = a − h and x = a.

8. Find the average rate of change of f (x) = x3 − 2x2 + 3 on the closed interval [1, 3].

9. m

10. m

11. Find the Taylor polynomial of degree n for the following functions.

(a) f (x) = sin x about x = π


3
(c) m
(b) m (d) m

12. m

13. m

14. For each of the following functions, find the following:

(i) domain. (v) interval of increase/decrease.


(ii) intercepts. (vi) local maximum/minimum values.
(iii) asymptotes. (vii) inflexion points and concavity.
(iv) behaviour of the function. (vii) sketch the curve.

(a) f (x) = 3x5 − 5x3 + 3. x2 (x + 1)3


(d) y =
(b) (x − 2)2 (x − 4)
(c)

15. m

232 MATH 122: Calculus I - Kojoga Aziedu


Chapter 6

Antidifferentiation and Integration.

6.1 Antiderivatives.
Definition 6.1. The Antiderivative.
Let f be any given function. Then for any other function F such that
F 0 (x) = f (x)
∀x in the domain of f is called the antiderivative of f .
Theorem 6.2. If F is an antiderivative of f on an interval I , then F (x) + C is also an antiderivative
of f on I, where C is an arbitrary constant. F (x) + C is called the most general antiderivative of f on
I.

Proof. Given that F is an antiderivative if f , then


F 0 (x) = f (x) .
Now,
d d d
(F (x) + C) = (F (x)) + (C)
dx dx dx
= F 0 (x) + 0 = f (x).

Hence, every function may have infinitely many antiderivatives.


Theorem 6.3. If F and G are both antiderivatives of the continuous function f on an interval I, then
for some constant C,
G (x) = F (x) + C ∀x ∈ I.

Proof. Suppose F and G are antiderivatives of f on the interval I. Then


F 0 (x) = f (x)
and
G0 (x) = f (x) .
That is
d
G0 (x) − F 0 (x) = (G (x) − F (x)) = 0
dx
and so
G (x) = F (x) + C
for some constant C.

233
Example 6.4. Consider the function f (x) = nxn−1 , n ∈ R.To find the function F (x) such that
F 0 (x) = f (x) = nxn−1 .
But
d n d n d
(x + C) = (x ) + (C) = nxn−1 + 0 = nxn−1
dx dx dx
for some constant C. Therefore,
F (x) = xn + C.

6.2 The Indefinite Integral


Definition 6.5. The Indefinite Integral.

Let F be an antiderivative of f on the interval I. Then ∀x ∈ I, we write


F 0 (x) = f (x)
or ˆ
f (x) dx = F (x) + C (6.1)

for some constant C. The notation in (6.1) above is called the indefinite integral of f . Thus,
ˆ
f (x) = f 0 (x) dx ∀x ∈ I. (6.2)

Definition 6.6. Indefinite Integration.

The process of finding the antiderivative or the indefinite integral of a function f (x) is called integration
or integrating f (x). For indefinite integral
ˆ
f (x) dx = F (x) + C,

ˆ
• the symbol is called the integral symbol.

• the function f (x) is called the integrand of the integral.


• the variable x is called the integration variable.
• C is called the constant of integration.

6.2.1 Some Rules of Indefinite Integration.


Let F and G be antiderivatives of the functions f and g respectively on the interval I. If k and C are
constant, then ∀x ∈ I, we have
ˆ ˆ
• kf (x) dx = k f (x) dx = kF (x) + C.
ˆ ˆ ˆ
• (f (x) ± g (x)) dx = f (x) dx ± g (x) dx = F (x) ± G (x) + C.

6.2.2 Some Standard Antiderivatives.

234 MATH 122: Calculus I - Kojoga Aziedu


ˆ ˆ
(i) kdx = kx + C (vii) sin xdx = − cos x + C
ˆ ˆ
xn+1
(ii) n
x dx = + C, n 6= −1 (viii) sec2 xdx = tan x + C
n+1
ˆ ˆ
1
(iii) dx = ln x + C (ix) sec x tan xdx = sec x + C
x
ˆ ˆ
1
(iv) x x
e dx = e + C (x) √ dx = sin−1 x + C
1−x 2

ˆ ˆ
ax 1
(v) x
a dx = +C (xi) √ dx = − cos−1 x + C
ln a 1 − x2
ˆ ˆ
1
(vi) cos xdx = sin x + C (xii) dx = tan−1 x + C.
1 + x2

Example 6.7. Evaluate the following integrals.


ˆ ˆ
(a) 4x5 − 4x−5 + 2 dx (d) 3ex + 5 cos x − 3 sec2 x dx
 

ˆ  √  ˆ  
7 1 3 1
(b)
3
4 x + 3− √
2 dx (e) 2
− csc z cot z + dz
x 2 x 1+z 2z
ˆ ˆ
4z 10 − 2z 4 + 15z 2 3 cos x − 4 sin x
(c) dz (f) dx.
z3 cos2 x

Solution.

(a)
ˆ ˆ
−5
5
4x5 − 4x−5 + 2 dx
 
4x − 4x + 2 dx =
ˆ ˆ ˆ
5 −5
= 4 x dx − 4 x dx + 2 dx
4 6 4 −4
= x − x + 2x + C
6 −4
2 6 1
= x + + 2x + C.
3 x4

(b)
ˆ  √  ˆ ˆ ˆ
3 7 1 2
−3 1 1
4 x + 3− √
2 dx = 4 x 3 dx + 7 x dx − x− 2 dx
x 2 x 2
4 5 7 −2 1 1
= 5 x3 + x − x2 + C
3
−2 2
12 5 7 1√
= x3 − 2 − x + C.
5 2x 2
(c)
ˆ ˆ 
4z 10 − 2z 4 + 15z 2

7 15
dz = 4z − 2z + dz
z3 z
1 8
= z − z 2 + 15 ln |z| + C.
2

235 MATH 122: Calculus I - Kojoga Aziedu


(d)
ˆ ˆ ˆ ˆ
x 2 x
sec2 xdx

3e + 5 cos x − 3 sec x dx = 3 e dx + 5 cos xdx − 3
= 3ex + 5 sin x − 3 tan x + C.

(e)
ˆ   ˆ ˆ ˆ
3 1 1 1 1
2
− csc z cot z + dz = 3 dz +(− csc z cot z) dz + dz
1+z 2z 1 + z2 2 z
1
= 3 tan−1 z + csc z + ln |z| + C.
2

(f)
ˆ ˆ  
3 cos x − 4 sin x 3 cos x 4 sin x
dx = − dx
cos2 x cos2 x cos2 x
ˆ
= (3 sec x − 4 sec x tan x) dx
ˆ ˆ
= 3 sec xdx − 4 sec x tan xdx
= 3 ln |sec x + tan x| − 4 sec x + C.

Example 6.8. (a) The graph graph of a certain y has the slope 4x3 − 5 at the point P (x, y) and
contain the point (1, 2). Find y.

(b) A particle P travel along the x-axis such its acceleration a at time is

a (t) = t + t2 .

If it starts at the origin with the speed 2 units, determine the position of the particle when t = 4.

Solution.

(a) Given that the slope of the curve at P (x, y) is 4x3 − 5, then
ˆ
dy 3
4x3 − 5 dx = x4 − 5x + C.

= 4x − 5 ⇒ y =
dx

Now, if (1, 2) lies on the curve, then

2 = 14 − 5 (1) + C ⇒ C = 6.

Therefore,
y = x4 − 5x + 6.

(b) Let the speed of the particle at time time t be v. Then


ˆ 
dv √ 2 1
 2 1
a (t) = = t+t ⇒v = t + t dt = t3/2 + t3 + C.
2
2
dt 3 3

236 MATH 122: Calculus I - Kojoga Aziedu


If the speed at the origin is 2 units, then t = 0 ⇒ v = 2. That is
2 1
3= (1)3/2 + (1)3 + C ⇒ C = 1.
3 3
Hence,
ˆ  
ds 2 1 2 3/2 1 3 4 1 1
v= = t3/2 + t3 + 1 ⇒ s = t + t + 1 dt = t5/2 + t4 + t2 + c
dt 3 3 3 3 15 12 2
where s is the position of the particle at time t and c is a constant to be determined. Again, when
t = 0,s = 0 ⇒ c = 0.
4 1 1
∴ s (t) = t5/2 + t4 + t2 .
15 12 2
Hence,
4 1 1 4 1 1
s (4) = (4)5/2 + (4)4 + (4)2 = (32) + (256) + (16)
15 12 2 15 12 2
568
= ≈ 37.87 units.
15

Example 6.9. Determine the function f (x) if

(a) f 0 (x) = 4x3 + 2 sin x + 7ex − 9; f (0) = 15. 1


(b) f 00 (x) = 6x − ; f (1) = −2, f 0 (1) = 5.
x2
ˆ
Solution. Recall that f (x) = f 0 (x) dx. Hence,

(a) given, f 0 (x) = 4x3 + 2 sin x + 7ex − 9 and f (0) = 15, we have
ˆ
4x3 + 2 sin x + 7ex − 9 dx

f (x) =

= x4 − 2 cos x + 7ex − 9x + C.

Now, when x = 0,

f (0) = 15 = (0)4 − 2 cos (0) + 7e(0) − 9 (0) + C ⇒ 10.

∴ f (x) = x4 − 2 cos x + 7ex − 9x + 10.

1
(b) from f 00 (x) = 6x − ; f (1) = −2, f 0 (1) = 5, we have
x2
ˆ ˆ  
0 00 1
f (x) = f (x) dx = 6x − 2 dx
x
1
= 3x2 + + C.
x
But
1
f 0 (1) = 3 (1)2 + +C =5⇒C =1
1
and so
1
f 0 (x) = 3x2 + + 1.
x
237 MATH 122: Calculus I - Kojoga Aziedu
Therefore, ˆ  
1
f (x) = 3x + + 1 dx = x3 + ln x + x + C1 .
2
x
Now, f (1) = −2.
∴ −2 = (1)3 + ln (1) + 1 + C1 ⇒ C1 = −4.
Thus
f (x) = x3 + ln x + x − 4.

6.3 The Definite Integral.


Definition 6.10. The Definite Integral.
Let f be a continuous function on the closed interval [a, b]. Then the integral
ˆ b
f (x) dx
a
is called the definite integral of f from the a to b.

• [a, b] is called the interval of integration;

 a called the lower limit; and


 b called the upper limit

of the integration.

6.3.1 Some Rules of the Definite Integral.


Let f (x) and g (x) be functions defined on [a, b]. If k and r are constants and c ∈ [a, b], then
ˆ a ˆ b
• f (x) dx = 0. • kdx = k (b − a).
a a
ˆ a ˆ b ˆ b
• f (x) dx = − f (x) dx. • f (x) ≥ 0 ∀x ∈ [a, b] ⇒ f (x) dx ≥ 0.
b a
a
ˆ b ˆ b ˆ b  ˆ a
• (kf (x) ± rg (x)) dx = k f (x) dx ± r ˆ a
g (x) dx.
a a a
2 f (x) dx, if f is even
• f (x) dx =
ˆ ˆ ˆ  0
b c b
−a 0, if f is. odd.
• f (x) dx = f (x) dx + f (x) dx.
a a c

ˆ −2 ˆ 1
Example 6.11. Given that f (x) dx = 13 and g (x) dx = −1, determine the value of
1 −2
ˆ 1
(2f (x) − 3g (x)) dx.
−2

238 MATH 122: Calculus I - Kojoga Aziedu


ˆ −2 ˆ 1
Solution. f (x) dx = 13 and g (x) dx = −1.
1 −2
ˆ 1 ˆ 1 ˆ 1
∴ (2f (x) − 3g (x)) dx = 2 f (x) dx − 3 g (x) dx
−2 −2 −2
= 2 (13) − 3 (−1) = 26 + 3 = 29.

ˆ 20 ˆ 20 ˆ 1
1
Example 6.12. Let f (x) dx = 4, f (x) dx = −3 and f (x) dx = . Find the value of
ˆ 10 10 2 2 2
2f (x) dx.
1

ˆ 20 ˆ 20 ˆ 1
1
Solution. f (x) dx = 4, f (x) dx = −3 and
f (x) dx = . Now,
10 2 2 2
ˆ 10 ˆ 10 ˆ 2 ˆ 10 
2f (x) dx = 2 f (x) dx = 2 f (x) dx + f (x) dx .
1 1 1 2

But
ˆ 20 ˆ 10 ˆ 20 ˆ 10 ˆ 20 ˆ 20
f (x) dx = f (x) dx + f (x) dx ⇒ f (x) dx = f (x) dx − f (x) dx.
2 2 10 2 10 2

ˆ 10  ˆ 1 ˆ 20 ˆ 20 
∴ 2f (x) dx = 2 − f (x) dx + f (x) dx − f (x) dx
1 2 10 2
 
1
= 2 − + 4 − (−3) = −1 + 14 = 13.
2

Theorem 6.13. The Definite Integral as a Function: The FTC I.


Let f be continuous on an interval [a, b]. Then if F is an antiderivative on [a, b], then ∀x ∈ [a, b]
ˆ x
F (x) = f (t) dt (6.3)
c

or ˆ x
dF d
= f (t) dt = f (x) ∀x ∈ I
dx dx c

for some dummy variable t and c ∈ [a, b] is a constant.

Proof. By the definition of the derivative, we have


ˆ x+h ˆ x 
dF F (x + h) − F (x) 1
= lim = lim f (t) dt − f (t) dt
dx h→0 h h→0 h c c
ˆ x+h   
1 x+h−h
= lim f (t) dt = lim f (k) , k ∈ [x, x + h] by the MVT for integtrals
h→0 h x h→0 h
1
= lim (hf (k)) = lim f (a) since as h → 0, k → x
h→0 h k→x
= f (x) .

239 MATH 122: Calculus I - Kojoga Aziedu


Intuitively, Theorem 6.13, which is called the Fundamental Theorem of Calculus I (FTC I), states
that:
ˆ x
• every continuous function f is the derivative of some other function, namely f (t) dt; or
a
ˆ x
• every continuous function F (x) = f (t) dt has an antiderivative, namely f ; or
a

• the process of differentiation and integration are inverse processes of one another.

Thus, if F is an antiderivative of f on [a, b], then ∀c ∈ [a, b], we have


ˆa ˆb
• F (a) = f (t) dt. • F (b) = f (t) dt.
c c

Corollary 6.14. If g (x) and h (x) are differential functions of x, then


ˆ g(x)
d
f (t) dt = f (g (x)) g 0 (x) (6.4)
dx a
and ˆ g(x)
d
f (t) dt = f (g (x)) g 0 (x) − f (h (x)) h0 (x) . (6.5)
dx h(x)

Proof. Use the Chain rule.

Example 6.15. Differentiate each of the following.


ˆ x ˆ x4
(a) f (x) = et sin2 (1 − x) dt (c) h (x) = sec tdt
2 1
ˆ 0 3 ˆ sin x
t +1 1
(b) g (x) = dt (d) i (x) = √ dt.
x2 t+1 cos x 1 − t2

Solution.

(a)
ˆ x
0 d
f (x) = et sin2 (1 − t) dt
dx 2
= ex sin2 (1 − x) .

(b)
ˆ 0 3
0 d t +1
g (x) = dt
dx x2 t+1
2 3 3
(x ) + 1 d 2
 (x2 ) + 1
= − x = −2x
x2 + 1 dx x2 + 1
6
2x (x + 1)
= − .
x2 + 1

240 MATH 122: Calculus I - Kojoga Aziedu


(c)
ˆ tan x
0 d
h (x) = sec2 tdt
dx 1
d
= sec2 (tan x) (tan x)
dx
= sec2 (tan x) sec2 x.

(d)
ˆ sin x
0 d 1
i (x) = dt √
dx cos x 1 − t2
1 d 1 d
= p (sin x) − √ (cos x)
1 − sin2 x dx 1 − cos2 x dx
cos x sin x
= + = 1 + 1 = 2.
cos x sin x

Theorem 6.16. Fundamental Theorem of Calculus II (FTC II).


Suppose f is a continuous function of x on [a, b]. If F is an antiderivative of f on [a, b], then
ˆ b
f (x) = F (b) − F (a) . (6.6)
a

Proof. Since F is an antiderivative of f on [a, b], we have, for some c ∈ [a, b] that,
ˆ x
F (x) = f (t) dt.
c

Now, for some c ∈ [a, b], we have that


ˆ b ˆ c ˆ b
f (x) dx = f (x) dx + f (x) dx
a a c
ˆ b ˆ a
= f (x) dx − f (x) dx
c c
F (b) − F (a) ,

by definition, ending the proof.

Theorem 6.16 is called the Fundamental Theorem of Calculus II (FTC II).

Example 6.17. Evaluate the following integrals.


ˆ 2 ˆ π
h x  x i2
3
(a) x 1 + x3 dx (d)

sin + cos dx.
1 0 2 2
ˆ ˆ 2
9 2z 5 − z + 3

1 (e)
(b) √ + 1 dt
z2
dz
4 t 1

ˆ ˆ (
2 3
6, if x > 1
(c) |z| dz (f) f (x) dx, f (x) =
−2 −2 3x , if x ≤ 1.
2

241 MATH 122: Calculus I - Kojoga Aziedu


Solution.

(a)
 2
ˆ 2 ˆ 2
 x2 x4 
x 1 + x3 dx = x + x4 dx = 
 
+ 
1 1 2 4
1
22 24 12 14
 
3 21
= + − + =6− = .
2 4 2 4 4 4

(b)

ˆ ˆ
 9
9   9 √
1  1

√ + 1 dt = t− 2 + 1 dt =  2 t + t 
4 t 4
4
√  √ 
= 2 9 + 9 − 2 4 + 4 = 15 − 8 = 7.

(c) Since (
z, z≥0
|z| =
−z, z < 0,
we have
ˆ 2 ˆ 0 ˆ 2
|z| dz = − zdz + zdz
−2 −2 0
 2 0
 2 2
x x
= − +
2 −2 2 0
= 2 + 2 = 4.

(d) Notice that


h x  x i2 x x x x
sin + cos = sin2 + cos2 + 2 sin cos
2 2 2   2x  2 2
x
= 1 + 2 sin cos = 1 + sin x.
2 2

 π3
ˆ ˆ π

π
3
h x  x i2 3
∴ sin + cos dx = (1 + sin x) dx =  x − cos x 
0 2 2 0
0
π π  π 1 π 1
= − cos − 0 + cos (0) = − + 1 = + .
3 3 3 2 3 2

(e)
ˆ 2 ˆ 2
2z 5 − z + 3
 
3 1 3
dz = 2z − + 2 dz
1 z2 1 z z
 4 2
24
 4 
z 3 3 1 3
= − ln z − = − ln 2 − − − ln (1) −
2 z 1 2 2 2 1
= 8 − ln 2 − 2 + 3 = 9 − ln 2.

242 MATH 122: Calculus I - Kojoga Aziedu


(f) As in (c) above,
ˆ 3 ˆ 1 ˆ 3 ˆ 1 ˆ 3
2
f (x) dx = f (x) dx + f (x) dx = 3x dx + 6dx
−2 −2 1 −2 1
 1  3

=  x3  +  6x  = 1 + 8 + 18 − 6 = 21.
−2 1

6.4 Methods of Integration.

6.4.1 Integration by Substitution.


This is the process of reversing the Chain Rule of differentiation.

Theorem 6.18. Let u be a real-valued function which has a derivative on [a, b]. Let I be an open interval
which contains the image of [a, b] under u. Given that f is a real-valued function that is continuous on
I, if F is an antiderivative of f , then
ˆ
f (u) u0 (x) dx = F (u) + C (6.7)
ˆ
= f (u) du (6.8)

where C is an arbitrary constant.

For a definite integral, the method of substitution is


ˆ b ˆ u(b)
0
f (u) u (x) dx = f (t) dt (6.9)
a u(a)
 b

=  F (u (t))  (6.10)
a
= F (u (b)) − F (u (a)) (6.11)

and t is a dummy variable.

To use the method of substitution to evaluate a definite integral,

• m

• m

• m

Example 6.19. Evaluate the following integrals.

243 MATH 122: Calculus I - Kojoga Aziedu


ˆ ˆ √
x
(a) 2
dx (c) 2z 2 1 − 4z 3 dz
x +1
ˆ ˆ  
√ 4 5
(b) x
e 1 + ex dx (d) 3 − dx
(1 + 2x) 1 + 2x

Solution.
ˆ
x
(a) For dx, let u = x2 + 1. Then du = 2xdx or xdx = 21 du. Therefore,
x2 +1
ˆ ˆ ˆ ˆ
x 1 11 1 1
dx = xdx = 2
du = 2
du
x2 + 1 x2 + 1 u u
= 12 ln |u| + C = 12 ln x2 + 1 + C


= ln x2 + 1 + C.

(b) Let u = 1 + ex . Then du = ex dx.


ˆ ˆ ˆ
x
√ √ x

x
e 1 + e dx = x
1 + e (e dx) = udu
3
3 23
= 2
u + C = 32 (1 + ex ) 2 + C
q
= 3
2
(1 + ex )3 + C.

(c) Let u = 1 − 4z 3 ⇒ du = −12z 2 dz.


ˆ √ ˆ
2 1 √ 1
∴ 3
2z 1 − 4z dz = − udu = − u3/2 + C
6 9
1
q
= − (1 − 4z 3 )3 + C.
9

(d) Let u = 1 + 2x ⇒ du = 2dx.


ˆ   ˆ  
4 5 1 4 5
∴ − dx = − du
(1 + 2x)3 1 + 2x 2 u3 u
 
1 2
= − 2 − 5 ln |u| + C
2 u
1 5
= − 2 − ln |1 + 2x| + C.
(1 + 2x) 2

Example 6.20. Evaluate the following integrals.


ˆ 1 3 ˆ e2
4x + 1 dt
(a) 8
dx (d)
0 1+x e t ln t
ˆ 8 √ ˆ
cos x + 1 1
dx
(b) √ dx (e) √
0 x+1 0 (2 − x) 4 − x
ˆ π
2
(c) 2 + sin x2 cos x2 dx
0

244 MATH 122: Calculus I - Kojoga Aziedu


Solution.

(a) Here, we can write


ˆ 1 ˆ 1 ˆ 1
4x3 + 1 4x3 1
dx = dx + dx.
0 1 + x8 0 1 + (x4 )2 0 1 + (x4 )2
ˆ 1
4x3 h πi
Now, for dx, let u = x ⇒ du = 4x dx where u ∈ 0, . With this substitution,
4 3
4 2 2
0 1 + (x )
when x = 0, u = 0 and when x = 1, u = 1. Hence,
ˆ 1 3 ˆ 1 ˆ 1
4x + 1 1 1
dx = du + dx
8 2 4 2
0 1+x 0 1+u 0 1 + (x )
1  1
= tan−1 u 0 + tan−1 x4 0


= tan−1 (1) − tan−1 (0) + tan−1 (1) − tan−1 (0)


 
π π π
= −0+ +0= .
4 4 2

√ √
(b) Let u = x + 1. Then du = 2√x+1
1
dx ⇒ dx = 2udu. Now, when x = 0, u = 0 + 1 = 1 and

when x = 8, u = 8 + 1 = 3. Therefore,
ˆ 8 √ ˆ 3
cos x + 1 cos u
√ dx = 2udu
0 x+1 1 u
ˆ 3
 3

= 2 cos udu = 2  sin u 


1
1
= 2 (sin 3 − sin 1) .

(c) Let u = 2+sin x2 . Then du = 12 cos x2 dx, and x = 0 ⇒ u = 2+sin 0 = 2; x = π ⇒ u = 2+sin π2 = 3.


ˆ π ˆ 3 3
u3

x 2 x 2

2 + sin 2
cos 2
dx = 2 u du = 2
0 2 3 2
2 38
= (27 − 8) = .
3 3

(d) Let u = ln t ⇒ du = 1t dt; t = e ⇒ u = ln e = 1, and t = e2 ⇒ u = ln e2 = 2.


ˆ e2 ˆ e2
1 1 1
dt = · dt
e t ln t e ln t t
ˆ
 2
2
1
= du =  ln u 
1 u
1
= ln 2 − ln 1 = ln 2.

245 MATH 122: Calculus I - Kojoga Aziedu


(e) Let u = 1
2−x
. Then x = 2 − u1 and so dx = 1
u2
du. Also, when x = 0, u = 1
2
and when x = 1, u = 1.
Hence,
ˆ 1 ˆ 1 1
dx 2 du
√ dx = q u
0 (2 − x) 4 − x2 0 u 4 − 2 − u1
2
ˆ 1 1
2 du
= q u 2
1
2 u 4 − 2 − u1
ˆ 1 1 ˆ 1
u2
du 4
= q = √ du
1
2 u 4−4+ 4
− 1 1
2
4u − 1
u2 u2
 1
1 √ 1 √ 
= 2 4u − 1  = 3−1 .
4 2
1
2

6.4.2 Trigonometric Substitutions.


Theorem 6.21. Let 0 ≤ θ ≤ π2 . Given that a, b ∈ R and a 6= 0, b 6= 0, we have the following.
ˆ   ˆ  
dx 1 bx dx 1 bx
(a) √ = sin−1 + C. (c) = tan−1 + C.
a2 − b 2 x 2 b a a2 + b 2 x 2 b a
ˆ   ˆ  
1 1 bx dx 1 bx
(b) √ dx = − cos−1 + C. (d) √ = sec−1
+ C.
a2 − b 2 x 2 b a x b 2 x 2 − a2 a a

h πi a
Proof. (a) Let bx = a sin θ for θ ∈ 0, . Then bdx = a cos θdθ ⇒ dx = cos θdθ. Therefore,
2 b
ˆ ˆ ˆ
1 1 a  a cos θ
√ dx = p cos θdθ = p dθ
a2 − b 2 x 2 a2 − a2 sin2 θ b b a 1 − sin2 θ
ˆ ˆ ˆ
1 cos θ 1 cos θ 1
= √ dθ = dθ = dθ
b cos2 θ b cos θ b
1
= θ + C.
b
 
bx
for some constant C. But bx = a sin θ ⇒ θ = sin −1
. Hence,
a
ˆ  
1 1 −1 bx
√ dx = sin + C. (6.12)
a2 − b 2 x 2 b a

h πi a
(b) Let bx = a cos θ for θ ∈ 0, . Then dx = − sin θdθ.
2 b
ˆ ˆ ˆ
1 1  a  a sin θ
∴ √ dx = √ − sin θdθ = − √ dθ
a2 − b 2 x 2 a 2 − a2 cos2 θ b b a 1 − cos2θ
ˆ ˆ
1 sin θ 1
= − dθ = − dθ
b sin θ b
1
= − θ+C
b
246 MATH 122: Calculus I - Kojoga Aziedu
where C is constant. ˆ  
1 1 bx
∴ √ dx = − cos−1 + C. (6.13)
a2 − b 2 x 2 b a

π a
(c) Let cbx = a tan θ for 0 ≤ θ ≤ , so that dx = sec2 θdθ.
2 b
ˆ ˆ ˆ
1 1 a
2
 a sec2 θ
∴ dx = sec θdθ = dθ
a2 + b 2 x 2 a2 + a2 tan2 θ b ab sec2 θ
ˆ
1 1
= dθ = θ + C
b b
where C is constant. ˆ  
1 1 bx
∴ 2 2 2
dx = tan−1 + C. (6.14)
a +b x b a

a π
(d) Let bx = a sec θ ⇒ dx = sec θ tan θdθ where 0 ≤ θ ≤ . Then .
b 2
ˆ ˆ
1 1 a 
√ dx = a √ sec θ tan θdθ
x b 2 x 2 − a2 sec θ a2 sec2 θ − a2 b
b
ˆ a tan θdθ ˆ
b 1
= a √ 2 = a dθ
a tan θ
b
1
= θ+C
a
where C is constant. ˆ  
1 1 bx
∴ √ dx = sec−1 + C. (6.15)
2 2
x b x −a 2 a a

Example 6.22. Evaluate the following integrals.


ˆ ˆ
1 π/2
cos x
(a) √ dx (c) √ dx
1 − x2 0 1 + sin2 x
ˆ ˆ √
3−1
dx dx
(b) √ (d) .
4x2 − 1 −1 x2 + 2x + 2

Solution.
ˆ
1 h πi
(a) For √ dx, let x = sin θ ⇒ dx = cos θdθ for θ ∈ 0, .
1 − x2 2
ˆ ˆ ˆ
1 cos θdθ
∴ √ dx = p = dθ
1 − x2 1 − sin2 θ
= θ + C.
Now, x = sin θ ⇒ θ = sin−1 x. Hence,
ˆ
1
√ dx = sin−1 x + C.
1−x 2

247 MATH 122: Calculus I - Kojoga Aziedu


π
(b) Let 2x = sec θ, 0 ≤ θ ≤ . Then 2dx = sec θ tan θdθ. Hence,
2
ˆ ˆ
dx 1 sec θ tan θdθ
√ = √
2
4x − 1 2 sec2 θ − 1
ˆ ˆ
1 sec θ tan θdθ 1
= √ = sec θdθ
2 tan2 θ 2
1
= ln |tan θ + sec θ| + C
2
1 √
= ln 2x + 4x2 − 1 + C.

2

(c) Let u = sin x so that du = cos xdx. Now, when

x = 0 ⇒ u = sin (0) = 0

and
π π 
x= ⇒ u = sin = 1.
2 2

ˆ ˆ
 1
π/2 1
cos x du
∴ √ 2
dx = √ =  tan−1 u 
0 1 + sin x 0 1 + u2
0
π
= tan−1 (1) − tan−1 (0) = − 0
2
π
= .
2

(d) Notice that

x2 + 2x + 2 = x2 + 2x + 1 + 1
= (x + 1)2 + 1.

Thus ˆ √ ˆ √
3−1 3−1
dx dx
2 = .
−1 |x + {z
2x + 2} −1 (x + 1)2 + 1
Complete the square

Now, let u = x + 1 ⇒ du = dx and x = u − 1. Also,

x u
√ −1 √0
3−1 3
ˆ √
3−1 ˆ √
3−1 ˆ √3
dx dx du
∴ 2
= 2 = 2
−1 x + 2x + 2 −1 (x + 1) + 1 0 u +1
 −1 √3 √ 
= tan u 0 = tan−1 3 − tan−1 (0)
π
= .
3

248 MATH 122: Calculus I - Kojoga Aziedu


6.4.3 Integration by Parts.
.
.
.
.
.

6.4.4 Integration of Rational Functions: The Method of Partial Fractions.


To integrate functions of the form

f (x)
,
g (x)
where both f (x) and g (x) are polynomials in x. We look at the cases of proper and improper cases.
The following tools are needed here.

• Techniques for resolving rational functions into partial fractions.


2
4ac − b2

b
• Completing of squares of quadratic terms: ax + bx + c = a x +
2
+ .
2a 4a
• Trigonometric and hyperbolic substitution. (See subsection 6.4.5 below.)

6.4.4.1 Integration of Proper Rational Functions.

We consider the cases where the denominator has a degree of at most 2.

f (x)
• If deg (g (x)) = 1, that is when the denominator is linear, i.e. g (x) = ax + b, a 6= 0, then
g (x)
k 1
takes the form . If we make the substitution u = ax + b ⇒ dx = du, we have
ax + b a
ˆ ˆ
k k 1
dx = du
ax + b a u
k
= ln |u| + C
a
k
= ln |ax + b| + C.
a

f (x) k
• If the denominator is quadratic, i.e. deg (g (x)) = 2, then will take the form 2
g (x) ax + bx + c
kx + l
or .
ax2 + bx + c
k
 can be easily evaluated by completing the squares in the denominator and using
ax2 + bx + c
trigonometric or hyperbolic substitutions.

249 MATH 122: Calculus I - Kojoga Aziedu


kx + l
 can best be evaluated best by use of partial fractions and trigonometric sub-
ax2 + bx + c
stitutions.

Example 6.23. Evaluate ˆ


x+4
dx.
x2 − 5x + 6
x+4
Solution. First, we resolve into partial fractions as follows.
x2− 5x + 6
x+4 x+4 A B
2
= = +
x − 5x + 6 (x − 2) (x − 3) x−2 x−3
A (x − 3) + B (x − 2)
=
(x − 2) (x − 3)
⇔ A = −6, B = 7.
Hence, we can write
x+4 7 6
= − ,
x2 − 5x + 6 x−3 x−2
and so
ˆ ˆ   ˆ ˆ
x+4 7 6 7 6
dx = − dx = dx − dx
x2 − 5x + 6 x−3 x−2 x−3 x−2
= 7 ln (x − 3) − 6 ln (x − 2) + C
" #
7
(x − 3)
= ln + C.
(x − 2)6

Example 6.24. Evaluate ˆ 3


x2 + 3x + 2
dx.
1 x (x2 + 1)

x2 + 3x + 2
Solution. We can express in partial fractions as
x (x2 + 1)
x2 + 3x + 2 2 3−x
= + .
x (x2 + 1) x x2 + 1
ˆ 3 ˆ 3
x2 + 3x + 2
 
2 3−x
dx = + dx
1 x (x2 + 1) 1 x x2 + 1
ˆ 3 ˆ 3 ˆ
2 3 1 3 2x
= dx + 2
dx − dx
1 x 1 x +1 2 1 x2 + 1
 3
 
−1 1 2
= 2 ln x + 3 tan x − ln x + 1
2 1
  2
 3
x
= ln √ + 3 tan−1 x
2
x +1 1
   
9 −1 1
= ln √ + 3 tan (3) − ln √ − 3 tan−1 (1)
10 2
 
9 3π
= ln √ + 3 tan−1 (3) − .
5 4

250 MATH 122: Calculus I - Kojoga Aziedu


6.4.4.2 Integration of Improper Rational Functions.

If R (x) is improper, then we can express it as


r (x)
R (x) = q (x) + ,
g (x)
for some polynomials q (x) and r (x). In this case, the integral becomes
ˆ ˆ  
f (x) r (x)
dx = q (x) + dx
g (x) g (x)
ˆ ˆ
r (x)
= q (x) dx + dx.
g (x)
Example 6.25. Evaluate ˆ
x2 + 1
dx.
x+1
Solution. Notice that this function is improper since deg (x2 + 1) = 2 > 1 = deg (x + 1). But
x2 + 1 2
=x−1+ ,
x+1 x+1
using long division. Hence,
ˆ ˆ 
x2 + 1

2
dx = x−1+ dx
x+1 x+1
ˆ ˆ
2
= (x − 1) dx + dx
x+1
ˆ ˆ d
dx
(x + 1)
= (x − 1) dx + 2 dx
x+1
= 12 x2 − x + 2 ln (x + 1) + C.

Example 6.26. Evaluate ˆ


x3 + 2x2
dx.
x2 + 1
Solution. Again the integrand is improper, and by long division, we have
x3 + 2x2 x+3
= x + 3 − .
x2 + 1 x2 + 1
Therefore,
ˆ ˆ 
x3 + 2x2

x+3
dx = x+3− 2 dx
x2 + 1 x +1
ˆ ˆ
x+3
= (x + 3) dx − dx
x2 + 1
ˆ ˆ ˆ
x 3
= (x + 3) dx − dx − dx,
x2 + 1 x2 + 1
ˆ ˆ d 2 ˆ
1 dx
(x + 1) 1
= (x + 3) dx − 2 2
dx − 3 2
dx
x +1 x +1
x + 3x − 21 ln x2 + 1 − 3 tan−1 x + C
1 2

= 2

= 1 2
2
x + 3x − ln x2 + 1 − 3 tan−1 x + C.

251 MATH 122: Calculus I - Kojoga Aziedu


6.4.5 Integration of Trigonometric Functions.
6.4.5.1 Basic Trigonometric Integrals.
ˆ ˆ
• cos xdx = sin x + C • csc cot xdx = − csc x + C
ˆ ˆ
1
• tan xdx = ln |sec x| + C = − ln |cos x| + C • cos (ax + b) dx = sin (ax + b) + C
a
ˆ ˆ
2 1
• sec xdx = tan x + C • sin (ax + b) dx = − cos (ax + b) + C
a
ˆ ˆ
2 1
• csc xdx = − cot x + C • tan (ax + b) dx = − ln |cos (ax + b)| + C
a
ˆ ˆ
1
• sec x tan xdx = sec x + C • cot (ax + b) dx = ln |sin (ax + b)| + C
a
ˆ  x π 
• sec xdx = ln |sec x + tan x| + C = ln tan + +C

2 4
ˆ
1
• sec (ax + b) dx = ln |sec (ax + b) + tan (ax + b)| + C
a
ˆ
1
• csc (ax + b) dx = − ln |csc (ax + b) + cot (ax + b)| + C
a

6.4.5.2 Products of Trigonometric Functions.

To evaluate integrals of the forms


ˆ ˆ ˆ
sin mx cos nxdx, sin mx sin nxdx, cos mx cos nxdx,

where m 6= n, we make use of the following trigonometric identities:

sin x cos y = 1
2
(sin (x + y) + sin (x − y)) (6.16)
sin x sin y = 1
2
(cos (x − y) − cos (x + y)) (6.17)
cos x cos y = 1
2
(cos (x − y) + cos (x + y)) . (6.18)

Example 6.27. Find


ˆ ˆ ˆ
(a) sin 3x cos 5xdx (b) sin x sin 2xdx (c) cos 15x cos 4xdx.

Solution.

(a) sin 3x cos 5x = 12 (sin 8x + sin 2x).


ˆ ˆ
1
∴ sin 3x cos 5xdx = (sin 8x + sin 2x) dx
2

= 12 − 81 cos 8x − 12 cos 2x + C

1
= − 16 cos 8x − 41 cos 2x + C.

252 MATH 122: Calculus I - Kojoga Aziedu


(b) sin x sin 2x = 12 (cos x − cos 3x).
ˆ ˆ
1
∴ sin x sin 2xdx = (cos x − cos 3x) dx
2

= 21 sin x − 13 sin 3x + C


= 21 sin x − 16 sin 3x + C.

(c) cos 15x cos 4x = 21 (cos 11x + cos 19x).


ˆ ˆ
1
∴ cos 15x cos 4xdx = 2
(cos 11x + cos 19x) dx
1
= 12 11 1

sin 11x + 19 sin 19x + C
1 1
= 22 cos 11x + 38 cos 19x + C.

6.4.5.3 Powers of Trigonometric Functions.

• Integrals of Powers of Sines and Cosines.


Consider integrals of the form ˆ
sinm x cosn xdx.

 Case I: Either m or n is Odd or Both are Odd.


Suppose n is odd, that is n = 2k + 1, k ∈ N. Then

cosn x = cos2k x cos x


k
= cos2 cos x
k
= 1 − sin2 cos x,

so that
ˆ ˆ
m n
k
sin x cos xdx = sinm x 1 − sin2 cos xdx.

Let u = sin x. Then du = cos xdx and the integral becomes


ˆ ˆ
m n
k
sin x cos xdx = um 1 − u2 du.

The integrand in the last integral is a polynomial in u and so can be easily evaluated.

Similarly, if m is odd, i.e. m = 2k + 1, then

ˆ ˆ
m n
k
sin x cos xdx = − 1 − u2 un du, u = cos x.

253 MATH 122: Calculus I - Kojoga Aziedu


 Case II: Both m and n are Even.
If both m and n are even, that is m = 2k and n = 2r, k, r ∈ N. Then as before, we can
make use of the following identities:
sin2 x = 1
2
(1 − cos 2x) (6.19)
cos2 x = 1
2
(1 + cos 2x) , (6.20)
and
ˆ ˆ
m n
sin x cos xdx = sin2k x cos2r xdx, k, r ∈ N
ˆ
k r
= sin2 x cos2 x xdx
ˆ
1
k 1 r
= (1 − cos 2x)
2 2
(1 + cos 2x) xdx
ˆ
1 1
= k · r (1 − cos 2x)k (1 + cos 2x)r dx
2 2
ˆ
1
= k+r (1 − cos 2x)k (1 + cos 2x)r dx. (6.21)
2
We shall be relying on binomial theorem to expand the factors (1 − cos 2x)k and (1 + cos 2x)r .

Example 6.28. Evaluate the following integrals.


ˆ ˆ
(a) 3 2
sin x cos xdx (b) sin2 x cos2 xdx.

Solution.
(a) Notice that
ˆ ˆ
3 2
sin x cos xdx = sin x sin2 x cos2 xdx
ˆ
1 − cos2 x cos2 x sin xdx.

=

now, let u = cos x so that du = − sin xdx.


ˆ ˆ
3 2
1 − u2 u2 du

∴ sin x cos xdx = −
ˆ
u2 − u4 du

= −

= − 31 u3 + 15 u5 + C
= − 13 cos3 x + 51 cos5 x + C.

(b)
ˆ ˆ  
2 2 1 1
sin x cos xdx = (1 − cos 2x) · (1 + cos 2x) dx
2 2
ˆ ˆ
1 2 1
sin2 2xdx

= 1 − cos 2x dx =
4 4
ˆ  
1 1 1 1
= (1 − cos 4x) dx = x − sin 4x + C.
4 2 8 4

254 MATH 122: Calculus I - Kojoga Aziedu


• Integrals of Powers of Tangents and Secants.
Consider integrals of the form ˆ
tanm x secn xdx.

 Case I: n is Even.
If n = 2k, k ∈ N, then we can use the identity
sec2 x = 1 + tan2 x
and make the substitution u = tan x. Then du = sec2 xdx and
ˆ ˆ
m n
tan x sec xdx = tanm x secn−2 x sec2 xdx
ˆ
= tanm x sec2k−2 x sec2 xdx, n = 2k; k ∈ N
ˆ
k−1
= tanm x sec2 x sec2 xdx
ˆ
k−1
= tanm x 1 + tan2 x sec2 xdx
ˆ
k−1
= um 1 + u2 du, u = tan x

which again reduces to a polynomial in u and can be easily evaluated.

 Case II: m is Odd or Both m and n are Odd.


Suppose m = 2k + 1, k ∈ N, then we can make the substitution u = sec x ⇒ du =
sec x tan xdx and use the identity
tan2 x = sec2 x − 1.
Hence,
ˆ ˆ
m n
tan x sec xdx = tan2k+1 x secn xdx
ˆ
= tan2k x secn−1 x sec x tan xdx, n = 2k; k ∈ N
ˆ
k
= sec2 x − 1 secn−1 xdu
ˆ
k
= u2 − 1 un−1 du, u = sec x

which also having a polynomial integrand in u.

 Case III: m is Even and n is Odd.


If m is even and n is odd, use the identity
tan2 x = sec2 x − 1
and use integration by parts (see Subsection ??) or a other methods to integrate seck x,
k ∈ N.

Example 6.29. Evaluate the following integrals.

255 MATH 122: Calculus I - Kojoga Aziedu


ˆ ˆ ˆ
(a) 9 5
sec x tan xdx. (b) 4 6
sec x tan xdx. (c) tan3 xdx.

Solution.

(a) Since both m = 5 and n = 9 are odd, we can write


ˆ ˆ
9 5
sec x tan xdx = sec8 x tan4 x sec x tan xdx
ˆ
2
= sec8 x sec2 x − 1 sec x tan xdx

and let u = sec x ⇒ sec x tan xdx so that


ˆ ˆ
9 5
2
sec x tan xdx = u8 u2 − 1 du
ˆ
u8 u4 − 2u2 + 1 du

=
ˆ
u12 − 2u10 + u8 du

=
1 13 2 1
= u − u11 + u9 + C
13 11 9
1 2 1
= sec13 x − sec11 x + sec9 x + C.
13 11 9

(b)
ˆ ˆ
4 6
sec x tan xdx = sec2 x tan6 x sec2 xdx
ˆ
1 + tan2 x tan6 x sec2 xdx.

=

Let u = tan x ⇒ sec2 xdx so that


ˆ ˆ
4 6
1 + u2 u6 du

sec x tan xdx =
ˆ
u2 + u8 du

=
1 3 1 9
= u + u +C
3 9
1 1
= tan3 x + tan9 x + C.
3 9

(c) Here, we write


ˆ ˆ
3
tan xdx = tan2 x tan xdx
ˆ
sec2 x − 1 tan xdx

=
ˆ ˆ
2
= tan x sec xdx − tan xdx.

256 MATH 122: Calculus I - Kojoga Aziedu


ˆ
For tan x sec2 xdx, let u = tan x ⇒ sec2 xdx. Therefore,
ˆ ˆ ˆ
3 − sin x
tan xdx = udu + dx
cos x
1 2
= u + ln |cos x| + C
2
1
= tan2 x + ln |cos x| + C.
2

ˆ ˆ
Alternatively, we can write 2
tan x sec xdx = sec x sec x tan xdx and let u = sec x ⇒
du = sec x tan xdx
ˆ ˆ
2
∴ tan x sec xdx = udu
1
= sec2 x + C.
2

• Integrals of Powers of Cotangents and Cosecants.


Integrals of the form ˆ
cotm x cscn xdx

are handled similarly to those of the form


ˆ
tanm x secn xdx.

Hence, we have the following scenarios.

 Case I: n is Even.
If n is even, make the substitution u = cot x, where du = − csc2 xdx and use the identity

csc2 x = cot2 x + 1.

That is,
ˆ ˆ
m n
cot x csc xdx = cotm x csc2k xdx, k ∈ N
ˆ
= cotm x csc2k−2 x csc2 xdx
ˆ
k−1 2
= cotm x cot2 x + 1 csc xdx
ˆ
k−1
= − um u2 + 1 du, u = cot u.

 Case II: m is Odd.


If m is odd, make the substitution u = csc x, where du = − csc x cot xdx and use the identity

cot2 x = csc2 x − 1.

257 MATH 122: Calculus I - Kojoga Aziedu


That is,
ˆ ˆ
m n
cot x csc xdx = cot2k+1 x cscn xdx, k ∈ N
ˆ
= cot2k x cscn−1 x cot x csc xdx
ˆ
k
= csc2 x − 1 cscn−1 x cot x csc xdx
ˆ
k
= − u2 − 1 un−1 du, u = csc u.

Example 6.30. Evaluate the following integrals.


ˆ ˆ
cot3 x
(a) cot x csc7 xdx (b) dx.
csc x

Solution.

(a) Notice that cot x csc7 x = csc6 x cot x csc x. Let u = csc x so that du = − cot x csc xdx.
ˆ ˆ ˆ
7
∴ cot x csc xdx = csc x cot x csc xdx = − u6 du
6

1
= − u7 + C.
7
That is
ˆ
1
cot x csc7 xdx = − csc7 x + C.
7

(b)

258 MATH 122: Calculus I - Kojoga Aziedu


6.5 Exercise.
1. m

2. m

3. m

4. m

5. Integrate some of these

(a)

6. m

259 MATH 122: Calculus I - Kojoga Aziedu


7. Evaluate the following integrals.
ˆ  √ ˆ  
4
(a) t + t 4 − t2 dt
2
(c)

√ + 2 cos 2x dx
ˆ 1 − 16x2
dx
(b)
(d) m
p
x |x| ln2 x − 1

8. m

9. Evaluate the following integrals.


ˆ ˆ 0 √
(a) dt (d) 2z 2 1 − 4z 3 dz
ˆ ln(1+π) −2
ˆ −6
(b)
 
(ex cos (1 − ex )) dx 4 5
0 (e) 3 − dx
ˆ 3 −2 (1 + 2x) 1 + 2x
(c) |3z − 5| dz (f) m
0

10. m

11. m

12. Integrate the following functions with respect to x.


x−1 (d) m
(a)
x2 (x + 2)
(e) m
x
(b) 2 (f) m
x −x−2
(g) m
3 (x + 1)
(c) 2 (h) m
x − 3x − 4

13. m

14. m

260 MATH 122: Calculus I - Kojoga Aziedu


Chapter 7

Applications of Integrations.

7.1 Area Under a Curve.


Let R denote the region lying between the curves

y = f (x)

and
y = g (x)
and the lines x = a and x = b. Then the area A of the region R bounded by the curves y = f (x),
y = g (x) and the lines x = a and x = b, given that f (x) ≥ g (x) for a ≤ x ≤ b, is given by
ˆ b
A= [f (x) − g (x)] dx. (7.1)
a

Figure 7.1: The area bounded by the curves y = f (x), y = g (x) and the lines x = a, x = b.

Deductions.

• Area Between the Curve, the x-axis and the Lines x = a, x = b.

261
Figure 7.2: The area bounded by the curves y = f (x), y = g (x) and the lines x = a, x = b.

ˆ b
A= f (x) dx. (7.2)
a

• Area Between the Curve, the y-axis and the Line x = b.

Figure 7.3: The area bounded by the curves y = f (x), y = g (x) and the lines x = a, x = b.

ˆ b
A= f (x) dx. (7.3)
0

• Do for x = g (y) ..........

Example 7.1. mmmm

Solution. mmm

Example 7.2. Find the area of the region bounded by the curves y = x2 + 1 and y = 4 − x2 , and the
lines x = −1 and x = 1.

Solution.

262 MATH 122: Calculus I - Kojoga Aziedu


Figure 7.4: Region between the curves y = x2 + 1, y = 4 − x2 and the lines x = ±1.

The required area is


ˆ 1 ˆ 1
2 2
3 − 2x2
    
A = 4−x − x +1 dx = dx
−1 −1
 1
14
=  3x − 32 x3  = sq. units.
3
−1

Example 7.3. Find the area of the region bounded enclosed by the parabolas y = x2 and y = x (2 − x).

Solution. First, we find the point of intersection of the two parabolas by solving them simultaneously.
Thus, solving
y = x2 , y = 2x − x2
simultaneously, we have
x2 = 2x − x2 ⇒ 2x (x − 1) = 0 ⇒ x = 0, x = 1.

Figure 7.5: Region between the curves y = x2 and y = x (2 − x).

Hence, the required area is


ˆ 1
´ ˆ 1
´
2 2
x − x2 dx
   
A = 2x − x
− x dx = 2
0 0
 1
1
= 2  21 x2 − 13 x3  = sq. units.
3
0

263 MATH 122: Calculus I - Kojoga Aziedu


Example 7.4. Find the area of the region bounded by the curves y = sin x, the y = cos x, x = 0 and
x = π2 .

Solution. The two curves y = sin x and y = cos x when


π
sin x = cos x ⇒ tan x = 1 ⇒ x =
4
for x ∈ 0, π2 .
 

Figure 7.6: Region between the curves y = cosx and y = sin x.

Hence, the required area is

A = A1 + A2
ˆ π ˆ π
4 2
= [cos x − sin x] dx + [sin x − cos x] dx
π
0 4
  π4   π2

=  sin x + cos x  +  − cos x − sin x 


π
0
   4 
1 1 1 1
= √ +√ − (0 + 1) + (−0 − 1) − − √ − √
2 2 2 2
√ 
= 2 2 − 1 sq. units.

Example 7.5. Find the area enclosed by the line y = x − 1 and the parabola y 2 = 2x + 6.

Solution.

Example 7.6. Given the curve (


4, if x > 2
f (x) =
x − 1, if x ≤ 2,
2

find the area enclosed by f (x), the x- and y-axis, and the line x = 6.

Solution.

264 MATH 122: Calculus I - Kojoga Aziedu


7.2 Volume of Solid (Surface) of Revolution.
Definition 7.7. Solid of Revolution.
A solid that has a central axis of symmetry is called a solid of revolution. Examples are the cone,
cylinder, etc.

3π π
The volume of revolution of a solid is formed when a solid is rotated through an angle, such as 2π, , ,
2 2
etc around an axis.

Figure 7.7: Volume of revolution formed by rotating y = f (x) between y = 0 and the lines x = a, x = b
through 2π.

The volume formed by rotating the region R enclosed by the curve y = f (x), the x-axis and the lines
x = a and x = b, through an angle of 2π is given
ˆ b
V = πy 2 dx. (7.4)
a

Deductions.

• The volume formed by rotating the region R enclosed by the curve x = g (y), the y-axis and the
lines y = c and y = d, through an angle of 2π is given
ˆ d
V = πx2 dy. (7.5)
c

Figure 7.8: Volume of revolution formed by rotating x = f (y) between x = 0 and the lines y = c, y = d
through 2π.


• What of thru 2
, etc?

• Parametric form page726 of calculus of single variables

Example 7.8. mmmm

Solution. mmm

Example 7.9. Find the area of the region bounded by the curves y = x2 + 1 and y = 4 − x2 , and the
lines x = −1 and x = 1.

Solution.

Figure 7.9: Region between the curves y = x2 + 1, y = 4 − x2 and the lines x = ±1.

265 MATH 122: Calculus I - Kojoga Aziedu


7.2.0.4 Volume of Surface of Revolution About the x-axis.

Example 7.10. mmmm

Solution. mmm

Example 7.11. mmmm

Solution. mmm

7.2.0.5 Volume of Surface of Revolution About the x-axis.

Example 7.12. mmmm

Solution. mmm

Example 7.13. mmmm

Solution. mmm

7.2.0.6 Volume of Surface of Revolution About the Any Given Axis.

Example 7.14. mmmm

Solution. mmm

Example 7.15. mmmm

Solution. mmm

266 MATH 122: Calculus I - Kojoga Aziedu


7.3 Exercise.
1. m

2. m

3. Find the area bounded by the following curves.

(a) y 2 = x − 4, y 2 = 12 x (c) y = x2 − 2x, y − x + 4


(b) x = 1 − y 2 , x = y 2 − 1 (d) .

4. m

267 MATH 122: Calculus I - Kojoga Aziedu


Chapter 8

First Order Ordinary Differential Equations.

8.1 Definitions and Terminologies.


Definition 8.1. Differential equations
A differential equation is an equation involving an unknown function and one or more of its derivatives.
The study of differential equations helps us to:

• discover the (differential) equations that describes a specified physical situation; and

• find the appropriate solution of that equation.

Definition 8.2. Solution of a differential equation.


The function y = u (x) is said to be a solution of the differential equation

f x, y, y 0 , . . . , y (n) = 0 (8.1)


on the interval I provided the derivatives u0 , u00 , . . . , u(n) exist on I and

f x, u, u0 , . . . , u(n) = 0 ∀x ∈ I. (8.2)


We also say y = u (x) satisfies (8.1).

Example 8.3. The function


y (x) = A cos x + B sin x (8.3)
is a solution of the differential equation y 00 + 9y = 0 for all real x since

y 0 (x) = −3A sin x + 3B cos x


y 00 (x) = −9A cos x − 9B sin x
= −9y (x) ∀x ∈ R.

Thus, the function (8.3) satisfies y 00 + 9y = 0.

Definition 8.4. General solution.


A solution of a differential equation is called a general solution if it contains arbitrary constants (or
parameters), and for every choice of these constants we obtain a solution. An nth order differential
equation may have a solution which contains n constants, and called an n-parameter family of solutions.

Example 8.5. The function (8.3) is a 2-parameter family of solution of y 00 + 9y = 0 since it contains
two arbitrary constants A and B.

268
Definition 8.6. Particular solution.
A solution of a differential equation is called a particular solution if it does not contain arbitrary
constants. For example, the function y (x) = cos x + sin x is a particular solution of y 00 + 9y = 0 if
A = B = 1.

Definition 8.7. System of differential equations.


A system of differential equations arises when there are two or more unknown functions dependent on
a single independent variable. An example is
df
= af − mf g,
dx
dg
= −bg + nf g,
dx
where a, b, m, n are constants.

Definition 8.8. Initial-value problems (IVPs).


An initial-value problem is a system consisting of the differential equation together with some conditions
imposed on the dependent variable and/or its derivatives at some point x0 in some interval I.
That is an IVP is the problem of finding the solution of the equation f x, y, y 0 , . . . , y (n) = 0 given that


y (i) (x0 ) = yi , i = 0, 1, . . . , n − 1.
Examples of IVPs are

• y 0 − y = 0; y (0) = 2
• y 00 + 9y = 0; y 0 (0) = 3, y (0) = 1.

8.2 Classifications of Differential Equations.

We can classify differential equations by type, order, or linearity:

8.2.1 Classification by Type.


Classification by type concerns the type of derivatives contained in the equation. A differential equation
is called an ordinary differential equation (ode) if the unknown function(s) (or dependent variables)
depend(s) only on a single independent variable, so that ordinary derivatives appear in the equation,
eg
dy
− 13y = ln x,
dx
d2 y dy
2
+ − cos y = ex .
dx dx
A differential equation is called a partial differential equation (pde) if dependent variable(s) is/are a
function of two or more independent variables, so that partial derivatives appear in the equation, eg
∂f ∂f
=− ,
∂x ∂y
∂ 2f 1 ∂f
2
= .
∂ x k ∂t

269 MATH 122: Calculus I - Kojoga Aziedu


8.2.2 Classification by Order.
The order of a differential equation is the order of the highest derivative that appears in it. For example
the differential equation
dy
• − 13y = ln x is a first order ordinary differential equation;
dx
∂ 2f 1 ∂f
• 2
= is a second order partial differential equation.
∂ x k ∂t

The general nth order ordinary differential equation with independent variable x and unknown function
y = f (x) can be represented as
F x, y, y 0 , y 00 , . . . , y (n−1) , y (n) = 0, (8.4)


dn y
where y (n) ≡ and F is a specific real-valued function of n + 2 variables.
dxn

8.2.3 Classification by Linearity.


An nth order ordinary differential equation is said to be linear if it can be written in the form
0
X di y
ai (x) i = g (x) . (8.5)
i=n
dx
That is, the
di y
• unknown function y and all of its derivatives , i = n, n − 1, . . . , 1, 0 are of degree 1;
dxi
• coefficients of all these terms are functions of the independent variable only; and

• equation does not contain any transcendental functions nor their derivatives of the dependent
variable, such as sin y, ln y, etc.
An ordinary differential equation which is not linear is said to be nonlinear.

8.2.4 Formation of differential equations


See Subsection 4.8.

8.3 First-order Differential Equations.


The general form of a first order linear ordinary differential equation is
dy
+ p (x) y = q (x) (8.6)
dx
where p (x) and q (x) are continuous functions on a given interval I. The equation (8.6) is called
homogeneous if q (x) = 0, otherwise it is called nonhomogeneous.
In the case where q (x) ≡ 0 the general form of a first order ordinary differential equation reduces to
dy
= F (x, y) . (8.7)
dx
This equation can be linear or nonlinear; we shall consider for now the linear cases.

270 MATH 122: Calculus I - Kojoga Aziedu


8.3.1 Methods of Solving First Order Differential Equations.
Here, we seek to find a solution of equation (8.7) via the following methods.

8.3.1.1 Separation of Variables Method.

Definition 8.9. Separable First Order Differential Equation.


A separable differential equation first order differential equation is a first-order differential equation in
dy
which the expression for can be written in the form
dx
dy
= f (x) g (y) . (8.8)
dx

To solve (8.8), we rewrite it in the form


dy
= f (x) dx (8.9)
g (y)
and integrate both sides to get the required solution:
ˆ ˆ
1
dy = f (x) dx.
g (y)

In particular, if f is independent of y, that is, if F (x, y) = f (x), then (8.8) becomes


dy
= f (x)
dx
or
dy = f (x) dx (8.10)
so that ˆ ˆ
dy = g (x) dx
or ˆ
y= g (x) dx + c.

Also, if on the other hand f is independent on x for which F (x, y) = g (y), then (8.8) becomes
dy
= h (y)
dx
or
dy
= dx, (8.11)
h (y)
and ˆ ˆ
dy
= dx
h (y)
or
ˆ
dy
x = + c.
h (y)
In both cases, that is in equations (8.10) and (8.11), we have obtained equations in which the variables
x and y appear on separate sides, and they are said to be thus separated. Equations of this nature are
called separable differential equations.

271 MATH 122: Calculus I - Kojoga Aziedu


Example 8.10. Solve the initial value problem
dy
= −6xy; y (0) = 7.
dx
Solution: We rewrite the equation as
dy
= −6xdx
y
which becomes separable in x and y. Now, integrating both sides, we have
ˆ ˆ
dy
= − 6xdx ⇒ ln |y| = −3x2 + c.
y

From the initial condition, y (x) > 0 near x = 0, so we may ignore the absolute value symbol:

ln y = −3x2 + c,
2 2 2
∴ y (x) = e−3x +c = ec e−3x = Ae−3x ; A = ec .

Hence, when y (0) = 0, we get A = 7, so the desired solution is


2
y (x) = 7e−3x .

Example 8.11. Solve the equation


(1 + x) dy − ydx = 0.

Solution:
dy dx
(1 + x) dy − ydx = 0 ⇒ =
y 1+x
which is again separated, hence
ˆ ˆ
dy dx
= ⇒ ln y = ln (1 + x) + ln c
y 1+x
= ln (c (1 + x)) ⇔ y = c (1 + x) .

Example 8.12. The Monod equation for conversion of a chemical substrate of concentration C into
its products at any time t is
dC aC
=
dt b+C
where a and b are positive constants. This equation, with Michaelis–Menten kinetics, describes how
the substrate is being used up through chemical reaction. If the initial concentration of the substrate
b
is 1 molar, determine, in terms of a and b, the concentration of the substrate at t = s.
2a

Solution. Separating variables, we have


   
b+C b
dC = 1 + dC = adt
C C
⇒ C + b ln C = at + D.

But C (0) = 1 ⇒ D = C.
 
at
∴ C + b ln C = at + C ⇒ C = exp .
b

272 MATH 122: Calculus I - Kojoga Aziedu


b
Now, when t = , we obtain
2a

  
a b
C = exp = e M.
b 2a

Example 8.13. m

Solution.

8.3.1.2 Integrating Factor Method: Solution of Nonhomogeneous Linear Ordinary Dif-


ferential Equations

To solve a nonhomogeneous (and even homogeneous) first order linear ordinary differential equation, the
method of use of integrating factors is most appropriate. What this method seeks to do is to rewrite the
left-hand side of (8.6) as the derivative of a certain product. Hence, we look out for a certain function
of x, called the integrating factor , which when we multiply through (8.6) with it guarantees that
the left-hand side can be recognised as a derivative. This is done by following the steps below:

• Calculate the integrating factor,which we shall denote here by µ (x), as


´
µ (x) = e p(x)dx
. (8.12)

• Multiply each side of the differential equation by µ (x).


dy
µ (x) + p (x) µ (x) y = µ (x) q (x)
dx

• Recognise and rewrite the LHS of the resulting equation as the derivative of a product, that is
d
[µ (x) y (x)] = µ (x) q (x) .
dx

• Finally, integrate this equation to get


ˆ
µ (x) y (x) = µ (x) q (x) dx + c,

and solve for y (x) to obtain the general solution of the original differential equation, i.e.
ˆ 
1
y (x) = µ (x) q (x) dx + c .
µ (x)
Example 8.14. Solve the initial value problem
dy
− 3y = e2x ; y (0) = 3.
dx
Solution: Here, we have p (x) = −3 and q (x) = e2x , so the integrating factor is
´
µ (x) = e− 3dx
= e−3x .

273 MATH 122: Calculus I - Kojoga Aziedu


Multiplying both sides of the given equation, we have
dy
e−3x − 3e−3x y = e−x
dx
which we can rewrite as
d
ye−3x = e−x .

dx
Hence, integration wrt x gives

ye−3x = −e−x + c ⇒ y (x) = ce3x − e2x .

Now, when x = 0 and y = 3, we have c = 4. Therefore, the desired particular solution is

y (x) = 4e3x − e2x .

Example 8.15. m

Solution.

8.4 Applications of First Oder Differential Equations.

8.4.1 Growth and Decay.


Page 184 of Adams, Single variable 415.
.
.

Example 8.16. m

Solution.

Example 8.17. m

Solution.

274 MATH 122: Calculus I - Kojoga Aziedu


8.4.2 Inductance-Resistance Electrical Circuits
Example 8.18. In an simple electric circuit system the electromotive force produces a voltage of E (t)
volts (V) and a current of I (t) ampères (A) at time t. If the resistance and the inductance in the
circuit are R ohms (Ω) and L henries (H) respectively, one of Kirchhoff’s laws states that the sum of
the voltage drops of RI due to the resistor and L dI
dt
due to the inductor is equal to the supplied voltage
E (t); that is,
Given that the current at time t = 0 is zero,

(a) determine the current at time t.

(b) What is the limiting value of the current?

(c) Suppose that a resistor of 3 Ω and an inductor of 9 H are in the circuit together with with a
constant voltage of 60 V, obtain I (t), the current after 10 s and the limiting value of the current.

Solution.

(a)
dI dI R E
L + RI = E =⇒ + I= (8.13)
dt dt L L

which is in the form of (8.6), i.e. p (t) = R


L
. Hence, the integrating factor is
´ R R
dt
µ (t) = e L = e L t.

Therefore, we can write (8.13) as


R dI R R t E R
eLt + eL I = eLt
dt L L
or
d  Rt  E Rt
eL I = eL ,
dt L
i.e.
ˆ
R
t E R
e I =
L e L t dt
L
E Rt
= eL + k
R
E R
∴ I (t) = + ke− L t .
R
Now, when I (0) = 0 =⇒ k = −E/R.
E E −Rt E  R

∴ I (t) ∴= − e L = 1 − e− L t .
R R R

(b) The limiting value of I is given by


E R

lim I (t) = lim 1 − e− L t
t→∞ t→∞ R
E 
−R t

= lim 1 − e L
R t→∞
E
= .
R

275 MATH 122: Calculus I - Kojoga Aziedu


(c)

Example 8.19. m

Solution.

8.4.3 Motion of a Particle Under Gravity With Air Resistance


Example 8.20. Suppose an object of mass m kg falling near the surface of the earth is retarded by a
air resistance proportional to its velocity v = v (t) at any given time t. Then Newton’s second law of
motion states that
dv
m = mg − kv, (8.14)
dt
where g is the acceleration due to gravity near the surface of the earth and k is a constant. If the
object starts from rest, find the velocity for any t > 0 and show that v (t) approaches a limiting value
as t → ∞.
Solution:
dv dv k
m = mg − kv =⇒ + v = g,
dt dt m
so that ´ k kt
dt
µ (t) = e m = em .
Therefore,
ˆ
kt kt mg kt
ve m = g e m dt = em + c
k
mg kt
=⇒ v (t) = + ce− m .
k
mg
Hence, since v (0) = 0, we have that c = − .
k
mg  kt

∴ v (t) = 1 − e− m ,
k
and
mg  kt
 mg
lim v (t) = lim 1 − e− m = .
t→∞ t→∞ k k

Example 8.21. m

Solution.

Example 8.22. m

Solution.

276 MATH 122: Calculus I - Kojoga Aziedu


8.4.4 Predator-prey Systems.
Example 8.23. m

Solution.

Example 8.24. m

Solution.

8.4.5 m
Example 8.25. m

Solution.

Example 8.26. m

Solution.

277 MATH 122: Calculus I - Kojoga Aziedu


Bibliography

[1] Krantz

278

You might also like